You are on page 1of 273

AS PER NEW ICAP EDUCATION SCHEME

ICAP CAF-02

TAX
PRACTICES
INCLUDING TOPICAL ICAP PAST PAPERS

TARIQ TUNIO
Tax Commissioner
FBR/SRB

Masters in
Tax Management
from IBA Karachi

Teacher of Tax, BL, CL & ItB

Founder of STT Solutions, ICTAP & PTAX Software

Co-founder ARTT Business School

STT Publications

1
CAF-02 Tax Practices

ICAP CAF 02

TAX PRACTICES
By TARIQ HUSSAIN TUNIO
© 2023 SAIMA TARIQ

Updated upto Finance Act 2023

All rights reserved. All content in this book are original and prepared by
the author - Sir Tariq Tunio. No part of this publication may be
reproduced, copied, translated, stored in a retrieval system, or
transmitted, in any way or by any means, or used in class room or tests
or exams by anyone, including faculty members, without prior
permission in writing of the Author.

All efforts have been made to avoid errors in this publication. Inspite of
that if any errors or discrepancy is found, author welcomes students to
point it out to him.

Contact : 0332-2130867

2
Tariq Hussain Tunio

S.No. Chapter Name Page No.


INCOME TAX
1 Charge of Tax 7
2 Key Concepts 11
3 Salary 31
4 Income from Property 56
5 Income from Business 70
6 Capital Gain 113
7 Income from Other Sources 128
8 Exemptions and Tax Concessions 136
9 Losses 147
10 Deductible Allowances 157
11 Tax Credits 161
12 Common Rules 169
13 Taxation of Individual and AOP 183
14 Returns 194
15 Assessment 209
16 Appeals 230
17 Audit and Record 243
18 Definitions 249

3
CAF-02 Tax Practices

4
Tariq Hussain Tunio

INCOME TAX
“Taxes are the Price You Pay for Civilization”

5
CAF-02 Tax Practices

6
Tariq Hussain Tunio

Chapter

01

CHARGE OF
TAX
CHAPTER SYNOPSIS
TOPIC SECTION
Charge of Tax 4
Tax on Dividends 5
Tax on Profit on Debt 7B
General provisions related to
8
Final Tax
Tax Regimes

7
CAF-02 Tax Practices

1. CHARGE OF TAX §.4

Charge of Tax (Method-1►Normal Method)

1. Income tax is an annual tax

▪ imposed on every person


▪ who has taxable income
▪ for the tax year
▪ at the tax rates specified in the First Schedule to the ITO-2001. §.4(1)

2. The income tax payable by a taxpayer for a tax year is computed

▪ by applying tax rate or rates

o applicable to the taxpayer


o under ITO-2001

▪ to the taxable income

o of the taxpayer
o for the year, and

▪ from the resulting amount

o shall be subtracted
o any tax credits allowed to the taxpayer for the year. §.4(2)

Charge of Tax (Method-2►Final Tax Method)

3. Certain classes of income, including income of certain classes of persons, may be


subject to:

▪ separate taxation, or
▪ collection of tax as a final tax person. §.4(4)

4. Income subject to separate or final taxation

▪ is not included in the computation


▪ of taxable income as per normal method. §.4(5)

Deduction of Tax at Sources

5. Where income tax is to be deducted at source or collected or paid in advance


under any provision of ITO-2001, it shall be so deducted, collected or paid. §.4(6)

8
Tariq Hussain Tunio

2. TAX ON DIVIDEND §.5

1. Income tax is imposed


▪ on every person who receives

o dividend from a company or


o treated dividend

▪ at the tax rate of 15%


o specified in Division III of Part I of the First Schedule,

2. Income tax imposed on dividend is computed


▪ by applying relevant tax rate
▪ to the gross amount of dividend.

3. This section shall not apply to a dividend that is exempt from tax under this
Ordinance.

4. Tax under this section is covered under FTR.

3. TAX ON PROFIT ON DEBT §.7B

1. Tax shall be imposed, at the rate of 15% on every person, other than a company,
who receives a profit on debt from any person.

2. Tax imposed shall be computed by applying the rate of tax to the gross amount of the
profit on debt.

3. This section shall not apply to a profit on debt that –


(a) is exempt from tax under this Ordinance; or
(b) exceeds five million Rupees.

4. Tax under this section is covered under FTR.

4. GENERAL PROVISIONS RELATING TO FINAL TAX §.8

Tax on Dividend FTS as Final Tax

1. The above-said income tax imposed on dividend and royalty/FTS of non-resident


person is treated as final tax.

Features of Final Taxation

2. Following are the main features of final tax:

(I) income subject to final tax is not chargeable under any head of income in
computing taxable income.

(II) no deduction is allowed for any expenditure incurred in deriving income


subject to final tax.

(III) the amount of income is not reduced by

9
CAF-02 Tax Practices

▪ any deductible allowance; or


▪ the set off of any loss.

(IV) final tax payable by a person is not reduced by tax credits, and

(v) liability of final tax in respect of dividend or royalty/FTR is discharged to the


extent that it is deducted at source

5. TAX REGIMES NTR v. FTR METHOD §.8

10
Tariq Hussain Tunio

Chapter

02

KEY CONCEPTS
CHAPTER SYNOPSIS
TOPIC SECTION
Charge of Tax 4(1)
Person 80
When does a person become a
2(66)
taxpayer
Residential status of persons 81-84
Tax Year 74
Taxable Income 9 to 11
Tax Rates 1st Schedule
ICAP Past Papers Questions

11
CAF-02 Tax Practices

1. KEY CONCEPTS OF INCOME TAX §.4(1)

1. There are four key concepts in charge of tax on income:

▪ Person
▪ Tax Year
▪ Taxable Income
▪ Tax Rates

2. PERSON §.80

1. There are seven persons under ITO-2001:

(i) an individual
(ii) an association of persons (AOP) (both local and foreign)
(iii) a company (both local and foreign)
(iv) the Federal Government
(v) a foreign government
(vi) a political subdivision of a foreign government
(vii) a public international organization. §.80(1)

3. WHEN DOES A PERSON BECOME A TAXPAYER ? §.2(66)

1. All taxpayers are persons, but all persons are not necessarily taxpayers.

Definition of Taxpayer

2. Taxpayer
✓ means:
▪ any person

o who derives an amount


o chargeable to tax under ITO 2001, &
✓ includes
▪ any representative of a person

o who derives an amount


o chargeable to tax under ITO 2001,

▪ any person

o who is required
o to deduct/ collect income tax under ITO 2001.

▪ any person required to furnish

o a return of income or
o pay tax under the ITO-2001. §.2(66)

12
Tariq Hussain Tunio

Chart# 1► PERSON

PERSON

Individual AOP Company Fed. Govt. Foreign Govt. PSD of For. Govt PIO

a company as defined in
Firm
Co. Act, 2017

Relation between a body corporate formed


persons who by/ under a Pakistani law
have agreed to
share the profits of
a business Modaraba
carried on by all or
any of them acting
for all. a body incorporated by/
under company law of a
foreign country relating to
HUF incorporation of company.

a co-operative society, a Unit trust means any trust under


finance society or any which beneficial interests are divided
Any artificial juridical into units such that the entitlements of
person & anybody other society established the beneficiaries to income or capital
of persons formed or constituted by/ under are determined by the number of units
under a foreign law any law held.

Non-profit organization
but does not Trust means an obligation annexed to
the ownership of property and arising
include a out of the confidence reposed in and
company a trust, an entity or a body of
persons established or accepted by the owner, or declared
constituted by or under any and accepted by the owner for the
benefit of another, or of another and
law for the time being in force
the owner, and includes a unit trust.

a foreign association,
incorporated or not, declared
by the FBR by general or
special order to be a company
for the purposes of ITO 2001
Small Company means a co. registered
on or after the 01.07.2005, under the
Provincial Government Co. Act, 2017, which has paid up capital
plus undistributed reserves not
exceeding Rs. 50 m, has employees not
Local Govt in Pakistan exceeding 250 any time during the year,
has annual turnover not exceeding Rs.
250 m, and is not formed by the splitting
up or the reconstitution of company
Small Company already in existence, is not a small and
medium enterprise.

4. RESIDENTIAL STATUS OF A PERSON §.81-84

Explanation for Students


 A resident person is chargeable to income tax in Pakistan in respect of
both his Pakistan-source Income (PSI) and Foreign-source Income (FSI).
 A non-resident person is chargeable to income tax in Pakistan only in
respect of his Pakistan-source Income (PSI).

13
CAF-02 Tax Practices

Resident & Non-resident Taxpayer

1. Resident Taxpayer means

▪ a taxpayer who is a resident person. §.2(53)

2. Non-resident Taxpayer means

▪ a taxpayer who is a non-resident person. §.2(38)

Resident Person

3. A person is treated as a resident person for a tax year if the person is:
▪ a resident individual,
▪ resident company,
▪ resident association of person, or
▪ the Federal Government. §.81(1)

Non-Resident Person

4. A person is treated as non-resident person for a tax year if the person is not a
resident for that year. §.81(2)

Chart# 2► RESIDENTIAL STATUS

TAXPAYER

RESIDENT NON-RESIDENT
TAXPAYER TAXPAYER

RESIDENT NON-RESIDENT
PERSON PERSON

A Person Who is
RESIDENT RESIDENT RESIDENT FEDERAL Not a Resident
INDIVIDUAL COMPANY AOP GOVT Person

Resident Individual

5. An individual becomes resident if any of the followings two conditions is fulfilled:

(i) Presence in Pakistan of 183 Days or More

An individual is treated as resident when:

▪ he is present in Pakistan
▪ for a period or periods of 183 days or more in aggregate
▪ in a tax year. §.82(a)

14
Tariq Hussain Tunio

TABLE 1.1: COMPUTATION OF 183 DAYS


 In computing 183 days, following days are counted as full days:
- a part of a day that an individual is present in Pakistan (including day of arrival
and day of departure)
- a public holiday
- a day of leave, including sick leave
- a day that the individual’s activity in Pakistan is interrupted because of a strike,
lock-out or delay in receipt of supplies
- a holiday spent by the individual in Pakistan before, during or after any activity
in Pakistan.
 A day or part of a day where an individual is in Pakistan solely by reason of being
in transit between two different places outside Pakistan does not count as a day
present in Pakistan. (Rule 14 of ITR 2002)

ILLUSTRATION
X landed in Karachi for an hour on 20.04.20X1, while traveling via Thai Airways
from Dubai to Bangkok. In deciding his residential status in Pakistan for TY 20X1,
this day will not be counted in 183 days required to make him resident in Pakistan, as
X is in transit in Pakistan on the day.

(ii) Employee or Official of Federal or Provincial Government Posted Abroad

An individual is treated as resident individual if the individual is

▪ an employee or official of

o the Federal Government or

o a Provincial Government, and

▪ is posted abroad in a tax year. §.82(c)

Example: Pakistan’s ambassadors posted in Pakistan’s embassies aboard.

(iii) Citizen of Pakistan, not resident elsewhere

• is not present in any other country for more than 182 days during the tax
year or
• who is not a resident taxpayer of any other country. §.82(d)

Resident Company

6. A company becomes resident for a tax year if one of the following conditions is
satisfied.

(i) The company

▪ is incorporated or formed by or under

▪ any law in force in Pakistan (i.e. local or domestic company).

15
CAF-02 Tax Practices

(ii) The control and management of the affairs of the company

▪ is situated wholly in Pakistan

▪ at any time in the year.

(iii) The company is

▪ a Provincial Government or

▪ Local Government in Pakistan.

Resident AOP

7. An association of persons (AOP) becomes resident for a tax year

▪ if the control and management of the affairs of that association

▪ is situated wholly or partly in Pakistan

▪ at any time in the year.

TABLE 1.2: RESIDENTIAL STATUS OF PERSONS


Person When Resident
Individual  Presence in Pakistan of 183 days or more in aggregate in a
tax year, or
 Employee/official of Federal/Provincial Govt. posted abroad
 Citizen of Pakistan who is not present in any other country for
more than 182 days during the tax year or who is not a
resident taxpayer of any other country.
AOP  Control & Management situated wholly or partly in Pakistan at
any time during the year
Company  Company incorporated/ formed under or by Pakistani law
 Control & Management situated wholly in Pakistan at any time
in the year
 Provincial Government
 Local Government
Fed. Govt.  FG is always resident person
Foreign Govt.  Foreign government is always non-resident person
Political sub-division of a  Political sub-division of a foreign govt. is always non-resident
Foreign Govt. person
Public International  A public international organization is always non-resident
Organization

16
Tariq Hussain Tunio

5. TAX YEAR §.74

Definition of Tax year

1. Tax year means:

▪ the tax year as defined in §.74(1) ('normal tax year ') and includes

▪ a special year or a transitional year that a person is allowed by Commissioner by


order.

Normal Tax Year

2. A normal tax year is


▪ a period of twelve (12) months
▪ ending on 30th June, and
▪ is denoted by the calendar year in which the said date falls. §.74(1)

Explanation for Students


A normal tax year is denoted as follows:
(i) Find out the ending date of the normal tax year.
(ii) Find out the calendar year in which that ending date falls.
(iii) The calendar year is used to denote the normal tax year.

ILLUSTRATION
Denote the Normal Tax Year (01-07-2002 – 30.06.2003)
 Ending date of the normal tax year — 30.06.2003
 Calendar year of ending date of the NTY — 2003
 Name of the normal tax year — TAX YEAR 2003

Special Tax Year

3. Special Tax Year is:

▪ person’s income year under repealed Ordinance that is different from the
normal tax year, or

▪ the twelve months' period that is different from the normal tax year that a
person is allowed by the Commissioner by an order u/s 74(3), and

▪ denoted by the calendar year relevant to normal tax year in which the
closing date of the special tax years falls. §.74(2)

Explanation for Students


special tax year is denoted as follows:
(i) Find out the ending date of the special tax year
(ii) Find out the relevant normal tax year in which that ending date falls
(iii) Find out the calendar year in which ending date of the aforesaid normal
tax year falls
(iv) The calendar year is used to denote the special tax year

17
CAF-02 Tax Practices

ILLUSTRATION
Denote this Special Tax Year → 01/01/2004 – 31/12/2004
Ending date of the special tax year — 31.12.2004
Relevant normal tax year of ending date of STY — 01.07.2004 - 30.06.2005
Calendar year of ending date of the NTY — 2005
Name of the special tax year — TAX YEAR 2005

Transitional Tax Year

4. Transitional tax year is the period, in case of transition of tax years, that comes
between:

▪ the end of the last tax year prior to change and


▪ the date on which the changed tax year commences. §.74(9)

ILLUSTRATION
A person having from normal tax year 1.07.2003 - 30.06.2004 (TY-2004) makes
transition to special tax year 01.01.2005 — 31.12.2005 (TY-2006). Due to this
transition, a period of six months (i.e. 01.07.2004 — 31.12.2004) is left out in between
the two tax years. This period is referred to as a transitional tax year.

FBR's Power to Allow Special or Normal Tax Year

5. The FBR may by notification in official gazette permit:

▪ a class of persons, having normal tax year, to use special tax year, or
▪ a class of persons, having special tax year, to use normal tax year.

EXAMPLES:

➢ Companies Manufacturing Sugar: 10 October to 30 September


➢ Persons Exporting Rice: 1 January to 31 December

Chart# 3► TAX YEAR

Tax Year

Has FBR Allotted Any


Type of Tax Year?

YES NO

Follow the Type of


Normal Tax
Tax Year Allotted

18
Tariq Hussain Tunio

Procedure for Transitions from NTY to STY or Vice Versa

6. Following are steps through which a person may make transition from normal tax
year to special tax year and vice versa.

Who May Apply for Change of Tax Years?


▪ A person having normal tax year may apply in writing to the Commissioner to
allow him to use a twelve months’ period, other than normal tax year, as
special tax year.

▪ A person who has been allowed to use a special tax year by Commissioner
may apply in writing to the Commissioner to allow him to use normal tax year.

Order by Commissioner
▪ The Commissioner may by an order, allow the applicant to use the special tax
year or normal tax year, as the case may be.

▪ The Commissioner is required to grant the above permission only if the


person has shown a compelling need to use special or normal tax year

▪ The permission to use normal or special tax year is subject to such conditions
as the Commissioner may impose.

▪ An order by the Commissioner is to be made after providing an opportunity of


being heard to the applicant

▪ In case of rejection of application, the Commissioner is required to record the


reasons for rejection in the order.

When does the Order Take Effect


▪ The order by Commissioner permitting use of special or normal tax year shall
take effect from such date, being the first day of the special tax year or the
normal tax year, as may be specified in the order.

Remedy to Dissatisfied Person - Review by FBR


▪ A person dissatisfied with the order of Commissioner may file a review
application to the FBR,

▪ The decision by FBR on such application is final.

Withdrawal of Permission

7. The Commissioner has the authority

▪ to withdraw his permission

▪ to use special tax year or normal tax year

▪ by order in writing.

8. Before passing this order

19
CAF-02 Tax Practices

▪ the Commissioner is required

▪ to give an opportunity of being heard to the taxpayer.

9. The person dissatisfied with the order of Commissioner

▪ may file a review application to the FBR, and

▪ the decision of the FBR is final on the matter.

6. TAXABLE INCOME §.9, 10 & 11

1. Income includes:
▪ any amount chargeable to tax under ITO 2001,

▪ any amount subject to collection or deduction of tax under sections 148, 150,
152(1), 153, 154, 156, 156A, 233, 234(5), and 236Z

▪ any amount treated as income under any provisions of ITO 2001, and

▪ any loss of income. §.2(29)

Explanation for Students


Definition of income is of ‘inclusive' type (as indicated by 'includes'), which merely
makes addition in the ordinary meaning of the term. Thus, under the Income Tax
Ordinance, 2001, income comprises of all the amounts ordinarily considered as
income as well as the amounts listed in the definition itself.

Amounts Ordinarily Considered as


Income = + Inclusions of ITO-2001
Income

Heads of Income
2. For the purposes of imposition of tax and computation of total income, all
income is classified under five heads of income, namely:

(i) Salary

(ii) Income from Property

(iii) Income from Business

(iv) Capital Gains

(v) Income from Other Sources. §.11(1)

Income under a Head of Income

3. The income of a person under a head of income for a tax year is the

20
Tariq Hussain Tunio

▪ the total of the amounts derived by the person in that year that are chargeable
to tax under the head

▪ reduced by total deductions, if any, allowed to the person for the year under
that head. §.11(2)

Total Income

4. The total income of a person for a tax year is:

(i) the sum of the person’s income under each of the five heads of income for
the year.

(ii) income exempt from tax under any prevision of the ITO 2001. §.10

Taxable Income

5. The taxable income of a person for a tax year is

▪ the total income, excluding exempt income, of the person for the year

▪ reduced, but note below zero, by total of deductible allowances. §.9

7. TAX RATES 1st SCHEDULE

Tax rates are specified in the First Schedule to the Income Tax Ordinance,
Guidelines 2001. Students are not required to memorize tax rates, as they are provided in
the question paper.

1. Tax Rates for Non-salaried Individual and AOP

Sr # Taxable income Rate of tax


1. Where the taxable income does not exceed Rs. 0%
600,000
2. Where the taxable income exceeds Rs.600,000 but 7.5% of the amount exceeding Rs.600,000
does not exceed Rs.800,000
3. Where the taxable income exceeds Rs.800,000 but Rs.15,000 + 15% of the amount exceeding
does not exceed Rs.1,200,000 Rs.800,000
4. Where the taxable income exceeds Rs.1,200,000 but Rs.75,000 + 20% of the amount exceeding
does not exceed Rs.2,400,000 Rs.1,200,000
5. Where the taxable income exceeds Rs.2,400,000 but Rs.315,000 + 25% of the amount exceeding
does not exceed Rs.3,000,000 Rs.2,400,000
6. Where the taxable income exceeds Rs.3,000,000 but Rs.465,000 + 30% of the amount exceeding
does not exceed Rs.4,000,000 Rs.3,000,000
7. Where the taxable income exceeds Rs.4,000,000 Rs.765,000 + 35% of the amount exceeding
Rs.4,000,000

2. Tax Rates for Salaried Individuals

Where the income of an individual chargeable under the head “salary” exceeds
Seventy-Five percent of his taxable income, the rates of tax to be applied shall be as set out
in the following table namely:

21
CAF-02 Tax Practices

Sr # Taxable Income Rate of tax

1. Where the taxable income does not exceed Rs.600,000 0%


2. Where the taxable income exceeds Rs.600,000 but 2.5% of the amount exceeding Rs.600,000
does not exceed Rs.1200,000
3. Where the taxable income exceeds Rs.1,200,000 but Rs.15,000 + 12.5% of the amount exceeding
does not exceed Rs.2,400,000 Rs.1,200,000
4. Where the taxable income exceeds Rs.2,400,000 but Rs.165,000 + 22.5% of the amount exceeding
does not exceed Rs.3,600,000 Rs.2,400,000
5. Where the taxable income exceeds Rs.3,600,000 but Rs.435,000 + 27.5% of the amount exceeding
does not exceed Rs.6,000,000 Rs.3,600,000
6. Where the taxable income exceeds Rs.6,000,000 Rs.1095,000 + 35% of the amount exceeding
Rs.6,000,000

3. Tax Rates for Companies

▪ Tax rate for companies (except banking companies) for TY 2024 is 29%.

▪ Tax rate for banking companies for TY 2007 onwards is 39%.

▪ Tax rate for small companies is 20%.

22
Tariq Hussain Tunio

Chart# 6► MODEL FOR COMPUTATION OF TAXABLE INCOME


Name of the Taxpayer: Mr. X
Residential Status: Resident/Non-resident
Tax Year: 20x1
Computation of Taxable Income
A SALARY
Amounts Chargeable XX
Less: Nil (No Deductions are Allowed) —
Income under the Head ‘Salary’ XX
B INCOME FROM PROPERTY
Amounts Chargeable XX
Less: Deductions Allowed (XX) /--
Income under the Head ‘IFP’ XX
C INCOME FROM BUSINESS
(I) NORMAL BUSINESS
Amount Chargeable XX
Less: Deductions Allowed (XX)
Normal Business Income XX
(II) SPECULATION BUSINESS
Amount Chargeable XX
Less: Deductions Allowed (XX)
Speculation Business Income XX
Income under the Head ‘IFB’ XX
D CAPITAL GAIN
(I) CG ON DISPOSAL OF CAPITAL ASSET OTHER THAN IMMOVABLE PROPERTY
Consideration received XX
L: Cost of Capital Asset (XX)
Capital Gain XX
(II) CG ON DISPOSAL OF CAPITAL ASSET BEING IMMOVABLE PROPERTY
Consideration received XX
L: Cost of Immovable Property (XX)
Capital Gain XX
(III) CG ON DISPOSAL OF SECURITIES
Consideration received XX
L: Cost of Security (XX)
Capital Gain XX
Income under the Head Capital Gains XX
E INCOME FROM OTHER SOURCES
Amounts Chargeable XX
Less: Deductions Allowed (XX)
Income under the Head ‘IFOS’ XX
F SUM OF ALL HEADS XX
Add: Exempt Income XX
G TOTAL INCOME XX
Less: Exempt Income (XX)
NET TOTAL INCOME XX
Less: Deductible Allowances (XX)
H TAXABLE INCOME XX

23
CAF-02 Tax Practices

PAST EXAMINATION QUESTIONS


Question # 1
Spring 2010 Q.5(a)

State the provisions of the Income Tax Ordinance, 2001 regarding the residential status of
companies and association of persons. (05)

Question # 2
Autumn 2011 Q.4

a. Briefly discuss the residential status of the following persons for the tax year 2011 under
the Income Tax Ordinance, 2001.

(i) Mr. Shah has been working as an Information Analyst in the Ministry of Foreign
Affairs. On 1 November 2010, he was posted to Pakistan Embassy in Canada for
three years.

(ii) Asif Learning Center is a partnership concern, providing IT training to professionals in


Pakistan, UAE and Saudi Arabia. Up to 31 July 2010, the management and control of
its affairs was situated partly in. Pakistan. However, with effect from 1 August 2010,
the entire management and control of the affairs of the partnership was shifted to
Dubai.

(iii) Mr. Liaquat was sent to Pakistan on a special assignment by his UK based company
on 1 March 2011. He left Pakistan on 9 September 2011.

(iv) Farooq Trading LLC was incorporated as a limited liability company in UAE. The
management and control of its affairs are situated wholly in Pakistan.
(08)

b. Explain the following as specified in the Income Tax Ordinance, 2001.

(i) Industrial Undertaking (05)


(ii) Fair Market Value (03)
(iii) Apportionment of Expenditures (04)

Question # 3
Autumn 2012 Q. 3(a)

State the provisions of the Income Tax Ordinance, 2001 with regards to the residential
status of individuals and companies. (05)

Question # 4
Spring 2013 Q. 3(a)

State the provisions of the Income Tax Ordinance, 2001 for determining the residential
status of an Association of Persons. (02)

Question # 5
Spring 2013 Q. 4(a)

Inspired Pakistan Limited (IPL) wants to change its accounting year from 30 June to
31 December as the income year of its parent company in USA ends on 31
December.

24
Tariq Hussain Tunio

Required:

Advise IPL about the requirements of the Income Tax Ordinance, 2001 regarding the change
of tax year from normal to special. (03)

Question # 6
Autumn 2013 Q. 4(b)

In view of the provisions of Income Tax Ordinance, 2001 and Rules made thereunder,
determine the residential status of the following persons for the tax year 2013:

(i) Ramiz proceeded to Saudi Arabia on 24 December 2012 to assume responsibilities on


his new job. He visited Karachi from 20 June 2013 to 24 june 2013 for presenting a
paper in a seminar but due to unavoidable circumstances, the seminar was cancelled.

(ii) Khalil, an officer working at Ministry of Foreign Affairs, since last three years, was
posted to the Pakistan's mission in Geneva from 1 August 2012 to 30 June 2013.

(iii) Ali Associates is a partnership firm and provides consultancy services in Pakistan as
well as United Kingdom (UK). The management and control of its affairs is situated
partly in UK and partly in Pakistan.

(iv) Smith, a Nigerian football coach, came to Pakistan on 28 February 2013. He left the
country on 31 August 2013. (07)

Question # 7
Autumn 2013 Q.5

a. Differentiate between 'Public company’ and ‘Private company' within the meaning of
Income Tax Ordinance, 2001. (05)
b. One of your clients Inqalab Limited wants to change its accounting year.

Required:

Write a brief note to the Finance Manager of the company explaining the requirements of
Income Tax Ordinance, 2001 as regards the following:

(i) Change in tax year. (03)

Question # 8
Spring 2016 Q.3

Under the provisions of the Income Tax Ordinance, 2001 explain the following:

(a) Special tax year (03)


(b) Transitional tax year (03)
(c) Order of application of various tax credits while computing the tax liability of the
taxpayer (03)
(d) General provisions/rules which may apply to income subject to final tax. (06)

25
CAF-02 Tax Practices

Question # 9
Autumn 2017 Q. 4(a)

Under the provisions of the Income Tax Ordinance, 2001, state the situations where
expenditure is required to be apportioned for the purpose of claiming a deduction. (03)

Question # 10
Autumn 2017 Q. 2(a)

Under the provisions of Income Tax Ordinance, 2001 and rules made thereunder:

(a) Discuss the residential status for tax year 2017 in each of the following situations:

(i) On 21 September 2016 Asif proceeded to Dubai to join his hew job. Due to certain
professional issues with his employer in Dubai, he resigned on 1 May 2017 and came
back to Pakistan. On 16 May 2017 he got a new job in Pakistan which he continued till
30 June 2017. (02)

(ii) Sami Associates is an association of persons and provides accounting services in Dubai.
On 2 January 2017, the entire management and control of its affairs was shifted from
Karachi to Dubai. (02)

Question # 11
Spring 2018 Q. 3(c)

On 1 December 20X7 Bruce Lee was appointed by a Chinese company as a Technical


Director for Pakistan. He has provided you the following details

Arrival in Pakistan 15 December 20X7


Joined office in Pakistan 20 December 20X7
Visit to Dubai on an official trip 21-30 March 20X8
Visit to South Korea for vacations 12-21 April 20X8
Visit to northern areas of Pakistan for personal nip 4-9 lune 20X8

In view of the provisions of the Income tax Ordinance, 2001 and related Rules
thereunder, comment on the residential status of Bruce Lee for the tax year 20X8.
(03)

Question # 12
Autumn 2018 Q. 2

Kaleem Limited (KL) is a listed company and its accounting year ends on 30 June. KL
is now considering to change its accounting year from 30 june to 30 September.

Under the provisions of the Income Tax Ordinance, 2001:

(a) briefly describe normal, special and transitional tax year. (06)
(b) state the requirements regarding change in tax year from normal to
special.(02)
(c) state the tax year corresponding to the income year ended 30 September 20X8
and the due date for filing the return of income. (02)

26
Tariq Hussain Tunio

Question # 13
Autumn 2018 Q. 3(a)

Hirani & Company (HC), a resident AOP, is engaged in the manufacturing of various
consumer products and is assessed under normal tax regime. During the year ended 30
June 20X8, HC’s sales was Rs. 14,000,000. It includes sales tax of Rs. 1,000,000 and
excise duty of Rs. 500,000. The taxable income for the year is Rs. 1,170,000.

Compute HCs tax liability for tax year 20X8, under the provisions of the Income Tax
Ordinance, 2001. (03)

Question # 14
Spring 2019 Q. 2

(i) Haider, a filer, was carrying on business as a cloth trader. On 28 October 20X7 there
was a fire in his shop and the entire stock of clothes costing Rs. 1,550,000 was
destroyed. The insurance company refused to pay the claim. Consequently, Haider
ceased his business on 31 January 20X8.

After cessation of business, Haider filed an appeal against the insurance company and
was able to recover Rs. 1,300,000 as full and final settlement from the insurance
company in tax year 20X9.

Required:

Under the Income Tax Ordinance, 2001:

a. state the requirements that Haider should comply with, on cessation of his
business on 31 January 20X8. (03)
b. briefly discuss the treatment of the recovered amount in the tax year 20X9.(02)

(ii) Mohsin has been working at the head office of Lewis Consulting, Inc. (LC1) situated in
New York, USA On 1 January 20X8, LCl had established its branch office in Pakistan
and had sent Mohsin for two years as Country Manager for looking after the Pakistan
operations.

During the tax year 20X9, apart from salary income, Mohsin earned/received the
following amounts:

• On 15 December 20X8, he conducted a seminar in USA for a fee of USD 18,000. On


his request, the event manager transferred the amount (net of tax) directly to his
personal bank account in Islamabad on 10 January 20X9.

• On 31 May 20X9, he earned income from his business established in USA and
brought 40% of the income to Pakistan.

Required:

Under the Income Tax Ordinance, 2001:


(i) state the residential status of Mohsin for the tax year 20X8. (01)
(ii) discuss the taxability of his foreign.source incomes for the tax year 20X9. (04)

27
CAF-02 Tax Practices

Question # 15
Autumn 2019 Q. 3(a)

Respond to the following independent scenarios, under the provisions of the Income
Tax Ordinance, 2001:

Jean Francois, a French designer, often visits to Pakistan for promotion of his
products. During his last visit he stayed in Pakistan from 10 July 20X8 to 25 February
20X9. Determine the residential status of Jean Francois for tax year 20X9, assuming
that the Commissioner has granted him permission to use calendar year as special
tax year. (02)

Question # 16
Spring 2021Q. 1(b)

Dr. Jamal is planning to establish a hospital as a non-profit organization.

Required:
Discuss the conditions that should be complied with by Dr. Jamal, under the Income
Tax Ordinance, 2001. (03)

Question# 17
Spring 2021 Q.1(b)

(a) What do you understand by the term ‘Turnover’ as provided in section 113 of
the Income Tax Ordinance, 2001? List the persons who are required to pay
minimum tax on the basis of turnover. (08)

(b) Gillani and Company (GC), a sole proprietor, is dealing in various consumer
products in Pakistan. During the year ended 30 June 20X2, GC’s taxable
income for the year was Rs. 1.6 million.

Required:

Under the provisions of the Income Tax Ordinance, 2001 compute the amount of net
income tax payable by GC and amount of income tax to be carried forward; if any, for
the tax year 20X2, in each of the following situations:

(i) GC’s sales were Rs. 120,500,000 inclusive of sales tax.


(ii) GC’s sales were Rs. 110,000,000 inclusive of sales tax. (05)

Question #18
Autumn 2021Q. 3(a)

(a) State the provisions of the Income Tax Ordinance, 2001 relating to each of the
following;-

I.Change of tax year from special to normal (02)

II.Change in the method of accounting for income chargeable to tax under the
head 'income from business’ (03)

Question #19
Spring 2022 Q. 2(a)

28
Tariq Hussain Tunio

Under the provisions of the Income Tax Ordinance, 2001 discuss the tax
implication/treatment in each of the following independent matters:

(a) Payment of any sum by a private company to its shareholder by way of a


loan.(03)
(b) Profit on debt received by a non-resident person on a security issued by a
resident person. (03)

Question # 20
Spring 2022 Q. 4(c)

Following information pertains to three unlisted companies:


Company Paid up Total Annual Shareholders
capital Reserves Turnover
A Limited 30 80 150 60% shares are held by a foreign
company
B Limited 80 (35) 220 40% shares are held by the
Provincial and Federal
governments
C Limited 40 5 500 100% shares are held by a local
group

Required:

Under the provisions of the Income Tax Ordinance, 2001 briefly discuss whether
each of the above companies can be classified as small, public or private. Also state
the additional information, if any, which may be required for determining the
classification of these companies. (07)

29
CAF-02 Tax Practices

30
Tariq Hussain Tunio

Chapter

03

SALARY
CHAPTER SYNOPSIS
TOPIC SECTION
2(20),2(21),2(22),
Introduction
12 & 69
What is Salary 12
Taxation of Core amounts 12
Taxation of Allowances 12
Taxation of Perquisites 13
Taxation of Re-imbursement of
12
Expenses
Taxation of Special Amounts 12

Taxation of Retirement Benefits 2nd & 6th Schedule

ICAP Past Papers Questions

31
CAF-02 Tax Practices

1. INTRODUCTION

Definitions

1. EMPLOYEE §.2(20)

Employee means

▪ any individual

▪ engaged in employment

2. EMPLOYER §.2(21)

Employer means

▪ any person

▪ who engages and remunerates an employee.

3. EMPLOYMENT §.2(22)

Employment includes

▪ a directorship or any other office involved in the management of a company;

▪ a position entitling the holder to a fixed or ascertainable remuneration; or

▪ the holding or acting in any public office.

Amounts Chargeable under 'Salary'

1. The amount chargeable to tax under this head is 'salary' i.e. employment income of
employees.

2. Hence, any amount, benefit or perquisite received by an employee from an


employer on account of employment is chargeable to tax under 'Salary'.

No Deductions Allowed

3. In computing income under the head 'Salary', no deductions are allowed for
any expenditure incurred by an employee in deriving his salary income.

Basis of Charge

4. Salary is charged to tax on ‘received basis’ (i.e. cash basis).

5. Any salary, except exempt, received in a tax year by an employee is


chargeable to tax in that year. Hence, 'advance salary' and 'salary-in-arrears' are also
chargeable to tax in the year of receipt.

When is Salary treated as Received?

6. An amount, benefit or perquisite is also treated as received if it is:

32
Tariq Hussain Tunio

(i) actually received by the person,


(ii) applied on behalf of the employee, at the instruction of employee, or under any
law, or
(iii) it is made available to the employee.

7. An amount or perquisite is also treated as received by an employee from any


employment if it is paid or provided:

▪ by the employee’s employer, an associate of the employer, or by a third


party under an arrangement with the employer or associate of the employer.

▪ to the employee, or to an associate of employee or to a third party under an


agreement with the employee or an associate of the employee.

▪ An amount, benefit or perquisite is also treated as received by an employee


when it is paid or provided by a ‘past employer’ or a ‘prospective employer.

Chart# 1► RECIEPT OF SALARY

Associate of Employer 3rd Party


(including past &
Employer Prospective)
(Under Arrangement)

Pay to

Associate of Employee 3rd Party


Employee (Under Agreement)

2. WHAT IS SALARY?

Definition of Salary

1. Salary means any amount received by an employee from any employment, whether
of a revenue or capital nature,

2. including

(a) any pay, wages or other remuneration provided to an employee, including

▪ leave pay,
▪ payment in lieu of leave,
▪ overtime payment,
▪ bonus,
▪ commission,
▪ fees,

33
CAF-02 Tax Practices

▪ gratuity or
▪ work condition supplements (such as for unpleasant or dangerous
working conditions):

(b) any perquisite, whether convertible to money or not;

(c) the amount of any allowance provided by an employer to an employee


including
▪ a cost of living allowance,
▪ subsistence allowance,
▪ rent allowance,
▪ utilities allowance,
▪ education allowance,
▪ entertainment allowance, or
▪ travel allowance,

Any allowance that is solely expended in the performance of the


employee‘s duties of employment shall not be included in salary.

Explanation – for removal of doubt, it is clarified that the allowance


allowance solely expended in the performance of employee’s duty does not
include –

(i) paid in monthly salary on fixed basis or percentage of salary; or


(ii) is not wholly, exclusively, necessarily or actually spent on behalf of
the employer

(d) the amount of any expenditure

▪ incurred by an employee that is paid or reimbursed by the employer,

▪ other than expenditure incurred on behalf of the employer in the


performance of the employee‘s duties of employment;

(e) the amount of any profits in lieu of, or in addition to, salary or wages, including
any amount received:

(i) as consideration for a person's agreement to enter into an


employment relationship;

(ii) as consideration for an employee‘s agreement to

▪ any conditions of employment or


▪ any changes to the employee‘s conditions of
employment;

(iii) on termination of employment, whether paid voluntarily or under


an agreement, including:

▪ any compensation for redundancy or loss of employment


and
▪ golden handshake payments;

(iv) from a provident or other fund

34
Tariq Hussain Tunio

▪ to the extent to which


▪ the amount is not a repayment of contributions
▪ made by the employee to the fund
o in respect of which the employee was not
o entitled to a deduction; and

(v) as consideration for an employee‘s agreement to a restrictive covenant


in respect of any past, present or prospective employment;

(f) any pension or annuity, or any supplement to a pension or annuity; and

(g) any amount chargeable to tax as Salary under employee share scheme.

35
CAF-02 Tax Practices

Chart# 2► DEFINITION OF SALARY

SALARY

Any amount received by an employee from any employment, whether of a revenue or capital
nature

& includes

Re-imburse-
Core ment of
Special Retirement
Allowance Perquisites
amounts Expenditure Amounts Benefits

Basic Salary Dearness Alwc Conveyance Payment or Re- Employee share Provident Fund
Wages Cost of Living Accommodation imbursement of Scheme (ESS) Gratuity
Overtimes Alwc Utilities expenditure incurred Tax on Tax Commutation of
Bonus House Rent Alwc Domestic Assistant by employee Leave Pension
Commission Conveyance Interest-free/ (except the Encashment Golden Hand-
Leave Pay Alwc Concessional loan expenditure that is Profits in lieu shake
Payment in Leave Fare Waiver/ Payment incurred by the of/in addition to
Pension
lieu of leave Assistance Alwc of Obligation employee in salary
Fees Entertainment Consideration Annuity
Transfer of performance of his for
Work conditions Alwc Property duties of person’s agreement Superannuation
supplement Subsistence Alwc Provision of employment) to enter into Fund
Arrear Salary Utilities Alwc services employment Benevolent Fund
Any other Education Alwc Any other Consideration for
Remuneration Travel Alwc perquisite employee’s
Medical Alwc agreement to any
Any other conditions of
allowance (except employment or
an allowance that changes therein.
is solely As consideration for
expended in the employee’s
performance of agreement to
employee’s duties restrictive covenant.
of employment).

Explanation for Students


The definition of ‘salary’ is very wide in its scope. All amounts, benefits, or
perquisites received by an employee in a tax year from his employer on account of
his employment are salary of the employee. However, for an amount, benefit or
perquisite to be considered salary, the pre-requisite is the existence of ‘employer-
employee relationship’ between the two.

3. TAXATION OF CORE AMOUNTS

1. Remunerations paid to an employee by employer for employment services are


ordinarily referred to as ‘Pay’. Pay may be given to an employee under different
names, such as these.

▪ Pay
▪ Wages
▪ Overtime payment

36
Tariq Hussain Tunio

▪ Bonus
▪ Commission paid by employer
▪ Fees
▪ Leave pay (i.e. remuneration paid while an employee is on leave)
▪ Payment in lieu of leave
▪ Work condition supplements (such as for unpleasant or dangerous working
conditions)
▪ Any other remunerations provided to an employee

2. All of these amounts are fully taxable under the head Salary.

4. TAXATION OF ALLOWANCES

1. Allowance is an amount given to an employee by employer for a particular purpose,


such as house rent allowance for meeting expenses of rent for accommodation.

2. All allowances are chargeable to tax, except when

▪ the allowance is exempt from tax.


▪ the allowance provided to an employee is solely expended in the performance
of the employee’s duties of employment.

ILLUSTRATION
X, working at ABC Courier Ltd, received a monthly fuel allowance of Rs. 3,000/- for
Tax Year 20X1, which was expended on fuel consumed in motorcycle listed in
delivery of consignments. Annual amount of fuel allowance received by X amounting
to Rs.36,000/-, which is not to be added to his salary for TY 20X1, as the amount is
solely expended in the performance of his duties of employment.

Cost of Living Allowance (COLA)

3. Cost of living may increase due to inflation and other factors. The allowance given to
an employee to keep up with changes in the cost of living is called Cost of Living
Allowance (COLA).

4. Cost of living allowance may also be given to an employee who is posted from low-cost
area to high-area where cost of living is higher than the area where the employee has been
previously working. Cost of Living Allowance is fully taxable.

Subsistence allowance

5. Subsistence allowance is the amount given by an employer to a new employee to


buy food, clothing, and pay for other necessities of life while awaiting the first pay.
Subsistence allowance is fully taxable.

House Rent Allowance

6. House Rent Allowance (HRA) is given by an employer to an employee to meet his


expenses in connection with rent of accommodation. HRA is fully taxable.

37
CAF-02 Tax Practices

Conveyance Allowance

7. Conveyance allowance refers to an amount given by employer to an employee to


meet the expenses of transportation from place of residence to place of duty. Since
such transportation does not constitute performance of duties of employment, hence,
it is fully taxable.

8. Conveyance allowance is not chargeable to tax if it is given to an employee

▪ to meet the expenses incurred on conveyance


▪ during performance of official duties

ILLUSTRATION
An employee based in Karachi, while visiting factory located in Lahore, is given
conveyance allowance for traveling from hotel (place of residence) to factory (place
of duty). The conveyance allowance in this case is not chargeable to tax, for being
expended in performance of duties of employment.

Dearness Allowance

9. An allowance given to mitigate the impact of inflation upon low income earning
employee on account of inflation is referred to as dearness allowance. Dear allowance
is fully taxable.

Utilities Allowance

10. Utilities Allowance is given by an employer to an employee to meet the expenses in


connection with utilities i.e. electricity, water, gas and telephone. Utilities allowance is
fully taxable.

Education Allowance

11. Education Allowance is given by an employer to an employee to meet the expenses of


education of his children. This allowance is fully taxable.

Entertainment Allowance

12. Entertainment allowance, given to an employee for his own or his family’s
entertainment, is fully taxable.

13. Whereas, entertainment allowance provided in connection with entertainment


of employer’s clients/customers is not chargeable to tax, for being solely expended in
the performance of duties of employment.

Travel Allowance

14. Travel Allowance given to an employee to meet expenses of his personal


travel is fully chargeable to tax, whereas, allowance given to an employee for official
travel is not chargeable, being solely expended on employee’s duties of employment.

Leave Fare Assistance Allowance

15. Leave fare assistance (LFA) allowance is given to an employee as a vacation


bonus. It is provided to employees to travel away from their place of work for

38
Tariq Hussain Tunio

recreation and annual leave, with a view to encourage high stress employees to
rejuvenate their mind and body. LFA is fully taxable.

Medical Allowance

16. Any medical allowance received by an employee is exempt upto ten per cent of his
basic salary if free medical treatment/hospitalization or both, or reimbursement of
medical charges/hospital charges or both are not provided in the terms of
employment.

Chart# 3► MEDICAL ALLOWANCE

Free medical treatment/hospitalization or both, or


reimbursement of medical charges/hospital charges
or both are provided in the Terms of Employment

YES NO

Medical Allowance
Medical Allowance
Exempt upto 10%
Fully Taxable
of Basic Salary

5. TAXATION OF PERQUISITES

Explanation for Students


Perquisites are benefits or facilities provided to an employee by an employer.
Perquisites may be convertible into money (e.g. bonds or shares) and not
convertible into money (e.g. accommodation or car). All perquisites provided
by an employer to employee are chargeable to tax, except when the
perquisite is exempt from tax the perquisite is specially granted to meet the
expenses wholly and necessarily incurred in the performance of the duties of
employment.

Accommodation/ Housing

1. If an employer provides accommodation or housing to an employee in a tax year, the


employee’s salary shall include an amount computed as follows:

Valuation equal to Alternative Amount

2. The value of accommodation provided by employer to employee is taken

▪ equal to the amount that would have been paid


▪ in case such accommodation was not provided.

Minimum Value Not to be Less than 45% of MTS/Basic Salary

3. The value of accommodation shall

39
CAF-02 Tax Practices

▪ in no case
▪ be less than 45% of;

o the Minimum of the Time Scale (MTS) of the basic salary or


o the basic salary where there is no time scale.

Explanation for Students


‘Time Scale’ of basic salary is a unit of measurement of basic salary over the
years and is fixed by employer in the following manner.
Rs. 25,000 – Rs. 5000 – Rs. 40,000
The first amount is the MTS of basic salary. It is the amount of basic salary in
the beginning of an employee’s a grade/scale. The second amount represents
annual increment that is added to employee’s basic salary every year. The
last figure is called the maximum of time scale of basic salary, and is the
amount of basic salary required to be achieved before promotion to the next
grade/scale.

Valuation at 30% for Certain Persons

4. Where House Rent Allowance is admissible @ thirty per cent,

▪ the value taken for accommodation/housing


▪ shall be an amount not less than30% of

o minimum of the time scale of basic salary or


o the basic salary where there is no time scale.
Explanation for Students
The Government employees serving in mufasil areas, i.e. field offices located
at distance, receive house rent allowance at 30% of basic salary, as against
45% of basic salary received by other employees. Such employees are given
a concession by valuing their facility of housing/accommodation at 30% of
MTS/BS, rather than 45% of MTS/BS.

Motor Vehicle

5. If motor vehicle is provided to an employee wholly or partly for his private use,
the employee’s salary shall include an amount computed as follows:

TABLE 2.1: VALUE OF MOTOR VEHICLE

The value of conveyance provided by the employer to the employee shall be taken equal to an
amount as below:
1 Partly for personal and 5% of:
partly for official use (a) the cost to the employer for acquiring the motor vehicle, or,
(b) the fair market value of the motor vehicle at the
commencement of the lease, if the motor vehicle is taken on
lease by the employer.
2 For personal use only 10% of:
(a) the cost to the employer for acquiring the motor vehicle; or,
(b) the fair market value of the motor vehicle at the
commencement of the lease, if the motor vehicle is taken on
lease by the employer. (Rule 5 of the ITR-2002)

40
Tariq Hussain Tunio

Services of Servants & Domestic Assistants

6. If services of a housekeeper, driver, gardener or other domestic assistant are


provided to an employee, the employee’s salary shall include:

▪ an amount equal to the total salary paid to such servant or assistant in that
year for services rendered to the employee,
▪ as reduced by any payment paid by employee to the employer for such
services.

Provision of Utilities

7. If utilities are provided to an employee, the employee’s salary shall include

▪ an amount equal to the fair market value of the utilities provided,


▪ as reduced by any payment made by the employee to the employer for the
utilities.

Waiver or Payment of Obligations by Employer

8. If an obligation of an employee

▪ to pay or repay an amount


▪ owing by the employee to the employer
▪ is waived in a tax year by the employer,
▪ the employee’s salary shall include the amount so waived.

ILLUSTRATION
X borrowed Rs. 500,000/- from his employer ABC Ltd, and repaid Rs. 400,000/-. The
balance amount of Rs. 100,000/- was waived by his employer in TY 200X. Amount of
Rs. 100,000/- waived by employer is chargeable to tax under Salary.

9. If an obligation of an employee to pay or repay an amount

▪ owing by the employee to another person


▪ is paid by the employer in a tax year,
▪ the employee’s salary shall include the amount so paid.

ILLUSTRATION
X, an employee at ABC Ltd, borrowed Rs. 800,000/- from a bank, which was paid by
his employer. The amount of Rs. 800,000/- is chargeable under the head Salary.

Transfer of Property

10. If property is transferred by employer to employee, the employee’s salary shall


include:

▪ an amount equal to the fair market value of the property

▪ determined at the time the property is transferred,

▪ as reduced by any payment made by the employee for the property.

41
CAF-02 Tax Practices

ILLUSTRATION
X is CEO of ABC Ltd. During Tax Year 200X, the company transferred a car, a sedan
worth Rs. 2.4 (m), to him. He had to pay only Rs. 400,000/- for the car. In this
example, Rs. 2 (m) is chargeable to tax as salary of X in Tax Year 200X.

Provision of Services

11. If services are provided by an employer to an employee, the employee’s salary shall
include:

▪ an amount equal to the fair market value of the services

▪ determined at the time the services are provided,

▪ as reduced by any payment made by the employee for the services.

Interest Free or Concessional Loan

12. If an employer makes a concessional or interest-free loan to his employee, the


benefit represented by the unpaid profit on debt is charged to tax to the employee if
the following conditions are satisfied:

▪ Loan is made on or after 1st July 2002


▪ Loan amount is not less than Rs. 1,000,000
▪ Either no profit on loan (i.e. interest) is payable by the employee or the rate of
profit on loan is less than the benchmark rate.
▪ The employee receiving loan should not have waived interest on his account
maintained with the employer.

Computation of Benefit Chargeable to Tax

13. The amount chargeable to tax to the employee under the head “Salary” for a tax year
shall include an amount as shown in the following table.

TABLE2.2 : VALUE OF INTEREST-FREE/CONCESSIONAL LOAN


Sr. Situation Amount to be Charged
1 No profit on loan is payable byAn amount equal to the profit on loan
the employee computed at the benchmark rate.
2 Profit on loan is payable by An amount equal to the difference
the employee at a rate less between the amount of profit on loan
than the bench mark rate paid by the employee in that tax year and
the amount of profit on loan computed at
the benchmark rate
3 Profit on debt is payable by Nil
employer at a rate equal to or
more than bench mark rate

Definition of ‘Benchmark Rate’


14. Benchmark rate means:
▪ a rate of 5% per annum for tax year commencing on 1 st July 2002, and

42
Tariq Hussain Tunio

▪ for subsequent tax years, the rate is to be taken

- at 1% above the rate applicable for immediately preceding tax year,


- but not exceeding 10% per annum in respect of any tax year. §.13(14)(a)

Concessional Loan When Used in Acquisition of Property

15. When concessional loan granted by the employer


▪ is wholly or partly used by employee

▪ for acquisition of any asset or property which produces income chargeable to


tax under a head of income,

▪ then the employee is treated as having paid an amount as profit on debt

▪ equal to the benchmark rate on the loan or that part of the loan used to
acquire the asset or property.

Explanation for Students


The deemed payment of profit on debt by employee does not affect taxation of
benefit to employee under the head Salary on account of free/concessional loan.
Regardless of the use of the free/concessional loan, the aforesaid benefit is always
added to employee’s salary, unless otherwise provided in law.

Effect of Waiver of Interest by Employee

16. The benefit represented by concessional or interest-free loan


▪ is not chargeable to tax in case of an employee

▪ who has waived interest on his account with the employer.

43
CAF-02 Tax Practices

Chart# 4► CONCESSIONAL OR INTEREST-FREE LOAN

Concessional/
Interest-free Loan

Loan Exceeding
Rs. 1,000,000

YES NO

Whether employee
No Amount is
has waived interest
chargeable to tax
on his account

YES NO

Amount equal to
No Amount is
BMR or Difference
chargeable to tax
is charged to tax

ILLUSTRATION
Being a religious person, X waived interest on his account maintained by his
employer ABC Ltd, due to which, he will not receive any interest from his employer
on any account maintained by the employer such as on Provident Fund. In Tax Year
20X1, he borrowed Rs.1,800,000/- interest free from his employer. Therefore, an
amount equal to benchmark rate will not be added to his salary in 20X1, as he has
waived interest on his account with the employer.

Valuation of Residual Perquisites §.13(13)

17. In case an employer provide a perquisite whose valuation is not specifically provide
in the ITO 2001, or the rules,

▪ the value of the perquisite is to be taken at an amount


▪ equal to the fair market value of the perquisite determined at the time it
is provided,
▪ as reduced by any payment made by the employee for the perquisite.

ILLUSTRATION
X, employed at ABC Ltd, received an air conditioner costing Rs. 38,000/-, from his
employer in Tax Year 200X for his personal use. The air conditioner is depreciated at
15% by the employer. In this illustration, an amount of Rs.5,700/- (38,000 x 15%)
will be added to employee’s salary.

Free Medical Treatment/Hospitalization or Reimbursement thereof

18. If free medical treatment, free hospitalization or both, or reimbursement of


medical charges/hospital charges or both are paid by employer to employee, the
amount paid by the employer on account of this benefit is fully exempt, subject to
following conditions:

44
Tariq Hussain Tunio

▪ Free medical treatment/hospitalization or both, or reimbursement of medical


charges/hospital charges or both are provided in accordance with the terms of
employment

▪ National Tax Number (NTN) of the hospital or clinic is given and

▪ The employer certifies and attests the medical/ hospital bills

Exemption for Food, Education, Medical Treatments, etc.

19. The following perquisites received by an employee by virtue of his employment are
exempt:

(i) free or subsidized food provided by hotels and restaurants to its employees
during duty hours;

(ii) free or subsidized education provided by an educational institution to the


children of its employees;

(iii) free or subsidized medical treatment provided by a hospital or a clinic to its


employees; and

(iv) any other perquisite or benefit for which the employer does not have to bear
any marginal cost, as notified by the Board.

6. TAXATION OF RE-IMBURSEMENT OF EXPENDITURE

1. In case an employee incurs expenditure (e.g. purchase of laptop or mobile phone),


and the amount of expenditure is either paid or reimbursed by the employer, the
amount paid or reimbursed is charged to tax to the employee under the head Salary.

2. However, any amount of expenditure paid or reimbursed by employer that is incurred


by an employee in the performance of his duties of employment is not included in
employee’s salary.

7. TAXATION OF SPECIAL AMOUNTS

Employee Share Scheme (ESS)

1. Employee Share Scheme means any agreement or arrangement under which a


company may issue shares in the company to:

▪ an employee of the company or


▪ an employee of an associated company; or
▪ the trustee of a trust and under the trust deed

- the trustee may transfer the shares


- to an employee of the company or to an employee of an associated
company

45
CAF-02 Tax Practices

Chart# 5► EMPLOYEE SHARE SCHEME

Company B
Company A (Associate of Co. A)

Mr. X Mr. Y
(Employee of Co. A) (Employee of Co. B)

Trustee
of a Trust

Receipt of Mere Right or Option is Not Chargeable

2. The value of a ‘right or option to acquire shares’ under an employee share scheme
granted to an employee is not chargeable to tax.

Gain on Disposal of Right or Option is Chargeable under Salary

3. If an employee, who has received the right or option to acquire shares under ESS,
disposes of the said right or option in a tax year, the amount chargeable to tax to the
employee is the amount of any gain computed as follows.

Consideration Received on Disposal of Right/Option


Less: Employee’s cost in respect of the right or option
Amount of Gain Chargeable Under the Head Salary

Issuance of Shares without Restriction

4. Where an employee is issued shares under ESS:

▪ the amount chargeable to tax to the employee is equal to


▪ the fair market value of the shares
▪ determined at the date of issue,
▪ as reduced by any consideration given by the employee for the shares
and for grant of a right or option to acquire the shares.

Issuance of Shares with Restriction

5. If shares are issued to an employee with restriction on their transfer the


amount chargeable to tax to the employee is

▪ the fair market value of the shares at the earlier of


- the date the employee has a free right to transfer the shares or
- the date employee disposes of the shares,

46
Tariq Hussain Tunio

▪ as reduced by any consideration given by the employee for the shares and for
the grant of a right or option to acquire the shares.

TABLE 2.3: EMPLOYEE SHAHRE SCHEME


Sr. Situation Amount to be Charged
1 Receipt of mere right or option Nil
to receive shares
2 Disposal the right or option to Consideration received on disposal the right or
receive shares option reduced by employees cost in respect of
right or option
3 Receipt of shares (without any FMV of the shares on the date of issue
restriction) reduced by any consideration paid by the
employee for shares, including any amount
paid for right or option to acquire shares
4 Receipt of shares with Nil
restriction
5 Date of free right or disposal of FMV of the shares on the earlier of date of free
shares received with restriction. right or disposal of shares reduced by any
consideration paid by the employee for shares,
including any amount paid for right or option to
acquire shares

Cost of Shares Acquired under ESS

6. The cost of the shares acquired by an employee under ESS is sum of:

▪ the consideration given by the employee for the shares,


▪ the consideration given by the employee for the grant of any right or option to
acquire the shares, and
▪ the amount chargeable to tax under the head “Salary” for receipt of shares
under ESS.

Taxation of Arrear Salary

7. In case an employee

▪ receives salary-in-arrears in a tax year and


▪ as a results he is chargeable to tax at higher tax rate
▪ than would have been applicable if the amount had been paid in the tax year
in which services were rendered,
▪ the employee may elect for the amount of salary-in-arrear to be taxed at the
tax rate of the tax year in which services were rendered.

8. Employee exercising the above option is required to follow the following procedure.

▪ Serve a notice in writing to the Commissioner through which application of tax


rates of the relevant tax year is made.

▪ This election must be made by:


- the due date for furnishing the return of income for the tax year in which
salary-in-arrears is received, or

- by such date as the Commissioner may allow.

47
CAF-02 Tax Practices

Tax-Free Salary (Tax on Tax)

9. If an employer agrees to pay tax due from an employee in a tax year, the employee
has to include the amount of tax payable by the employer in his salary.

10. The sum paid by employer is deemed as perquisite as it represents the employee’s
obligation towards the Government, being paid by employer.

11. The amount of the employee’s income chargeable under the head “Salary” is
grossed up by the amount of tax payable by the employer on behalf of the employee.
Since, the method of grossing up involves tax-on-tax, hence, this topic is popularly
called tax on tax. §12(3)

Employer Agrees to Pay a Fixed Amount of Tax

12. In case employer has only agreed to pay a fixed amount of tax, the said amount is
added to employee’s salary and tax worked out thereon. The extra amount of tax,
being the difference between the tax paid by employer and the tax calculated on
gross salary, is paid by the employee himself.

Employer Agrees to Pay All the Amount of Tax

13. In case the employer has agreed to pay all the tax payable by employee on his
salary, then the employee’s salary is to be grossed up by the amount of tax payable
by the employer.

8. TAXATION OF RETIREMENT BENEFITS

Golden Handshake Amount

1. An employee
▪ who has received an amount on termination of employment
▪ whether paid voluntarily or under an agreement,
▪ including any compensation for redundancy or loss of employment and
▪ golden handshake payments in a tax year
▪ may elect by notice in writing to the Commissioner for the amount to be
taxed at the rate computed in accordance with the following formula, namely:

Total Tax Paid/Payable For 3 Preceding Years


X GHS Amount
Total Taxable Income For 3 Preceding Years

Procedure for Application of Average Rate

2. For the application of average tax rate of three preceding tax years on golden
handshake amount, the taxpayer is required to elect it by a notice in writing to the
Commissioner.

3. This election must be made:


- by the due date for furnishing the employee’s return of income for the
tax year in which the amount was received; or
- by such later date as the Commissioner may allow.

48
Tariq Hussain Tunio

Explanation for Students


The taxpayer is given the option of getting the GHS receipts taxed at the
average tax rate of three preceding years because the average tax rate is
normally lower than the current tax rate. In this way, a relief is provided to the
taxpayers in the form of reduction in the tax rate. In case the recipient has a
past service of less than three years, the average rate of the actual years
during which the recipient was in employment will be adopted.

Provident Fund

 Statutory Provident Fund (SPF)

4. A provident fund to which the Provident Funds Act, 1925, applies is referred to as
Statutory Provident Fund. It is established for employees of the Federal Government,
local government, public sector organization and local authority.

5. Any payment received from statutory provident fund is fully exempt from tax –Clause
(22) of Part I of 2nd Schedule.

 Recognized Provident Fund (RPF)

6. Recognized provident fund is a fund which is recognized by the Commissioner


(under Part I of the Sixth Schedule to the Income Tax Ordinance, 2001). –§2(48)

7. Employees own contribution received from RPF is not part of his income, hence, not
charged to tax. –§12(2)(e)(iv)

8. Employer’s contribution to employee’s provident fund account is taxable annually,


and an amount exceeding lower of one-tenth of salary or Rs. 150,000 is charged to
tax. –Rule 3(a) Part I of 6th Schedule

9. Interest credited to employee’s recognized provident fund is taxable annually under


the head Salary, and an amount exceeding higher of one-third of Salary or interest amount
calculated at rate notified by the Federal Government i.e. 16% is charged to tax. –Rule 3(b),
Part I of 6th Schedule

10. Definitions – Rule.14, Part I of 6th Schedule

Salary for the purposes of recognized provident fund includes dearness allowance, if
the terms of employment so provide, but excludes all other allowances and
perquisites.

Balance to the credit of an employee means the total amount to the credit of his
individual account in a provident fund at any time.

11. Exemption

The accumulated balance (which comprises of contributions by employer and


employee and interest) due and becoming payable to an employee at the time of his
retirement is exempt from tax. –Clause (23), Part I of 2nd Schedule.

 Un-recognized Provident Fund (URPF)

49
CAF-02 Tax Practices

12. A provident fund that is not recognized by the Commissioner (under Part I of the
Sixth Schedule to the Income Tax Ordinance, 2001) is referred to as un-recognized
provident fund.

13. Employees own contribution received from RPF is not part of his income, hence, not
charged to tax.

14. Neither employer’s contribution nor interest credited to employee’s recognized


provident fund is taxable on annual basis.

15. Any amount received from an un-recognized provident fund except repayment of
employee’s own contribution is chargeable to tax only when it is received by the
employee from the provident fund.

TABLE 2.4: PROVIDENT FUND

Types of Employee’s Employer’s Interest Payment


Sr
Provident Annual Annual Annual of Gross
No
Funds Contribution Contribution Credited Amo u n t
1 Statutory
Not Taxable Exempt Exempt Exempt
Provident
Fund
2 Recognized Not Taxable Amount exceeding Amount exceeding Exempt, as already
Provident lower of 10% of higher of 1/3 of Salary taxed on annual
Fund salary or Rs. or interest amount basis.
150,000 is taxed on calculated*- @ 16% is [Cl (22) of 2nd
annual basis. taxed annually Schedule]

3 Un-recognized Not Taxable Wholly Taxable but Wholly Taxable but not Gross amount
Provident Fund not taxed on tax on annual-basis. received, except
annual-basis. employee’s own
contribution, is taxed
at the time it is
received.

Pension

16. Any pension received by a citizen of Pakistan from a former employer is


exempt from tax, other than where the person continues to work for the employer (or
an associate of the employer).

17. If the person receives more than one such pension, the exemption applies only
to the higher of the pensions received.

Benevolent Fund

18. Any benevolent grant paid from the Benevolent Fund to the employees or
members of their families in accordance with the provisions of the Central Employee
Benevolent Fund and Group Insurance Act, 1969 is exempt.

50
Tariq Hussain Tunio

Gratuity

19. Any income representing any payment received by way of gratuity or commutation of
pension by an employee on his retirement or, in the event of his death, by his heirs is
exempt as shown in the following table.

TABLE 2.6: GRATUITY

Sr Type of Gratuity Exempt Amount


1 In the case of an employee of the Government, a The amount receivable in
Local Government, a statutory body or accordance with the rules and
corporation established by any law for the time conditions of the employee’s
being in force services
2 Any gratuity fund approved by the Commissioner Any amount receivable from
in accordance with the rules in Part III of the this gratuity fund
Sixth Schedule
3 In the case of any other employee who receives The amount not exceeding
an amount under any scheme applicable to all three hundred thousand
employees of the employer and approved by the rupees receivable
Federal Board of Revenue for the purposes of
this sub-clause
4 Any employee not covered in the above three Fifty per cent of the amount
categories. receivable or seventy-five
thousand rupees, whichever
is the less
Provided that nothing at Sr. No. 4 in this table shall apply:
(a) to any payment which is not received in Pakistan;
(b) to any payment received from a company by a director of such company who
is not a regular employee of such company;
(c) to any payment received by an employee who is not a resident individual; and
(d) to any gratuity received by an employee who has already received any gratuity
from the same or any other employer.

51
CAF-02 Tax Practices

PAST EXAMINATION QUESTIONS

Question # 1
Spring 2015 Q.3

Munir resigned from his employment with Ali Industries Limited (AIL) with effect from 31
December 2014. He received following amounts in final settlement:

■ Rs. 150,000 as Leave Encashment


■ Rs. 4,000,000 under a Golden Handshake Scheme.

Munir had received a salary of Rs. 350,000 per month for a period of six months up to
December 2014. His taxable income and tax liability during the preceding five tax years were
as under:

Tax year 2010 2011 2012 2013 2014


Total taxable 2,000,000 2,450,000 2,700,000 3,100,000 3,650,000
income (Rs)
Total tax paid 90,000 100,000 273,500 130,000 185,000
(Rs)

Required:
As a tax consultant, advise Munir about the amount of income tax payable by him for the tax
year 2015, under the Income Tax Ordinance, 2001. (06)

Question # 2
Autumn 2017 Q. 3(a)

Under the provisions of the Income Tax Ordinance, 2001 compute taxable income or loss,
under the correct head of income for tax year 2017, in each of the following cases:

(a) Under an employee share scheme, 30,000 shares of Dawood Limited were
issued to Qamar, on 1 August 2013 for Rs. 30 each. According to the scheme, he was
not allowed to sell/transfer the shares before completion of three years from the date
of issue.

The face value of each share is Rs. 10 per share. Fair market value of each share on
different dates was as follows:

1 August 2013 30 June 2016 31 July 2016


Rs. 40 Rs. 30 Rs. 50

He sold 10,000 shares on 31 May 2017 for Rs. 65 per share. (04)

Question # 3
Spring 2018 Q. 2(c)

52
Tariq Hussain Tunio

Hasrat has been working as Director HR in Shakir Limited (SL) for many years. During the
tax year 20X8 he received basic salary of Rs. 6 million. SL also contributed Rs. 50,000 per
month towards a recognized provident fund.

An equal amount was contributed by Hasrat. Interest income of Rs. 3,391,000 at the rate of
20% of accumulated balance of the fund was credited to Hasrat's account.
(04)

Question # 4
Spring 2020 Q. 5(b)

Sajid retired from Sun Chemicals Limited (SCL) as a marketing manager with effect from 31
December 2019. He received the following amounts in final settlement from SCL:

(i) Leave encashment of Rs. 600,000.


(ii) Rs. 4,000,000 from unapproved provident fund. 50% of
this amount was contributed by Sajid.
(iii) Un-approved gratuity of Rs. 2,500,000.

He also acquired the vehicle, provided/to him by SCL, at accounting written down value of
Rs. 500,000. The market value of the vehicle at the time of retirement was Rs. 2,000,000.

Required:
Under the Income Tax Ordinance, 2001 and Rules made thereunder, discuss the tax-
treatment of the above benefits received by Sajid on retirement. (04)

Question # 5
Spring 2021 Q. 4(b)(i)

On 31 December 20X1, Dr. Jamal resigned from his employment with General Hospital
Limited. In January 20X2, he received following amounts in final settlement:

■ Rs. 600,000 as leave encashment


■ Rs. 8,510,000 from recognised provident fund.
■ Rs. 1,300,000 and Rs. 1,700,000 as salary arrears relating to tax year 20W9
and 20X0 respectively.

Dr. Jamal had received a monthly salary of Rs. 500,000 from July 20X1 to
December 20X1. His taxable income and tax liability during the preceding four tax
years were as under:

Tax year 20W8 20W9 20X0 20X1


Total taxable income (Rs.) 2,800,000 3,200,000 3,800,000 4,800,000
Total tax paid 359,500 404,500 300,000 630,000

53
CAF-02 Tax Practices

Required:
As a tax consultant, advise Dr. Jamal about the amount of income tax payable by him for
the tax year 20X2, under the Income Tax Ordinance, 2001. (07)

Question # 6
Spring 2021 Q. 4(b)(i)

Briefly explain the provisions of the Income Tax Ordinance, 2001 and Rules made
thereunder relating to:
i. interest free loan provided by an employer to its employee for marriage of his/her
daughter. (02)

Question # 7
Spring 2014 Q.6(a)

State the provisions of ITO,2001 relating to foreign-source salary of resident individuals. (03)

Question # 8
Spring 2008 Q.2(b)

Mr. Waseem received an amount of Rs. 50,000 as arrears of salary pertaining to the tax
year 2011 in the tax year 2012. Discuss the options available with Mr. Waseem under the
ITO,2001 and what matters should he consider in deciding the best option. (04)

Question # 9
Autumn 2003 Q.6(a)

Under what circumstances a resident individual is entitled to claim exemption from tax on his
foreign source salary, and when is the foreign tax treated as having been paid? (04)

Question # 10
Autumn 2005 Q.8

A nationalized bank after privatization has announced a Golden Hand Shake Scheme
for its employees under which lump sum payments are proposed to be made to
employees who opt for the scheme. Discuss the chargeability of above amounts in the
hands of employees. (04)

Question # 11
Spring 2008 Q.3(b)

A company intends to launch an Employee Share Scheme for its employee and for the
purpose of educating its employees in this regards, the management wants to prepare
a summary containing the taxability of the following:
(i) Option granted to an employee.

(ii) Disposal of the option to acquire shares under the employee share scheme.

(iii) Shares issued to an employee under the option that are subject to restriction
on transfer.

54
Tariq Hussain Tunio

Explain the timing and valuation aspects in respect of the above, with reference to the
ITO,2001. (09)

Question # 12
Spring 2007 Q.1(a)

Briefly explain the taxability or exemption of the following allowances or perquisites:


(i) Leased motor vehicle provided to an employee, exclusively for his personal use.
(ii) Medical allowance paid at 10% of basic salary. (06)

55
CAF-02 Tax Practices

Chapter

04

INCOME FROM
PROPERY
CHAPTER SYNOPSIS
TOPIC SECTION
Income chargeable under the
15
head
Basis of charge 15
What is 'rent'? 15
Exclusions from income from
15
property
Fair market principle 15
Non-adjustable amount in
16
connection with building
Deductions in computing
15A
income from property
ICAP Past Papers Questions

56
Tariq Hussain Tunio

1. INTRODUCTION

Income Chargeable Under The Head

1. The rent received or receivable by a person

▪ for a tax year,


▪ except when exempt,
▪ is chargeable to tax under the head “Income from Property”.

Basis of Charge

2. Rent is charged to tax when it is 'received' or 'receivable' for a tax year.

Explanation for Students


The words ‘received’ and ‘receivable’ when used with ‘for a tax year’ show
that income from property is chargeable to tax on accrual basis. Thus,
'advance rent' pertaining to future years is not chargeable in the year of
receipt, but rather in the year to which it belongs and in which it is earned.

ILLUSTRATION
X received Rs.1.2 (m) as rent for letting on rent his office premises to ABC Ltd in Tax Year
20X1, half of which is received as advance rent for Tax Year 20X2. His taxable income for
Tax Year 20X1 is Rs. 600,000/-.

2. WHAT IS RENT?

1. Rent means:

▪ any amount received or receivable

- by the owner of land or a building


- as consideration for the use or occupation of, or the right to use or
occupy, the land or building,

▪ any forfeited deposit paid under a contract for the sale of land or a building,
and

▪ an amount treated as rent u/s 16 i.e. non-adjustable amounts received in


relation to a building.

3. EXCLUSIONS FROM INCOME FROM PROPERTY

1. Rent from sub-lease of a land or building. (Chargeable under IFOS)

2. Rent received or receivable in respect of the lease of a building together with plant
and machinery. (Chargeable under IFOS)

3. Amount included in rent for the provision of

▪ amenities,
▪ utilities or

57
CAF-02 Tax Practices

▪ any other service connected with the renting of the building. (Chargeable
under IFOS)

4. FAIR MARKET VALUE PRINCIPLE

1. A person is treated as having derived fair market rent

▪ for the period the property is let on rent

▪ in case the rent received or receivable for the property is less than the fair
market rent. .

Exclusion from Fair Market Principle

2. Fair market principle is not applicable where

▪ the fair market rent

▪ is included in the income of the lessee (i.e. tenant)

▪ under the head 'Salary'.

Explanation for Students


Fair market rent is not taken in case of an employer who owns a house and
provides it to his employee as perquisite, and fair market value (computed as
per the rules 4 i.e. alternative amount or 45% of MTS/basic salary, whichever
is higher) is added to the employee’s income under the head ‘Salary’.

In such situations, there will be no tax implication on the employer and


deemed amount of rent on the basis of fair market principle will not be added
to his income under the head ‘Income from Property’, as fair market principle
is not applicable in his case.

5. NON-ADJUSTABLE AMOUNT IN CONNECTION WITH BUILDING

Non-adjustable Amount is treated as Income

1. If the owner of a building receives from a tenant an amount that is not


adjustable against rent, the non-adjustable amount is treated as rent chargeable to
tax under Income from Property.

2. The non-adjustable amount is charged to tax

▪ in the tax year in which it is received and

▪ the following nine tax years

▪ in equal proportion.

58
Tariq Hussain Tunio

Explanation for Students


▪ It is common practice that the owner of building receives an amount not
adjustable against rent, which, being a security deposit, is refunded to the
tenant after the termination of tenancy.
▪ Non-adjustable amount is treated as income only in respect of a building
and not in respect of land.
▪ In case an amount is received by owner of a building that is adjustable
against rent, it is adjusted against monthly rent payable by the tenant and
charged to tax as regular rent. It is in the nature of ‘advance rent’, and is
not chargeable to tax separately in way the non-adjustable amount is
charged to tax.

Refund of Amount on Termination

3. Non-adjustable amount is not charged to tax in the year in which it is refunded and in
the subsequent tax year if:

▪ the non-adjustable amount is refunded by an owner to a tenant

▪ on termination of tenancy before the expiry of ten years,

Taxation of Succeeding Amount

4. If one non-adjustable amount (‘earlier amount’) is refunded by owner to an earlier


tenant on termination of tenancy before expiry of ten years and another non-
adjustable (‘succeeding amount’) is received from a succeeding tenant:

▪ the succeeding amount is charged to tax

▪ in the year in which it is received and the succeeding nine tax years

▪ in equal proportion,

▪ after reducing from it that portion of the earlier amount which was charged to tax.

59
CAF-02 Tax Practices

Chart# 1► INCOME FROM PROPERTY §.15-16

exempt under ITO-2001

Rent from sub-lease of a


land or building (Chargeable
under IFOS)

received or receivable by
If less than FMV RENT a person for a TY
exclusions
Rent received or receivable
in respect of lease of a
building together with plant
and machinery. (Chargeable
A person is treated under IFOS)
as having derived
fair market rent for
the period the Amount included in rent for
property is let on rent the provision of amenities,
in case the rent utilities or any other service
received or
means: connected with the renting of
receivable for the the building. (Chargeable
property is less than under IFOS)
the fair market rent

Fair market principle Any amount received or An amount treated


is not applicable receivable by owner of land or Any forfeited as rent u/s 16 i.e.
where the fair market building as consideration for deposit paid non-adjustable
rent is included in the - use under a contract amounts received
income of the lessee - occupation or for sale of land or in relation to a
(i.e. tenant) under - right to use or occupy land or building, and building
the head 'Salary'. building

Where owner of building receives Where an amount (“earlier Where NAA is refunded to a
a non-adjustable amount, the amount”) is refunded by owner to tenant on termination of tenancy
amount is treated as rent tenant on termination of tenancy before expiry of 10 years and the
chargeable under “IFP” in TY in before expiry of 10 years, no owner lets out the building to
which it was received and the portion of amount is allocated to another person (“succeeding
following nine tax years in equal TY in which it is refunded or to tenant”) and receives from
proportions. any subsequent TY. succeeding tenant NAA
(“succeeding amount”), the
succeeding amount as reduced
by such portion of the earlier
amount as was charged to tax is
treated as rent and charged to
tax in 10 years in equal
proportion.

60
Tariq Hussain Tunio

6. DEDUCTION IN COMPUTING INCOME FROM PROPERTY

Repair Allowance

1. A allowance in respect of repairs to a building is allowed

▪ equal to 1/5th of the rent chargeable to tax


▪ in respect of the building
▪ computed before any other deduction is allowed

Insurance Premium

2. Any premium paid or payable by the person in the year

▪ to insure the building


▪ against the risk of damage or destruction

Property Taxes

3. Any local rate, tax, charge or cess

▪ in respect of the property or the rent from the property paid or payable by the
person
▪ to any local authority or government in the year,
▪ not being income tax.

Ground Rent

4. Any ground rent paid or payable by the person in the year in respect of the
property.

Profit on money borrowed

5. Any profit paid or payable by the person in the year

▪ on any money borrowed


▪ including by way of mortgage,
▪ to acquire, construct, renovate, extend or reconstruct the property.

Share in rent and Share in Apperception in Value Paid to HBFC or Banks

6. The share in rent from the property and share towards appreciation in the value of
property (excluding the return of capital) paid or payable by the person to HBFC or a bank
where

▪ the property has been acquired, constructed, renovated, extended, or


reconstructed by the person
▪ with capital contributed by the House Building Finance Corporation (HBFC) or
a scheduled bank
▪ under a scheme of investment in property
▪ on the basis of sharing the rent made by the Corporation or bank.

61
CAF-02 Tax Practices

Profit/Interest paid on Mortgage or Charge

7. Where the property is subject to mortgage or other capital charge, the amount of
profit or interest paid on such mortgage or charge.

Rent Collection Charges

8. Any expenditure (not exceeding 4% of the rent chargeable to tax in respect of the
property for the year computed before any deduction allowed) paid or payable by the
person for the purpose of collecting the rent due in respect of the property.

Legal Services

9. Any expenditure paid or payable by the person for legal services acquired

▪ to defend the person’s title to the property or

▪ any suit connected with the property in a court.

Deduction for Recoverable Rent

10. Where there are reasonable grounds for believing that any unpaid rent in respect of
the property is irrecoverable, an allowance equal to the unpaid rent where:

(i) the tenancy was bona fide, the defaulting tenant has vacated the
property or steps have been taken to compel the tenant to vacate the
property and the defaulting tenant is not in occupation of any other
property of the person;

(ii) the person has taken all reasonable steps to institute legal
proceedings for the recovery of the unpaid rent or has reasonable
grounds to believe that legal proceedings would be useless; and

(iii) the unpaid rent has been included in 'Income from Property' for the tax
year in which the rent was due and tax has been duly paid on such
income.

11. Where any unpaid rent allowed as a deduction as above is wholly or partly
recovered, the amount recovered shall be chargeable to tax in the tax year in which it
is recovered.

General Points relating to Deduction under IFP

Double Deduction is Not Allowed

12. An expenditure that has been allowed as deduction under IFP is not allowable
as deduction under any other head of income.

62
Tariq Hussain Tunio

Chart# 2►DEDUCTIONS UNDER IFP §.15A

A allowance in respect of repairs to a building is allowed equal to 1/5 th of the rent chargeable to tax
in respect of the building computed before any other deduction is allowed

Any premium paid or payable by the person in the year to insure the building against the risk of
damage or destruction

Any local rate, tax, charge or cess in respect of the property or the rent from the property paid or
payable by the person to any local authority or government in the year, not being income tax

SPECIFIC
Any ground rent paid or payable by the person in the year in respect of the property

Any profit paid or payable by the person in the year on any money borrowed including by way of
mortgage, to acquire, construct, renovate, extend or reconstruct the property.

The share in rent from the property and share towards appreciation in the value of property
D (excluding the return of capital) paid or payable by the person to HBFC or a bank where  the
property has been acquired, constructed, renovated, extended, or reconstructed by the person 
E with capital contributed by the House Building Finance Corporation (HBFC) or a scheduled bank 
under a scheme of investment in property  on the basis of sharing the rent made by the
D Corporation or bank.
U
C Where the property is subject to mortgage or other capital charge, the amount of profit or interest
paid on such mortgage or charge.
T
I Any expenditure (not exceeding 4% of the rent chargeable to tax in respect of the property for the
year computed before any deduction allowed) paid or payable by the person for the purpose of
O collecting the rent due in respect of the property.

N Any expenditure paid or payable by the person for legal services acquired to defend the person’s
title to the property or any suit connected with the property in a court.
S
Where there are reasonable grounds for believing that any unpaid rent in respect of the property is
irrecoverable, an allowance equal to the unpaid rent where  the tenancy was bona fide, the
defaulting tenant has vacated the property or steps have been taken to compel the tenant to vacate
the property and the defaulting tenant is not in occupation of any other property of the person;  the
person has taken all reasonable steps to institute legal proceedings for the recovery of the unpaid
rent or has reasonable grounds to believe that legal proceedings would be useless; and  the
unpaid rent has been included in 'Income from Property' for the tax year in which the rent was due
and tax has been duly paid on such income.

Where any unpaid rent allowed as a deduction as above is wholly or partly recovered, the amount
recovered shall be chargeable to tax in the tax year in which it is recovered.

An expenditure that has been allowed as deduction under IFP is not allowable as deduction under any
other head of income.

The provisions of section 21 ('Deductions Not Allowed') shall apply in determining deductions allowed to a
person under IFP in the same manner as they apply to “Income from Business”
GENERAL
If a deduction is allowed for any expenditure incurred and the person has not paid the liability or a part of if
within 3 years of the end of the tax year in which the deduction was allowed, the unpaid amount of the
liability is chargeable to tax under IFP in the first tax year following the end of 3 years.

If unpaid liability is charged to tax after 3 years and subsequently the person pays the liability or a part of it,
the person is allowed a deduction for the amount paid in the tax year in which the payment is made.

63
CAF-02 Tax Practices

Deductions Not Allowed

13. The provisions of section 21 ('Deductions Not Allowed') shall apply

▪ in determining deductions allowed to a person under IFP

▪ in the same manner as they apply to “Income from Business”.

Add Back of Unpaid Liability

14. Where a person has been allowed a deduction

▪ for any expenditure incurred in deriving rent chargeable to tax under IFP and
▪ the person has not paid the liability or a part of the liability
▪ within 3 years of the end of the tax year in which the deduction was allowed,
▪ the unpaid amount of the liability shall be chargeable to tax under IFP in the first
tax year following the end of 3 years.

15. Where an unpaid liability is charged to tax after 3 years and subsequently the person
pays the liability or a part of the liability, the person is allowed a deduction for the
amount paid in the tax year in which the payment is made.

64
Tariq Hussain Tunio

PAST EXAMINATION QUESTIONS

Question # 1
Spring 2012 Q. 4(b)

Yaqoot and Loha are joint owners of a bungalow which has been rented out'for Rs. 70,000
per month.

Required:
Discuss the taxability of Yaqoot and Loha in respect of above income, in the light of Income
Tax Ordinance, 2001. (03)

Question # 2
Spring 2014 Q. 4

Bashir and Jameel jointly own a house in Karachi. Bashir has 75% share in the house. On 1
September.20X3, the house was let out at an annual rental value of Rs. 6,500,000. This
amount includes Rs. 186,000 per month for utilities, cleaning and security.

During the tax year 20X4, the owners incurred the following expenditures in relation to the
house:

Rupees
Utilities, cleaning and security 650,000
Repair and maintenance 810,000
Insurance premium 240,000
Collection charges 25,400
Mark-up on amount borrowed for extension of the house 840,000

Bashir and jameel have no other source of income. All the above expenses were incurred by
them jointly.

Required:

Calculate taxable income of Bashir and jameel under appropriate heads of income for the
tax year 20X4. (10)

Question # 3
Spring 2015 Q. 4

(a) (i) Explain the term Rent in context of 'Income from property'. (02)
(ii) Specify the head of income under which the following amounts would be
chargeable to tax:
• rent from sub lease of a building.
• amount included in rent for the provision of amenities, utilities and any other
service connected a with renting of the building. (02)

(b)On 1 july 2014, Fahim agreed to rent out a house to Mirza at a monthly rent of Rs.
180,000 with effect from 1 August 2&14 and received one year’s rent in advance. He also
received Rs. 800,000 as a security deposit, which was partly used to repay die security

65
CAF-02 Tax Practices

deposit amounting to Rs. 400,000 received from the previous tenant in July 2010 and
partly used for renovation of the house.

Fahim also incurred the following expenses in respect of the above house:

a. property tax of Rs. 15,000.


b. payment of interest amounting to Rs. 200,000 to his friend agains amount
borrowed for renovation of the house.
c. insurance premium of Rs. 110,000.
d. Rs. 5,000 per month to Wasif for collection of rent

Required:
Under the provisions of the Income Tax Ordinance, 2001 compute the taxable income of
Fahim for tax year 2015 assuming he has no other income. (07)

Question # 4
Autumn 2016 Q. 4

On 1 July 2015 Farrukh borrowed Rs. 8,000,000 from Star Bank Limited and acquired a
plot of land in Hub Industrial Zone for Rs. 6,500,000. He invested the rest of the loan in a
business venture with his friend. The above loan carries mark-up at a rate of 12% per
annum and is repayable in eight equal quarterly instalments starting from 1 July 2016. On
1 August 2015 Farrukh decided to sell the plot of land to Zulfiqar Motors for Rs.
10,000,000 and received a deposit of Rs. 500,000 from them. On 15 August 2015 Farrukh
forfeited the deposit on refusal of Zulfiqar Motors to purchase the plot of land.

On 1 September 2015 Farrukh let out the plot of land to his friend Atif at a monthly rent of
Rs. 150,000. He received an un-adjustable deposit of Rs. 200,000 from Atif and paid Rs.
80,000 for levelling the ground, Rs. 50,000 as ground rent, Rs. 12,000 as insurance
premium against the risk of damage or destruction by water logging and Rs.140,000
against rent collection charges. Farrukh had paid Rs. 25,000 to a firm of professional
valuers which determined the annual rental value of the plot of land at Rs. 2,160,000.

Required:

Under the provisions of the Income Tax Ordinance, 2001 and Rules made thereunder,
compute under the relevant head of income, taxable income of Farrukh for tax year
2016.(12)

Question # 5
Spring 2019 Q. 3(c)

On 1 July 20X8, Zahid rented out his properties as follows:

(i) An apartment was rented to Abdul Qadir at monthly rent of Rs. 40,000. Zahid
received a non-adjustable security deposit of Rs. 300,000 which was partly used to
repay the non-adjustable security deposit amounting to Rs. 175,000 received from
the previous tenant in July 20X3. He also spent Rs. 20,000 on repairs of the
apartment in February 20X9.

(ii) A bungalow was rented to a bank. Zahid and his younger brother are joint
owners of the bungalow in the ratio of 60:40 respectively. The annual rent agreed

66
Tariq Hussain Tunio

with the bank was Rs. 6,000,000 which is inclusive of Rs.100,000 per month for
utilities, cleaning and security. Zahid paid Rs. 35,000 per month for providing these
services.

Required:

Under the provisions of Income Tax Ordinance, 2001 compute total and taxable income of
Zahid for the tax year 20X9 under appropriate heads of income. (07)

Question # 6
Autumn 2019 Q. 2

(a) Explain the term 'Rent' with relation to ‘Income from property’. (02)
(b) During the tax year 20X9, Amjad carried out the following transactions in respect of
his properties:

(i) On 1 July 20X8, Amjad purchased a factory building in Sukkur along with
the installed machinery at the price of Rs. 9 million and Rs. 3 million
respectively. To manage the shortage of funds of Rs.. 2,000,000, he
borrowed the same on 1 July 20X8 from his friend Shamshad through a
crossed cheque. The loan carries interest at the rate of 18% per annum.

On 1 January 20X9, he let out this building along with the machinery to
Basit at a monthly rent of Rs. 500,000 payable in advance.

(ii) On 1 July 20X8, Amjad let out his residential property situated in DHA
Karachi to Mirza Limited at a monthly rent of Rs. 300,000. Rent for the two
years was received in advance on 1 August 20X8.

(iii) On 1 July 20X8, Amjad also entered into an agreement with Zeeshan for
the sale of his plot situated in Quetta for Rs. 50 million. The plot had been
purchased for Rs. 40 million in 20X4. Under the terms of sale agreement,
he received Rs. 5 million at the time of signing the agreement and the
balance was to be received on 30 September 20X8. However, due to
financial difficulties, Zeeshan failed to pay the balance amount on the due
date and consequently, Amjad forfeited the advance in accordance with
the terms of the agreement.

On 10 April 20X9, he finally sold the plot to Jamshed for Rs. 65 million.

(iv) Following expenditures were incurred by Amjad in respect of his


properties in Sukkur and Karachi:

Details of expenditures Property situated in


Sukkur Karachi
Repair & maintenance – building 270,000 70,000
Repair & maintenance - machinery 50,000 -
Ground rent 50,000 10,000
Insurance – building 150,000 20,000
Total 520,000 100,000
Required:

67
CAF-02 Tax Practices

In view of the provisions of the Income Tax Ordinance, 2001 compute under appropriate
head of income, taxable income of Amjad for the tax year 20X9. (10)

Question # 7
Autumn 2019 Q. 3(e )

Farhan and Imran jointly own a building in Quetta. The Building has been rented out to a
company. Discuss the tax treatment of income from such property. (02)

Question # 8
Autumn 2002 Q.3(a)

What is chareable to tax under the head “income from property’’? (01)

Question # 9
Autumn 2006 Q.1(a)

Describe the provisions of the Income Tax Ordinance,2001 regarding non-adjustable amount
received from a tenant by the owner of a building. (05)

Question # 10
Spring 2015 Q.4(a)

(i) Explain the term ‘Rent’ in context of Income from Property.


(ii) Specify thehead of income under which the following amounts would be chargeable
to tax:
• Rent from sub lease of a building
• Amount included in rent for the provision of amenities, utilities and any
other service connected with renting of the building.
Question # 11
Autumn 2002 Q.4

ABC Associates owns a building which is on rent. The following information is available:

 Rent received from tenants 2,300,000


 Depreciation on building under the 3 rd schedule to the 400,000
Ordinance
 Property tax 100,000
 Municipal/local government taxes (agreements with tenants 100,000
provide that tenant should pay the municipal taxes)

Rent received includes Rs.600,000 for three year commencing from July 01 of the
current year. ABC Associates follow accrual basis of accounting and its income year is
July-June 20x8.

Required:
Compute the income from property of ABC Associates.

Question # 12
Winter 2012 Q.4(a)

68
Tariq Hussain Tunio

Mr. Sohail, a resident individual, owns a building in Clifton area of Karachi. On 1.10.2021 he
rented out the building to Mr. Baqir at an annual rent of Rs. 1,200,000. This amount include
Rs. 15,000 per month for arranging two security guards for the building. Following expenses
were incurred by Mr. Sohail on the building during the tax year 2022:

Repairs and renovation Rs. 35,000


Property tax 20,000
Insurance premium 10,000
Rent collection charges 3,000

Mr. Sohail also paid a salary of Rs. 4,000 per month to each of the two security guards at
building.

Required:
Under the provision of ITO,2001 calculate the tax liability of Mr. Sohail under the appropriate
heads of income for the tax year 2022.(06)

Question # 13
Spring 2003 Q.5

Mr. Amir-ud-din has recently constructed an office complex for the purposes of letting out.
The office complex is also equipped with its own electric generators for which tenants are
separately charged on a monthly basis. As per terms and conditions, Mr. Amir-ud-din is also
entitled to signing amount, which is non-refundable. For the tax year 20X7 the following
Information has been provided to you for the computation of his income from property and
tax liability thereon: (10)

Amount in
Rs.
Rent for the year already received 1,150,000
Rent for the year though due but irrecoverable 50,000
Signing amount (non-adjustable/non-refundable) 1,000,000
Fire and water tax paid to the local authority 20,000
Lawyer’s fee for suit to recover rent 50,000
Lawyers fee for drafting master rent agreement 10,000
Salary of the caretaker who also collects monthly rent 36,000
Insurance premium being one per cent of market value 200,000
of the property
Repair maintenance expenditure 50,000

69
CAF-02 Tax Practices

Chapter

05

INCOME FROM
BUSINESS
CHAPTER SYNOPSIS
TOPIC SECTION
What is business? 2(9)
Structure of taxation of business income General
Incomes covered under income from 18
business
Structure of deductions under IFB General
Deductions in computing IFB 20
Depreciation 22
Initial allowance 23
Intangibles 24
Pre-commencement Expenditure 25
Scientific Research Expenditure 26
Employee Training & Research 27
Profit on Debt 28
Deduction for Bad Debts 29
Deduction not Allowed 21
Speculation Business 19
Tax Accounting 32
Cash-basis Accounting 33
Accrual-basis Accounting 34
Stock-in-Trade 35
ICAP Past Papers Questions

70
Tariq Hussain Tunio

1. DEFINITION OF BUSINESS

1. “Business” include:

▪ Any trade, commerce, manufacture, profession, vocation, or

▪ An adventure or concern in the nature of trade, commerce, manufacture,


profession or vocation, but

▪ does not include employment

Explanation for Students


Business includes not only trade, commerce, manufacture, profession, or
vocation but also any activity that in itself is not trade, commerce,
manufacture, profession, or vocation but which possesses some features
essential to trade, commerce, manufacture, profession, or vocation. Such
activities are described as adventure or concern in the nature of trade,
commerce, manufacture, profession, or vocation.

ILLUSTRATION
X, a full time teacher, derived income amounting to Rs. 200,000/- from one-off sale of
air conditioners, purchased in winter to make some profit. This venture of X is not
trade, but is in the nature of trade. Hence, X’s income from sale of air conditioners is
chargeable to tax under Income from Business.

2. STRUCTURE OF TAXATION OF BUSINESS INCOME

1. Income from Business deals with two types of businesses:

(1) Normal business


(2) Speculation business.

While computing ‘Income from Business’, speculation business is treated as separate


and distinct.

2. Any profit or gain from speculation business is included in income under the head
Income from Business.

3. Whereas loss from speculation business (‘speculation loss’) is kept separate and carried
forward to next six tax years to be set of only against speculation income of the person.

COMPUTATION OF INCOME FROM BUSINESS

(A) NORMAL BUSINESS


Amounts Chargeable (§.18) xx
L: Deductions Allowed (§.20-29) (xx)
Income from Normal Business xx
(B) SPECULATION BUSINESS
Amounts Chargeable(§.19) xx

71
CAF-02 Tax Practices

L: Deductions Allowed (§.20-29) (xx)


Income from Speculation Business xx
Income from Business xx

3. INCOMES CHARGEABLE UNDER INCOME FROM BUDINESS §.18

1. The profits and gains of any business carried on by a person at any time in the year.

2. Any income derived by any trade association, professional association, or similar


association from the sale of goods or provision of services to its members.

Examples

Karachi Chamber of Commerce and Industry, Pakistan Sports Association etc.

Explanation:

It is clarified that income derived by a co-operative societies from the sale of goods,
immovable property or provision of services to its members is and has always been
chargeable to tax under the provisions of Income Tax Ordinance,2001.

3. Any income from hire or lease of tangible movable property

Example

Income from rent-a-car business.

4. The fair market value of any benefits or perquisite (whether convertible into money or
not) derived by a person in the course of or by virtue of a past, present, or
prospective business relationship.

Explanation: A benefit derived by a person when his ‘debt’ or ‘profit on debt’ is


waived by any bank under any scheme of State Bank of Pakistan (including under
SBP Circular No. 29/2002) is included in the benefit derived by the person in a
business relationship, hence, chargeable to tax under Income from Business.

5. Any management fees derived by a management company including a


modarba management company.

Examples

A hotel management company engaged in managing hotels, or an asset


management company (AMC) engaged in managing mutual funds.
6. Any profit on debt derived by a person whose business is to derive profit on
debt.

Example

Banks

7. An amount received or receivable


▪ by

72
Tariq Hussain Tunio

 a scheduled bank,
 investment bank,
 development finance institution,
 modaraba, or
 a leasing company,

▪ in connection with

o assets (whether or not owned by any of them) and


o leased to another person.

ILLUSTRATION
XYZ Ltd, being a leasing company, received Rs. 35 (m) in TY 20X1 in connection
with lease of assets. Out of this amount, Rs. 25 (m) were received as principal
amount, whereas Rs. 10 (m) represented mark up. XYZ Ltd’s income under the head
Income from Business for TY 20X1 is Rs. 35 (m).

8. Any amount received

▪ by  a banking company or  a non-banking finance company,

▪ where such amount represents distribution by

o a Mutual Fund or
o a Private Equity and Venture Capital Fund

▪ out of its income from profit on debt.

ILLUSTRATION
XYZ Ltd, a Non-banking Finance Company, received Rs. 400,000/- in 20X1 as
dividend from ABC Income Fund, a mutual fund engaged in deriving income from
profit on debt. This dividend, being distribution from a mutual fund out of its profit on
debt income, is chargeable to XYZ Ltd under Income from Business.

73
CAF-02 Tax Practices

Chart# 1► BUSINESS DEDUCTIONS §.18

Profits & gains of any business carried on by a person at any time in TY


A
M Any income derived by a trade, professional, or similar association from the sale of goods or
O provision of services to its members
U
N Any income from hire or lease of tangible movable property
T
FMV of any benefits or perquisite (whether convertible into money or not) derived by a person
S
in the course of or by virtue of a past, present, or prospective business relationship

C
Any management fees derived by a management company including a modarba management
H company
A
R Any profit on debt derived by a person whose business is to derive profit on debt
G
E
An amount received or receivable by  a scheduled bank,  investment bank, 
A development finance institution,  modaraba, or  a leasing company, in connection with
B assets (whether or not owned by any of them) and leased to another person
L
E Any amount received by  a banking company or  a non-banking finance company, where
such amount represents distribution by a Mutual Fund or a Private Equity and Venture Capital
Fund out of its income from profit on debt

LESS: [Normal]
Depreciation
General Depreciation
Principles Initial
Allowance
Intangibles
D Allowed
E
D P.C.E
U
C Special Scientific
Deductions Research Exp
T
I Employee
O Training Exp

N Not
Allowed
S POD

Bad
Debts

INCOME FROM BUSINESS

5. GENERAL PRINCIPLES:
DEDUCTIONS IN COMPUTING INCOME FROM BUDINESS §.20

1. In computing income from business, a deduction is allowed for

▪ an expenditure
▪ incurred by a person in a tax year
▪ wholly and exclusively for the purpose of business

2. Any expenditure incurred by a person in a tax year wholly and exclusively for
the purpose of business in connection with:

74
Tariq Hussain Tunio

▪ acquisition of depreciable assets,


▪ acquisition of an intangible with a useful life of more than one year, or
▪ pre-commencement expenditure

is depreciated or amortized in accordance with the provisions of the Income Tax


Ordinance, 2001 (i.e. § 22, 23, 24, and 25).

3. A deduction is allowed for any expenditure incurred by an amalgamated company on


legal and financial advisory services and other administrative cost relating to planning
and implementation of amalgamation.

4. Cost incurred in acquiring animals which are used for the purpose of business or
profession otherwise than as stock-in-trade but which have died or become
permanently useless for such purpose, is allowed as deduction as per following
formula.
Act u a l Co st o f t h e An im a l t o t he T a xpa ye r
L: Amount realized in respect of Carcasses/ Animal
Amount Allowed as Deduction in Income from Business

Chart# 2► GENERAL PRINCIPLES: DEDUCTIONS UNDER IFB §.20

GENERAL PRINCIPLES:
DEDUCTION UNDER IFB

ALL EXPENDITURES: AMALGAMATION EXPENDITURES: ANIMAL EXPENDITURES:


CAPITAL EXPENDITURES:
In computing income from A deduction is allowed for any Cost incurred
An expenditure incurred
business, a deduction is expenditure ▪ in acquiring animals
wholly and exclusively for
allowed for an ▪ incurred by an amalgamated ▪ which are used for the
the purpose of business in
▪ expenditure connection with company purpose of business or
▪ incurred by a person in ▪ acquisition of ▪ on legal and financial advisory profession
a tax year services and ▪ otherwise than as stock-in-
depreciable assets,
▪ wholly and exclusively ▪ acquisition of an ▪ other administrative cost trade
for the purpose of relating to planning and ▪ but which have died or
intangible with useful life
business implementation of become permanently
of > 1 year,
amalgamation. useless for such purpose,
▪ pre-commencement
▪ is allowed as deduction as
expenditure
per following formula.
▪ is depreciated or
amortized.

Act u a l Co st o f t h e An im a l t o t he T a xpa ye r
L: Amount realized in respect of Carcasses/ Animal
Amount Allowed as Deduction in Income from Business

6. DEPRECIATION §.22

1. A person is allowed a deduction for the depreciation of the person’s depreciable


asset used in person’s business in the tax year.

75
CAF-02 Tax Practices

2. For claim of depreciation deduction, following conditions must be fulfilled:

▪ The asset has to be a depreciable asset.

▪ Ownership of depreciable asset belongs to the person.

▪ The depreciable asset is used (fully or partially) in person’s business.

Explanation for Students


In allowing depreciation deduction for a tax year, full year policy is followed
i.e. even if the depreciable asset is not used for the whole tax year,
depreciation deduction for full year is allowed.

3. Where a depreciable asset is jointly owned by a taxpayer and an Islamic financial


institution licensed by the State Bank of Pakistan or Securities and Exchage
Commission of Pakistan, as the case may be, pursuant to an arrangement of
Musharika financing or diminishing Musharika financing, the depreciable asset shall
be treated to be wholly owned by the taxpayer. §.22(15)

4. The total deductions allowed to a person in respect of a depreciable asset cannot


exceed the cost of that asset. –§.22(7)

Definition of Depreciable Asset

5. Depreciable asset means

▪ any  tangible movable property,  immovable property (other than


unimproved land), or  structural improvement to immovable property

▪ owned by a person

▪ that

o has normal useful life exceeding one year,


o is likely to lose value as a result of normal wear and tear, or
obsolescence, and
o is used wholly or partly by the person in deriving income from
business chargeable to tax.

▪ but shall not include

o any tangible movable property,


o immovable property, or
o structural improvement to immovable property
o in relation to which
• a deduction has been allowed under another section of
ITO-2001
• for the entire cost of the property or improvement
• in the tax year in which the property is acquired or
improvement made.

76
Tariq Hussain Tunio

What is Structural Improvement? §22(15)

6. ‘Structural improvement’ includes any building, road, driveway, car park, railway line,
pipeline, bridge, tunnel, airport runway, canal, dock, wharf, retaining wall, fence,
power lines, water or sewerage pipes, drainage, landscaping or dam.

7. Cost of a depreciable asset, being immovable property or structural improvement,


does not include the cost of land. –§22(13)(b)

Chart# 3► DEPRECIABLE ASSET

Depreciable Asset

Tangible Movable Immovable Property Structural


(except unimproved
Property Improvement
land)

Normal Useful Likely to lose value Fully or partly used in


due to wear/tear & deriving income from
Life > 1 year obsolescence business

Computation of Depreciation Deduction

8. Depreciation deduction for a tax year is computed

▪ by applying the prescribed rates of depreciation

▪ against the written down value of the depreciable asset

o at the beginning of a tax year.

Computation of Written Down Value (WDV)

9. Written down value at the beginning of a tax year is computed in the following way:

▪ WDV for the tax year in which depreciable asset is acquired:

Written Down Value = Cost – Initial Allowance

▪ WDV for the subsequent tax year:

WDV = Cost – Total Depreciation Deductions including Initial Allowance


Rates of Depreciation

77
CAF-02 Tax Practices

10. Depreciation rates are provided in the Third Schedule to the Income Tax Ordinance,
2001, as follows:

S# DEPRECIABLE ASSET RATES


I. Building (all types) 10%
II. Furniture (including fittings) and machinery and plant (not
otherwise specified), Motor vehicles (all types), ships,
technical or professional books 15%
III. Computer hardware including printer, monitor and allied
items, machinery and equipment used in manufacture of I.T.
products, aircrafts and aero engines 30%
IV. In case of mineral oil concerns the income of which is liable to
be computed in accordance with the rules in Part-I of the Fifth
Schedule.
(a) Offshore platform and production installations 20%
V A ramp built to provide access to person with disabilities not
exceeding Rs. 250,000 each 100%

Disposal of Depreciable Asset

11. No depreciation deduction is allowed in a tax year in which depreciable asset is


disposed by a person.

12. If the consideration received on disposal of a depreciable asset exceeds its WDV at
the time of disposal, the result is gain, which is chargeable to tax under Income from
Business.
13. However, if consideration received on disposal of a depreciable asset is less than its
WDV, the result is loss, which is allowed as deduction in computing Income from
Business.

SPECIAL CASE: 1

Partial Business Use of a Depreciable Asset

14. Depreciation deduction allowable in case a depreciable asset is wholly used to


derive income from business is not fully allowable in case the depreciable asset is
partly used in deriving income from business and partly for another use.

15. Depreciation deduction in this case is restricted to the fair proportional part of
the depreciation deduction that would have been allowed, had the asset been wholly
used to derive income from business.

How to Compute WDV of Partial Business Use Depreciable Asset

16. Written Down Value of a depreciable asset that is partly used in deriving
Income from Business is computed on the basis that the asset has been solely used
to derive income chargeable to tax under Income from business.

17. Thus, depreciation deduction computed for each tax year (both allowed and
not allowed) is deducted from the cost of the asset.

78
Tariq Hussain Tunio

Addition of Disallowed Amount at the Time of Disposal

18. In case of disposal of a partial-business-use depreciable asset, the Written Down


Value of the asset as computed above is increased by the amount of depreciation
that has not been allowed on account of its non-business use.

Cost of the Depreciable Asset


Less: Total Depreciation Deductions
Add: Disallowed Amount of Depreciation
Written Down Value of the Asset

SPECIAL CASE: 2

Passenger Transport Vehicle Not Plying for Hire

19. For the purpose of depreciation deduction, the cost of a passenger transport vehicle
not plying for hire shall not exceed Rs. 7.5 (m).

20. Where such vehicle is disposed, the consideration received for the purposes of
computing gain or loss is to be computed as per the following formula:

Amount received on Rs. 7.5 (m)


X
disposal of the Vehicle Actual cost of Acquiring the Vehicle

SPECIAL CASE: 3

Disposal of Immovable Property

21. The consideration received on disposal of a property is treated as cost of that


property, if:

▪ the property disposed is immovable


▪ consideration received on its disposal exceeds the cost of the property,

SPECIAL CASE: 4

Export or Transfer of a Depreciable Asset

22. Where a depreciable asset that has been used by a person in Pakistan is exported or
transported out of Pakistan, the person is treated as:
▪ having disposed of the asset at the time of export or transfer
▪ for consideration received equal to the cost of the asset.

79
CAF-02 Tax Practices

Charter# 4► DEPRECIATION §.22

The total deductions allowed to a person in respect of a


depreciable asset cannot exceed the cost of that asset.

 Depreciation deduction is total deductions allowed


DEPRECIABLE  allowed i/r/o depreciable i/r/o depreciable asset
 DEPRECIATION
asset used in person’s cannot exceed the cost
ASSETS business in TY of that asset


Means Computation: by applying depreciation
rates against written down value
any  tangible movable property 
immovable property (other than
unimproved land) or  structural First Year Subsequent Years
improvement to immovable property
owned by a person that  has NUL Cost Cost
>1 yr  is likely to lose value due to L: Total Depreciation including
normal wear & tear or obsolescence L: Initial Allowance
Initial Allowance
and  is used wholly or partly in
deriving income from business Written Down Value Written Down Value
chargeable to tax.

Passenger Transport Vehicle:


but shall not include any tangible For the purpose of depreciation deduction, the cost of a passenger
movable property, immovable transport vehicle not plying for hire shall not exceed Rs. 7.5 (m).
property, or structural improvement to In case of disposal of such vehicle, consideration received is
immovable property in relation to computed as follows:
which a deduction has been allowed
Rs. 7.5 (m)
under another section of ITO-2001 for Amount received on
X
the entire cost of the property or SPECIAL disposal of the Vehicle Actual cost of Acquiring the Vehicle
improvement in the tax year in which CASES
the property is acquired or
improvement made Export/Transfer Outside Pakistan of Depreciable Assets:
Where a depreciable asset that has been used by a person in
Pakistan is exported or transported out of Pakistan, the person is
treated as having disposed of the asset at the time of export or
‘Structural improvement’ includes transfer for consideration received equal to the cost of the asset.,
any building, road, driveway, car
park, railway line, pipeline, bridge,
tunnel, airport runway, canal, dock, Disposal of Immovable Property:
wharf, retaining wall, fence, power The consideration received on disposal of a property is treated as
lines, water or sewerage pipes, cost of that property, if the property disposed is immovable and
drainage, landscaping or dam. consideration received on its disposal exceeds the cost of the
property.
Cost of a depreciable asset, being
immovable property or structural
Depreciable Assets Used Partly in Business:
improvement, does not include the
In case of partial business use, depreciation deduction allowed is
cost of land.
restricted to fair proportional part of the amount that would be
allowed if the asset was wholly used to derive income from
business chargeable to tax

WDV in this case is computed on the basis that the asset


has been solely used to derive IFB chargeable to tax
DISPOSAL

On disposal WDV is increased by the amount not


allowed due to partial business use

If consideration received on If consideration received on


No depreciation
disposal of depreciable asset disposal of a depreciable
deduction is allowed in
exceeds its WDV at the time asset is less than its WDV, the
a tax year in which
of disposal, the result is gain, result is loss, which is allowed
depreciable asset is
which is chargeable to tax as deduction in computing
disposed by a person.
under Income from Business. Income from Business

80
Tariq Hussain Tunio

Taxation of leasing Business

23. Depreciation deduction in respect of a leased asset is allowed only to a lessor (being
a leasing company, an investment bank, a modaraba, a scheduled bank, or a
development finance institution).

24. The depreciation deduction of a leased depreciable asset is deductible only against
lease rental income derived by a lessor in respect of the leased assets.

25. Any asset owned and leased by a lessor (being a lasing company, an investment
bank, modaraba, scheduled bank or development finance institution) is treated as
being used by the lessor in deriving his/its taxable income (both in case of finance as
well as operating lease). –§22(13)(c)

7. INITIAL ALLOWANCE §.23

1. An initial allowance deduction equal to 25% of the cost of an eligible depreciable


asset (EDA) (being plant and machinery) is allowed

▪ to a person
▪ who places an EDA into service in Pakistan
▪ for the first time in a tax year

What is an EDA?

2. Eligible depreciable asset means a depreciable asset other than the following:

(i) Road transport vehicle not plying for hire (i.e. a vehicle that is used in
person’s own business and cannot be hired by others for fare)

(ii) Furniture and fittings

(iii) Plant or machinery that has been used previously in Pakistan


(i.e. second-hand plant or machinery purchased from within Pakistan.)

(iv) Plant or machinery in relation to which a deduction has been allowed


under another section of ITO-2001 for the entire cost of the asset in the
tax year in which the asset is acquired.

(v) Immovable property or structural improvement to the immovable property.

Year in Which Initial Allowance is Allowed

3. The initial allowance is allowed in later of:

▪ the tax year in which the EDA is used for the first time, or
▪ the tax year in which commercial production is commenced.

ILLUSTRATION
ABC Ltd purchased a new plant for the manufacturing of motorcycle engines in Lahore. It
started trial production in March 20X1. Commercial production was started in July 20X1. In
this case initial allowance will be allowed in Tax Year 20X2.

81
CAF-02 Tax Practices

Initial Allowance in Respect of Leased Asset

4. Initial allowance in respect of a leased asset

▪ is allowed
▪ only to a lessor being

o a leasing company,
o an investment bank,
o a modaraba,
o a scheduled bank, or
o a development finance institution.

5. The initial allowance is deductible only against

▪ lease rental income


▪ derived by a lessor in respect of leased assets.

Chart# 5► INITIAL ALLOWANCE §.23

IA is allowed in later of  tax year in


IA deduction is allowed to a which EDA is used for the first time, or 
person who places an EDA
INITIAL into service in Pakistan
the tax year in which commercial
ALLOWANC production is commenced.
E (IA)

equal to 25% of cost


of

If EDA is o a leasing company


Eligible Depreciable o an investment bank
Asset (EDA) (Being leased
by o a modaraba
plant & machinery) o a scheduled bank, or
o a development finance
institutions

Eligible depreciable
asset means a
depreciable asset
other than the Initial allowance is
following: allowed to the
lesssor

which is deductible
2.Furniture & 4.Plant or only against lease
1.Road fittings machinery in rental income
transport 3.Plant or
relation to derived by a lessor in
vehicle not machinery
which a respect of leased
plying for hire that has been
deduction has assets
(i.e. a vehicle used
been allowed
that is used in previously in
under another
person’s own Pakistan
section of ITO-
business and 2001 for the
cannot be entire cost of
hired by the asset in the
others for 5.Immovable property or tax year in
fare) structural improvement to the which the asset
immovable property is acquired.
82
Tariq Hussain Tunio

8. INTANGIBLES §.24

1. Amortization deduction is allowed to a person in respect of cost of person’s intangible


asset that:

▪ is wholly or partly used in deriving income from business chargeable to tax,


and

▪ has normal useful life exceeding one year.

2. Total amortization deduction allowed in the current as well all previous tax years
cannot exceed the cost of the intangible. –§24(7)

Definition of Intangibles

3. Intangible means:

▪ any patent, invention, design or model, secret formula or process, copyright,


trade mark, scientific or technical knowledge, computer software, motion
picture film, export quotas, franchise, license, intellectual property, or other
like property or right, contractual rights, and

▪ any expenditure that provides an advantage or benefit for a period of more


than one year

▪ other than expenditure incurred to acquire a depreciable asset or unimproved


land.

▪ But shall not include self-generated goodwill or any adjustment arising on


account of accounting treatment.

Computation of Amortization Deduction

4. Amortization deduction is computed in accordance with the following formula:

Amortization Deduction = Cost / Normal Useful Life of Intangible in Whole Years

5. Cost of intangible means any expenditure incurred in acquiring or creating the


intangible, including any expenditure incurred in improving or renewing the intangible.
–§24(11)

6. If entire cost of an intangible is allowed as deduction under the Income Tax


Ordinance, 2001, in the year in which it is acquired, no amortization deduction is
allowed, for the reason that the cost has been fully amortized in one go. –§24(2)

7. An intangible that does not have an ascertainable useful life shall be treated as if it
had a normal useful life of 25 years.

Restricting Amortization to Number of Days Intangible is Used

8. In case an intangible is not used for the whole of a tax year in deriving income from
business, the amortization deduction is restricted to the number of days the intangible was
used in the year, as per the following formula:

83
CAF-02 Tax Practices

No. of days the Intangible is used


Amortization deduction x
Total days in a year

What is meant by ‘use’ of an Intangible Asset?

9. An intangible is treated as used on a day if it is available for use on that day,


including the non-working day.
Explanation for Students
Days on which an intangible is used in deriving income from business include
the days on which it is:

▪ actually used
▪ available for use

Examples of days on which the intangible is not available for use:

▪ In case of acquisition of intangible during a tax year, it is not available


for use on days of the year preceding the acquisition of the intangible.
▪ Intangible is not available for use in case of a fault or defect therein
e.g. virus in software.
▪ Intangible is not available for use in case of a legal restriction on its
use.

Disposal of Intangible

10. No amortization deduction is allowed in a tax year in which the intangible is disposed
of.

11. If the consideration received on the disposal of an intangible exceeds its WDV at the
time of disposal, gain arises, which is chargeable to tax under Income from Business
in the tax year in which disposal is made.

12. If consideration received is less than the WDV of an intangible at the time of its
disposal, loss is incurred. Loss is allowed as deduction in computing income from
business in the tax year in which disposal is made.

Partial Business Use of an Intangible

13. Amortization depreciation for intangibles allowable in case an intangible is


wholly used to derive income from business is not fully allowable in case the
intangible is partly used to derive income from business and partly for another use.

14. Amortization deduction in this case is restricted to the fair proportional part of
the deduction that would have been allowed, had the intangible been wholly used to
derive income from business.

Computation of WDV for the Purpose of Disposal

15. For the purposes of computing gain or loss at the time of disposal of an
intangible, its written down value is computed as follows:

84
Tariq Hussain Tunio

▪ In case of full business use of the intangible:


Written Down Value = Cost – Total Amortization Deductions Allowed

▪ In case of partial business use of the intangible:

WDV = Cost – Total amortization deductions that would have been


allowed if the intangible was fully used in business

11. PRE-COMMENCEMENT EXPENDITURE §.25

1. A person is allowed a deduction for pre-commencement expenditure by amortizing


the expenditure on straight-line basis at the rate of 20% every year starting from the
first tax year after commencement of the person’s business.

2. Total deductions allowed in respect of pre-commencement expenditure in the current


tax year and all previous tax years cannot exceed the amount of the pre-
commencement expenditure.

3. No deduction is allowed under pre-commencement expenditure where a deduction


has been allowed , under ITO-2001, for the entire amount of the pre-commencement
expenditure in the tax year in which it is incurred.

Definition of Pre-commencement Expenditure

4. Pre-commencement Expenditure means any expenditure that:

▪ is incurred before the commencement of business


▪ is incurred wholly and exclusively to derive income chargeable to tax
▪ is cost of feasibility studies, construction of prototypes, and trial production
activities

5. is not any expenditure that is incurred in acquiring land, depreciable asset or an


intangible.

12. SCIENTIFIC RESEARCH EXPENDITURE §.26

1. A deduction is allowed for scientific research expenditure incurred in Pakistan wholly


and exclusively for the purpose of deriving income from business chargeable to tax.

Definition of Scientific Research Expenditure

2. Scientific research expenditure means any expenditure that is incurred by the person
(including person’s contribution to any scientific research institution) on:

▪ scientific research undertaken in Pakistan

▪ for the purposes of developing the person’s business

Other than:

85
CAF-02 Tax Practices

▪ expenditure incurred in the acquisition of any depreciable asset or intangible

▪ expenditure incurred in the acquisition of immovable property

▪ expenditure incurred for the purpose of ascertaining the existence, location,


extent or quality of a natural deposit

Definition of Scientific Research

3. Scientific Research means any activity undertaken in Pakistan in the fields of natural
or applied science for the development of human knowledge. –§26(2)

Definition of Scientific Research Institution

4. Scientific Research Institution means any institution certified by the FBR as


conducting scientific research in Pakistan.—§26(2)

13. EMPLOYEE TRAINING AND FACILITIES §.27

1. A person is allowed a deduction for any expenditure (other than capital expenditure)
incurred in a tax year in respect of:

▪ any educational institution or hospital in Pakistan established for the benefit of


the person‘s employees and their dependents.

▪ any institute in Pakistan established for the training of industrial workers


recognized, aided, or run by the Federal/Provincial/Local Government.

▪ the training of any person, being a citizen of Pakistan, in connection with


a scheme approved by the FBR for the purposes of this deduction.

14. PROFIT ON DEBT §.28

Profit on Debt used for Business Purposes

1. A deduction is allowed in a tax year for any profit on debt incurred by a person
in the tax year to the extent that the proceeds or benefit of the debt have been used
by the person for the purposes of business.

Lease Rental Deduction to a Lessee

2. A deduction is allowed to a lessee for any lease rental incurred by a person in


the tax year to a scheduled bank, financial institution, an approved modaraba, an
approved leasing company or a Special Purpose Vehicle on behalf of the Originator
for an asset used by the person for the purposes of business. [section 28(1)(b)].

Defintions

3. Approved leasing company means a leasing company approved by the


board for the purposes of clause (b) of the sub-section (1) of section 28.

86
Tariq Hussain Tunio

4. Approved Modaraba means a modaraba approved by the board for the purposes of
clause (b) of the sub-section (1) of section 28.

5. The deduction on account of lease rentals the cost of a passenger transport vehicle
not plying for hire to the extent of principal amount shall not exceed two and a half
million rupees.

Deduction for amounts incurred to Modaraba and PTC Holders

6. A deduction is allowed for any amount incurred by a person in the tax year to a
modaraba or a participation term certificate (PTC) holder for any funds borrowed and
used by the person for the purposes of Business.

Deduction to Scheduled Banks relating to PLS Account

7. A deduction is allowed for any amount incurred by a scheduled bank in the tax year
to a person maintaining a profit or loss sharing account or a deposit with the bank as
a distribution of profits by the bank in respect of the account or deposit.

15. BAD DEBTS §.29

1. A deduction is allowed to a person for bad debts if all of the following conditions are
fulfilled:

▪ The amount of bad debt was previously included in person’s income from
business chargeable to tax, or it is in respect of money lent by financial
institutions in driving income from business.

▪ The amount of bad debts is written off in the person’s books of accounts.

▪ There are reasonable grounds to believe that the debt is irrecoverable.

Bad Debts not to exceed the Amount Written Off in Books

2. The amount of the deduction allowed to a person for bad debts shall not exceed the
amount of the debt written off in the books of accounts of the person in that tax year.

Recovery of Bad Debts

3. In case a bad debt amount is recovered by a person, after having been previously
allowed as deduction, it is chargeable to tax in the tax year in which it is recovered.

Recovery of Bad Debt: Situations No. 1

4. Where the amount received exceeds the difference between the whole of such bad debt
and the amount previously allowed as a deduction under this section, the excess shall be
included in the person’s income under the head “Income from Business” for the tax year in
which it was received.

Recovery of Bad Debt: Situations No. 2

87
CAF-02 Tax Practices

5. Where the amount received is less than the difference between the whole of such
bad debt and the amount allowed as a deduction under this section, the shortfall shall
be allowed as a bad debt deduction in computing the person’s income under the
head “Income from Business” for the tax year in which it was received.

16. DEDUCTION NOT ALLOWED §.21

1. Income Tax Paid or Payable

Any income tax paid or payable by a person in Pakistan or a foreign country.

2. Income Tax Deducted/Collected at Source

Any amount of income tax deducted from the person at source.

3. Payment For Certain Expenses Without Deduction Of Tax At Source

Expenditure from which the person is required to deduct or collect tax under the
Income Tax Ordinance, 2001, unless the person has paid or deducted and paid the
tax

Disallowance in respect of purchases of raw materials and finished goods under this
clause shall not exceed 20% of purchases of raw materials and finished goods

Recovery of any amount of tax under sections 161 or 162 shall be considered as tax
paid.

4. Commission

Amount of commission paid or payable in respect of supply of products listed in the


3rd schedule of the sales tax Act,1990, where the amount of commission paid or
payable exceeds 0.2% of the gross amount of supplies thereof unless the person to
whom commission is paid or payable, as the case may be, is appearing in the active
taxpayer list under the Income Tax Ordinance,2001.

5. Entertainment Expenditure

Entertainment Expenditure in excess of limits or in violation of conditions laid down in


the Income Tax Rules, 2002.

Rule 10 of the Income Tax Rules, 2002, deals with these conditions. It defines
entertainment as follows:

Entertainment Expenditure means the provision of meals, refreshments, and


reasonable leisure facilities in accordance with the tradition of business and
subject to overall norms and customs of business in Pakistan. ‹R#10›

A person is allowed a deduction for entertainment expenditure only when it is


incurred on the entertainment of persons related directly to the person’s business
and fulfills one of the following conditions:

88
Tariq Hussain Tunio

▪ Expenditure is incurred outside Pakistan on entertainment in connection with


business transaction, or where such expenditure is allocated as head office
expenditure.
▪ Expenditure is incurred in Pakistan on entertainment of foreign customers and
suppliers,
▪ Expenditure is incurred on entertainment of customers and clients at the
person’s business premises,
▪ Expenditure is incurred on entertainment at a meeting of shareholders,
agents, directors or employees, or
▪ Expenditure is incurred on entertainment at the opening of branches.

6. Contribution to Un-recognized/Un-approved Funds

Employers make contribution to various funds established for the benefit of their
employees. The contribution to these funds is in the nature of salary and is,
therefore, deductible expense in computing Income from Business of the employer.

However, the funds in which contribution is made are required to be approved funds.
Hence, any amount of contribution made to: (i) unrecognized provident fund, (ii)
unapproved pension fund, (iii) unapproved superannuation fund, and (iv) unapproved
gratuity fund is not allowed as deduction.

An amount in excess of fifty percent of total contribution made by a person


(employer) to approved gratuity fund, approved pension fund or an approved
superannuation fund shall be disallowed.

7. Contribution to a Fund without Effective Arrangements for Tax Deduction

Any amount of contribution made by an employer to (i) any provident fund, or (ii)
other fund established for the benefit of employees is not deductable unless the
person has made ‘effective arrangements’ to secure that tax is deducted from any
payments made by the fund in respect of which the recipient is chargeable to tax
under the head "Salary".

8. Penalty or Fine

Amount of fine or penalty paid or payable for violation of any law, rule, or regulation.

9. Personal Expenditure

10. Amount Carried to Reserve or Capitalized

Any amount carried forward to a reserve fund (Example: retained earnings) or


capitalized in anyway.

11. Certain Amounts Paid by AOP to Its Members

Profit on debt, brokerage, commission, salary or other remuneration paid by an


association of persons to its members.

12. Payment Otherwise than Banking Channel – Individual & AOP

If the aggregate of a single account head is more than Rs. 250,000/-, the person
making an expenditure under that account head is required to make the payment

89
CAF-02 Tax Practices

through crossed bank cheque, crossed bank draft or crossed pay order or other
crossed banking instruments if the individual expense is more than Rs. 25,000/.

However, online transfer of payment from the business account of the payer to the
business account of payee as well as payments through credit card shall be treated
as transactions through the banking channel, subject to the condition that such
transactions are verifiable from the bank statements of the respective payer and the
payee.

Any expenditure exceeding 25,000 under this account head made otherwise than by
crossed banking instruments such as payment in cash or bearer cheque is
disallowed as expense.

Exception: This rule of making payment through a crossed banking instrument does
not apply to the payments for utility bills, freight charges, travel fare, postage, &
payment of taxes, duties, fees, or any other statutory obligation.

ILLUSTRATION
ABC ltd. incurred following expenses on account of advertisement during the tax year
20X1:
Date Amount Mode of Payment
12.08.20X0 Rs. 35,000/- Cash
06.09.20 X0 80,000/- Crossed Cheque
15.09.20 X0 136,000/- Crossed Cheque
Total 251,000/-

Since the aggregate under a single account head i.e ‘Advertisement’ exceeds
Rs.250,000/-, ABC Ltd. was required to make the payment of Rs. 35,000/- and Rs.
136,000/- through a crossed banking channel. Since this requirement is not fulfilled
in respect of Rs. 35,000/-, hence, it will not be allowed as deduction in computing
income from business.

13. Expenditure by a taxpayer being a company for a transaction, paid or payable under
a single account head which, in aggregate, exceeds rupees two hundred and fifty
thousand, made other than by digital means from business bank account of the
taxpayer notified to the Commissioner under Income Tax Ordinance,2001.

Exception: This rule does not apply for expenditures not exceeding Rupees twenty-
five thousand and expenditures on account of utility bills, freight charges, travel fare,
postage, & payment of taxes, duties, fees, or any other statutory obligation.

14. Payment otherwise than Digital Means – Company

Expenditure by a taxpayer being a company for a transaction paid or payable under a


single account head which, in aggregate, exceeds rupees two hundred and fifty
thousand, made other than by digital means from business bank account of the
taxpayer notified to the Commissioner under section 114A:

This shal not apply in the case of –

a. Expenditures not exceeding Rs. 25,000;&


b. Expenditures on account of –

90
Tariq Hussain Tunio

(i) utility bills,


(ii) freight charges,
(iii) travel fare,
(iv) postage, &
(v) payment of taxes, duties, fees, or any other statutory obligation.

This clause shall be effective from such date as the Board may notify.

15. Payment of Salary

Any salary exceeding Rs. 32,000/- per month paid or payable otherwise than by the
following modes is not allowed as deduction to the employer.
a. by crossed cheque or
b. by direct transfer of funds to the employee’s bank account or
c. through digital means
Expenditure

16. Any expenditure paid or payable of capital nature is not allowed (except depreciation,
intangible & PCE).

17. Any expenditure in respect of sales promotion, advertisement and publicity in excess
of 10% of turnover incurred by pharmaceutical manufacturers.

18. any expenditure on account of utility bill in excess of such limits and in violation of
such conditions as may be prescribed.

19. any expenditure attributable to sales made to persons required to be registered but
not registered under the Sales Tax Act, 1990 by an industrial undertaking computed
according to the following formula, namely:-

(A/B) x C
Where —

A is the total amount of deductions claimed under this Part;


B is the turnover for the tax year; and
C is the total amount of sales exclusive of sales tax and federal excise duty to
persons required to be registered but not registered under the Sales Tax Act, 1990
where sales equal or exceed Rs. 100 million per person:

Provided that disallowance of expenditure under this clause shall not


exceed 10% of total deductions claimed under this Part:

Provided further that the Board may, by notification in the official Gazette,
exempt persons or classes of persons from this clause on the basis of
hardship.

20. Expenditure upto 8%, claimed by a person who, where required, fails to integrate his
business with the FBR through approved fiscal electronic device and software, will be
disallowed.

91
CAF-02 Tax Practices

17. SPECULATION BUSINESS §.19

Taxation of Speculation Business

1. Speculation Business is treated as distinct and separate from normal business


carried on by the person.

2. Deductions allowed under normal business are also allowed under speculation
business. Similarly, deductions not allowed under normal business are also not
allowed under speculation business.

3. Common expenditures incurred for both normal business and speculation business
are to be apportioned [§.67] as if the profits and gains arising from a speculation
business were a separate head of income.

4. Profits and gains arising from the speculation business for a tax year are included in
the person‘s income chargeable to tax under the head Income from Business for that
year; and

5. Loss under speculation business cannot be set off from normal business income of
the person. It is required to be carried forward and set off only against speculation
business income of the person for the next six tax years.

Definition of Speculation Business

6. Speculation Business means:

▪ any business in which a contract for the purchase and sale of any commodity
(including stocks and shares) is made, and
▪ the contract is periodically or ultimately settled otherwise than by the actual
delivery or transfer of the commodity.

Other than

▪ a contract in respect of raw materials is entered into by a person in the course


of a manufacturing to guard against loss through future price fluctuations for
the purpose of fulfilling the person‘s other contracts for the actual delivery of
the goods to be manufactured.
▪ a contract in respect of merchandise is entered into by a person in the
course of mercantile business to guard against loss through future price
fluctuations for the purpose of fulfilling the person‘s other contracts for the
actual delivery of the merchandise to be sold.
▪ contract in respect of stocks and shares is entered into by a dealer or
investor therein to guard against loss in the person‘s holding of stocks and
shares through price fluctuations.
▪ a contract is entered into by a member of a forward market or stock
exchange in the course of any transaction in the nature of jobbing arbitrage to
guard against any loss which may arise in the ordinary course of the person‘s
business as such member.

92
Tariq Hussain Tunio

18. TAX ACCOUNTING §.32

1. A person’s income chargeable to tax is computed in accordance with the method of


accounting regularly employed by such person.

2. A company is required to account for its Income from Business on accrual basis,
whereas other persons (e.g. individuals and AOPs) have an option to account for
their Income from Business on cash or accrual basis.

Person Method of Accounting


Company Accrual Basis
Others (Individual & AOP) Accrual or Cash Basis

FBR’s Power to Prescribe a Method of Accounting

3. The Federal Board of Revenue (FBR) has the authority to prescribe any class of
persons to account for their Income from business on cash or accrual basis.
Explanation for Students
The FBR’s power to prescribe a method of accounting is only in relation to a
class of persons and not to any person individually. Using this authority, the
Board may require any person to follow cash basis or accrual basis of
accounting, so much so that FBR’s power even extends to assigning cash
basis accounting to companies.

Chart# 8► METHOD OF ACCOUNTING

Method of Accounting

Whether FBR has


prescribed any Method

YES NO

Follow the Method Individual &


Prescribed Company
AOP

Cash Accrual Accrual

93
CAF-02 Tax Practices

Changes in the Method of Accounting

4. A method of accounting being followed by a person may be changed in the following


manner:

▪ The person wishing to change his accounting method has to apply in writing
to the Commissioner.

▪ The Commissioner may approve a person’s application for change in


accounting method only if satisfied that the change is necessary to clearly
reflect the person’s Income from Business.
▪ If not satisfied, the CIR may reject the application.

▪ The Commissioner has to issue an order in both cases.

5. If a person’s method of accounting has changed as a result of CIR’s order, the


person is required to make adjustments to items of income, deduction, credit, or any
other items affected by the change so that no item is omitted and no item is taken
into account more than once.

19. CASH BASIS OF ACCOUNTING §.33

1. Under the cash-basis of accounting, a person accounting for income chargeable to


tax under the head ‘Income from Business’:

▪ derives income when it is received by the person, and

▪ incurs expenditure when it is paid by the person.

20. ACCRUAL BASIS OF ACCOUNTING §.34

1. Under the Accrual-basis of accounting, a person accounting for income chargeable to


tax under the head ‘Income from Business’:

▪ derives income when it is due to the person, and


-
▪ incurs expenditure when it is payable by the person.

When is an Amount Due?

2. An amount is due to a person when the person becomes:


▪ entitled to receive it

- even if the time for discharge of the entitlement is postponed or


- the amount is payable by installments.
When is an Amount Payable?

3. An amount is payable by a person when:

▪ all the events that determine liability have occurred and

94
Tariq Hussain Tunio

▪ the amount of the liability can be determined with reasonable accuracy.

Income is chargeable when ‘due’ Deduction is allowed when ‘payable’


Due = when persons becomes entitled Payable = when all the events that
to receive it even if the time for determine liability have occurred and
discharge of entitlement is postponed the amount of liability can be
or the amount is payable by determined with reasonable accuracy.
installments.

Add Back of Trading Liability Unpaid for Three Year

4. In case a person has been allowed a deduction for any expenditure on accrual basis
and the person has not paid the liability (or part of it) within three years of the end of
the tax year in which the deduction was allowed, the unpaid amount of the liability is
chargeable to tax under Income from Business in the first tax year following the end
of the three years.

5. When such unpaid liability is subsequently paid, the person is allowed a deduction for
the amount paid in the tax year in which the payment is made.

6. Where a person has been allowed a deduction in respect of a trading liability and
such person has derived any benefit in respect of such trading liability the value of
such benefit is chargeable to tax under Income from Business for the tax year in
which such benefit is received.

21. STOCK-IN-TRADE §.35

Definition of Stock-in-Trade

1. Stock-in-trade means:

▪ Anything produced/ manufactured/ purchased otherwise acquired for


manufacture, sale or exchange
▪ any materials or supplies to be consumed in the production or manufacturing
process
▪ but does not include stocks or shares

Computation of Cost of Stock-in-Trade

2. For the purposes of determining a person’s Income from Business, the cost of stock-
in-trade disposed of by a person is computed as follows:

(A + B) – C
Where,

A = is the opening value of the person’s stock-in-trade for the year


B = is cost of stock-in-trade acquired by the person in the year
C = is the closing value of stock-in-trade for the year

Opening Value of Stock-in-Trade

3. The opening value of stock-in-trade of a person for a tax year is:

95
CAF-02 Tax Practices

▪ the closing value of the person’s stock-in-trade at the end of the previous
year, or

▪ in case a person has started a new business in the year, the fair market value
of any stock-in-trade acquired prior to the commencement of the business.

4. This fair market value is determined at the time the stock-in-trade is ventured in the
business.

Closing Value of Stock-in-Trade

5. The closing value of a person’s stock-in-trade for a tax year is the lower of:

▪ Cost, or
▪ Net realizable value (NRV) of stock-in-trade at the end of the year.

Methods of Computation of Stock-In-Trade

6. A person accounting for Income from Business on a cash basis may compute the
person’s cost of stock-in- trade on the prime-cost method or absorption-cost
method, and a person accounting for such income on an accrual basis is required to
compute the person’s cost of stock-in-trade on the absorption-cost method.

Method of Accounting Method of Computing Stock-in-Trade


Cash Basis Prime-Cost Method or Absorption-cost Method
Accrual Basis Absorption-cost Method

▪ Prime-cost Method means the generally accepted accounting principle under


which the cost of stock-in-trade is the sum of direct material costs, direct labour
costs, and variable factory overhead costs.
▪ Direct labour costs means labour costs directly related to the manufacture or
production of stock-in-trade.

▪ Direct material cost means the cost of materials that become an integral
part of the stock-in-trade manufactured or produced, or which are consumed in
the manufacturing or production process.

▪ Variable factory overhead cost means those factory overhead costs which
vary directly with changes in volume of stock-in-trade manufactured or produced.

▪ Absorption-cost Method means the generally accepted accounting principle


under which the cost of an item of stock-in-trade is the sum of direct material
costs, direct labour costs, and factory overhead costs.

▪ Factory overhead costs means the total costs of manufacturing or


producing stock-in-trade, other than direct labour and direct material costs.

Cost of Stock-In-Trade in Case Items Are Not Identifiable

7. Where particular items of stock-in-trade are not readily identifiable, a person


may account for that stock on the first-in-first-out method or the average cost
method but, once chosen, a stock valuation method may be changed only with the

96
Tariq Hussain Tunio

written permission of the Commissioner and in accordance with any conditions that
the Commissioner may impose.

8. First-in-first-out method means the generally accepted accounting principle under


which the valuation of stock-in-trade is based on the assumption that stock is sold in
the order of its acquisition.

9. Average-cost method means the generally accepted accounting principle under


which the valuation of stock-in-trade is based on a weighted average cost of units on
hand.

97
CAF-02 Tax Practices

PAST EXAMINATION QUESTIONS


Question # 1
Autumn 2009 Q. 4

(a) State the conditions which a tangible asset should meet to qualify as a depreciable asset.
(04)

(b) During the tax year 2009, Ishaq Enterprise disposed off the following assets:

(i) an immovable property was sold for Rs. 200 million. The cost of immovable
property was Rs. 100 million. Up to tax year 2008, tax depreciation of Rs.10
million had been allowed on the immovable property.

(ii) a plant was exported to Nepal. The export proceeds amounted to Rs. 28 million.
The cost and written down value of the plant was Rs. 25 million and Rs. 18
million respectively.

(iii) three trucks were disposed off for Rs. 2.5 million. They were acquired in tax year
2008. The tax written down value of trucks at the beginning of tax year 2009 was
Rs. 2.4 million. The trucks were being used partly i.e. 60% for business
purposes. The rate of depreciation for tax purposes is 15%.

Required:
Compute the tax gain or loss on disposal of each of the above assets. (06)

Question # 2
Autumn 2010 Q. 4(b)

You are the tax consultant of Ideal Associates who are engaged in the business of
manufacture and sale of electronic goods for the last twenty years. The firm has requested for
your opinion in respect of the following:

(i) Provision forbad debts.


(ii) Payment against a liability which was outstanding since 2006 and had been
added back into the taxable income of the firm in 2009.
(iii) Initial depreciation allowance on a three-year old plant, which has been
imported from China. The remaining useful life of the plant is 7 years.

Required:
Advise the management on the treatment of the above transactions, under the Income
Tax Ordinance, 2001. (07)

Question # 3
Spring 2011 Q. 3

Carrot Ltd (CL) is engaged in the manufacture, import and sale of electronic
appliances for the past twenty years. When reviewing the company's tax provisions,

98
Tariq Hussain Tunio

you noticed the following amounts appearing in the tax calculation for the year ended June
30,20X2.

I. Expenditure of Rs. 450,000 on promotion of a product which is expected to


generate revenue for twelve years.
II. Bad debt in respect of a staff loan, Rs. 25,000.
III. Initial allowance of Rs. 4,000,000 on a used equipment acquired locally from
MSD Limited.
IV. Financial charges amounting to Rs. 100,000 and depreciation amounting to Rs.
300,000 on a vehicle acquired on finance lease from Radish Leasing. Lease
rentals paid during the year amounted to Rs. 400,000.

Required:
Under the provisions of Income Tax Ordinance, 2001 discuss the admissibility of the above
amounts for tax purposes. (15)

Question # 4
Autumn 2012 Q. 6

In the context of Income Tax Ordinance 2001,

(a) state the meaning of "Intangible". (04)


(b) discuss the rules relating to claiming of amortization deduction on intangibles. (07)

Question # 5
Spring 2013 Q. 5

Describe the methods of accounting that may be adopted under the Income Tax Ordinance,
2001 by the following persons deriving income chargeable to tax under the head ‘Income
from Business'.
(i) A company
(ii) Any person other than a company (04)

(b) State the provisions of the Income Tax Ordinance, 2001 relating to the change in method
of accounting for income chargeable to tax under the head ‘Income from Business'. (03)

Question # 6
Autumn 2014 Q. 4

In Income Tax Ordinance, 2001 the term "disposal" has a wider connotation than sale
because it includes exchange, relinquishment, and extinguishment.
List the situations under which an asset owned by a person shall be treated to have been
disposed of. (05)

Question # 7
Autumn 2015 Q. 2

Under the provisions of the Income Tax Ordinance, 2001 what would be the cost of an asset for

99
CAF-02 Tax Practices

the purpose of depreciation deduction in each of the following circumstances?

(a) Mr. Aamir acquired a new machine partly in exchange for an old machine. He paid
freight to bring the old machine to the seller's location and also purchased cooling
equipment which was attached to the new machine for its smooth functioning.
(04)

(b) Mr. Saulat acquired production-machinery by utilizing a loan repayable in euro. The loan
is expressed in rupees and is repayable in two years', time. Mr. Saulat also received
20% subsidy on such machinery from the Provincial Government.
(04)

(c) On 1 July 2015 Mr. Talha started using his personal computer for business purposes.
He also had to upgrade the operating system to comply with his business needs.
(02)

(d) Mr. Rahi constructed a furnace for his factory in Korangi Industrial Area. (02)

Question # 8
Spring 2016 Q. 2

Akram has recently established an advertising agency in the name and style of Azad
Advertising. For introducing his business to both international and local clients, he has
allocated considerable chunk of his marketing budget to entertainment expenditures. Under
the Income Tax Ordinance, 2001 and Rules made thereunder, advise Akram about the
prescribed limits/conditions for the deduction of entertainment expenditure.(07)

Question # 9
Spring 2017 Q. 2(a)

(a) Explain the term 'disposal of assets' as referred to in the Income Tax Ordinance, 2001.
(05)

Question # 10
Autumn 2017 Q.3

Under the provisions of the Income Tax Ordinance, 2001 compute taxable income or
loss, under the correct head of income for tax year 2017, in each of the following
cases:

(a) Sarwar Enterprises sold an immovable property for Rs. 50 million. The cost
of the immovable property was Rs. 30 million. Tax depreciation of Rs. 6 million had
been allowed on the immovable property up to the tax year 2016. (2.5)

(b) Shams Industries Limited (SIL) sold and exported one of its plants to a
Nigerian Company. The sale proceeds received id SIL's account amounted to Rs.
25 million. The cost and tax written down value of the plant was Rs. 20 million and

100
Tariq Hussain Tunio

Rs. 7 million respectively. (2.5)

Question # 11
Autumn 2017 Q. 5 & Autumn 2011 Q.3(a)

Under the Income Tax Ordinance, 2001 certain persons are required to pay minimum tax
amounting to 1.25% of their turnover from all sources.

(a) Explain the term 'Turnover' for the purpose of determining the minimum tax. (05)
(b) List the persons who are required to pay minimum tax. (03)
(c) Discuss the provisions relating to carry forward of minimum tax paid to the subsequent
years. (02)

Question # 12
Autumn 2018 Q.3

(a) Hirani & Company (HC), a resident AOP, is engaged in the manufacturing of various
consumer products and is assessed under normal tax regime. During the year ended 30
June 20X8, HC's sales was Rs. 140,000,000. It includes sales tax of Rs. 10,000,000 and
excise duty of Rs. 5,000,000. The taxable income for the year is Rs. 6,170,000.

Compute HC's tax liability for tax year 20X8, under the provisions of the Income Tax
Ordinance, 2001. (03)

(b) The accounting profit before tax of Bashir Associates (BA) for the year ended 30 June
20X8 is Rs. 1,200,000.
Last year, BA had written off balances outstanding from two of its debtors namely Pulse
International (PI) and Hussain Global (HG) which were partly allowed by the tax authorities.
Details are as follows:
PI HG
Rupees
Amounts written off 1,150,000 925,000
Allowed by tax authorities 825,000 240,000

During the current tax year, BA received Rs. 652,000 from PI and Rs. 346,000 from HG, in
full settlement of their debts.

In the light of the Income Tax Ordinance, 2001 compute BA's taxable income for the tax year
20X8. (05)

Question # 13
Spring 2019 Q. 3(b)

Following transactions pertain to Salam Limited (SL) which took place during the tax year
20X9:

(i) A machine costing Rs. 1,800,000, being used in SL's Karachi factory was transferred

101
CAF-02 Tax Practices

to its subsidiary in Ghana. The fair market value and tax written down value of the
machine on the date of transfer were Rs. 2,500,000 and Rs. 600,000 respectively.
(02)

(ii) On 1 January 20X9, SL entered into a forward contract for the purchase of raw
materials to be used in its business to guard against loss through price fluctuations. On
the date of maturity of the forward contract, SL did not take the delivery of the raw
materials but the contract was settled by making a payment of Rs. 500,000.
(03)

Required:
Explain the taxability of the above transactions.

Question # 14
Autumn 2019 Q. 5

(a) Identify any three situations in which the fair market value of the assets shall be
treated to be the cost of the asset. (03)

(b) During the tax year 20X9, Salman Shahid sold the following assets:

(i) A vehicle used by manager-in-charge of his garment factory for Rs. 7.8 million. The
vehicle was purchased for Rs. 8.1 million in tax year 20X6. (03)

Required:
Under the provisions of the Income Tax Ordinance, 2001 compute under the appropriate
head of income, the amount to be included in the taxable income of Salman Shahid for the
tax year 20X9.

Question # 15
Spring 2020 Q. 3(a)

Under the provisions of the Income Tax Ordinance, 2001 and Rules made
thereunder, discuss:
(a) the prescribed limits/conditions for the deduction of entertainment
expenditure.(06)

Question # 16
Spring 2021 Q. 2(b)

Gillani and Company (GC), a sole proprietor, is dealing in various consumer products
in Pakistan. During the year ended 30 June 20X2, GC’s taxable income for the year
was Rs. 1.6 million.

Required:
Under the provisions of the Income Tax Ordinance, 2001 compute the amount of net
income tax payable by GC and amount of income tax to' be carried forward, if any, for
the tax year 20X2, in each of the following situations:

102
Tariq Hussain Tunio

(i) GC's sales were Rs. 120,500,000 inclusive of sales tax.


(ii) GC’s sales were Rs. 110,000,000 inclusive of sales tax. (05)

Question # 17
Autumn 2021 Q. 3(a)(ii)

State the provisions of the Income Tax Ordinance, 2001 relating to each of the following:

Change in the method of accounting for income chargeable to tax under the head 'income
from business'. (03)

Question # 18
Spring 2022 Q.2
(a) Under the provisions of the Income Tax Ordinance, 2001 discuss the tax
implication/treatment in each of the following independent matters:
(i) Purchase of immovable property in cash. (03)

(b) For the purpose of this part of the question, assume that the date today
is 31 August 2022. During the year ended 30 June 2022, Faster & Co.
(FC) started a new project. Following information is available:

▪ Incurred Rs. 5 million on feasibility study of the project.


▪ Obtained a 3% loan of AED 2 million from a UAE bank on
1 January 2022 for the purchase of plant and machinery.
The interest is payable annually and principal amount is
repayable at the end of third year.
▪ Installed the plant and machinery at a cost of Rs. 150
million on 14 March 2022.

The exchange rates of 1 AED to PKR on different dates are as follows:

01-Jan-2022 30-Jun-2022 Average between


1-Jan-2022 to 30-Jun-
2022
Rs. 50 Rs. 55 Rs. 53

Required:
Compute the amount of allowable deduction in determining the taxable income of
FC-for tax year 2022. (04)

Question # 19
Spring 2020 Q.4(b)

Respond to the following situation, under the provisions of the Income Tax Ordinance,2001:

103
CAF-02 Tax Practices

On 1 July 2014, Ahmed purchased two sculptures for Rs. 410,000 and Rs. 475,000
respectively. On 30 November 2019, during the shifting of his house, he lost both the
sculptures. On 15 January 2020, he received insurance claim of Rs. 940,000 in a single
transaction against the loss of two sculptures. The fair market value of both the sculptures at
the time of loss was estimated at Rs. 360,000 and Rs. 540,000 respectively. Compute
Ahmed’s taxable income or loss for the above transaction.(04)

Question # 20
Autumn 2019 Q.5(a)

Identify any three situation in which the fair market value of the asset shall be treated to be
the cost of the asset. (06)

Question # 21
Autumn 2019 Q.3 ( c ), (d)

a) Sikander has revalued his factory building in accordance with International Financial
Reporting Standards and consequently charged depreciation on the revalued
amount. Explain the tax implication of the revalution. (02)

b) Shahbaz has acquired machinery for his new factory against a loan repayable in
USD. Discuss what be the cost of machinery for the purpose of depreciation
deduction. (02)

Question # 22
Spring 2013 Q.5

a) Describe the methods of accounting that may be adopted under the ITO,2001 by the
following persons driving income chargeable to tax under the head ‘income from
business’.
(i) A company
(ii) Any person other than a company (03)

b) State the provisions of the Income Tax Ordinance,2001 relating to the change
in method of accounting for income chargeable to tax under the head income from
business. (03)

Question # 23
Autumn 2012 Q. 6

In the context of Income Tax Ordinance,2001,


c. State the meaning of Intangible. (04)
d. Discuss the rules relating to claiming of amortization deduction on intangibles.
(07)

Question # 24
Spring 2007 Q. 2 (b)

Discuss the provisions of the Income Tax Ordinance relating to the computation of
opening and closing stock. (04)

104
Tariq Hussain Tunio

Question # 25
Autumn 2007 Q.6 (b)

A person who places an eligible depreciable asset into service in Pakistan for the first time in
a tax year shall be allowed initial depreciation allowance. List down the assets which do not
come under the purview of "eligible depreciable assets" for the purpose of initial allowance.
(04)

Question # 26
Autumn 2007 Q.5(b)

Briefly discuss the allowability of the following against business income chargeable to tax:

(i) Payment of utility bills in cash amounting to Rs. 15,000.


(ii) Penalty of Rs.5,000 paid for late submission of quarterly statement under the Income
Tax Ordinance, 2001.
(iii) Depreciation on leased assets amounting to Rs. 150,000. (05)

Question # 27
Spring 2008 Q.4 (a)

Discuss the taxability of the following under the Income Tax Ordinance, 2001:
(i) Bad debts (05)
(ii) Non-adjustable rent (04)
(iii) Speculation business (04)

Question # 28
Autumn 2008 Q.4

Mr. Sajid is a sole proprietor involved in the distribution of fans manufactured by a Pakistani
Company. He is in the process of computing his business income for tax year 2008 but is not
clear about the tax treatment of the following items:

(i) Commission of Rs.20,000 was paid to his employee but no tax was deducted by
him.

(ii) In his books of account, an expense of Rs.50,000 has been charged on account
of various household expenses.

(iii) During the year, he purchased a car at a cost of Rs. 1,200,000 which has been
used for personal as well as business purposes. Mr. Sajid wants to claim
depreciation (including initial allowance) on the full amount of cost.

(iv) Computer software was purchased on January 1, 2008 at a cost of Rs.900,000.


The software is likely to used for twelve years.

(v) His business assets worth Rs.500,000 were destroyed by the earthquake in
October 2005. He claimed the loss in his return for tax year 2006 but an amount
of Rs. 150,000 was disallowed by the taxation officer. In tax year 2008, the
Government gave him a compensation of Rs.400,000 on this account.

105
CAF-02 Tax Practices

(vi) Professional tax of Rs. 100,000 was paid to the Government of Punjab. Such tax
is to be paid by every person who is engaged in the trading business in the
territory of Punjab. He considers it inadmissible as it is a tax paid to a provincial
government.

Explain the correct tax treatment of the above items under the Income Tax Ordinance,2001.
(12)

Question # 29
Autumn 2009 Q.3

a. State the conditions which a tangible asset should meet to quality as a depreciable
asset.
b. During the tax year 20x9, Ishaq Enterprise disposed of the following assets:
(i) An immovable property was sold for Rs. 200 million. The cost of
immovable property was Rs. 100 million. Till last year, tax depreciation of
Rs. 10 million had been allowed on the immovable property.
(ii) A plant was exported to Nepal. The export proceeds amounted to Rs. 28
million. The cost and written down value of the plant was Rs. 25 million
and Rs. 18 million respectively.
Required:
Compute the tax gain or loss on disposal of each of the above assets.

Question # 30
Autumn 2004 Q.3

a. List down the assets on which "Initial allowance" cannot be claimed? (04)
b. What are the prescribed rates of normal depreciation on the following assets as per
the Third Schedule to the Income Tax Ordinance, 2001? (04)
(i) Factory building
(ii) Residential quarter for labour
(iii) Furniture
(iv) Plant and machinery
(v) Computer and hardware
(vi) Technical books
(vii) New ships
(viii) Motor vehicle

Question # 31
Autumn 2004 Q. 3

Describe the expenses which are allowable as a deduction on account of employees


training and facilities? (04)

Question # 32
Autumn 2004 Q.4

Discuss the conditions required to be fulfilled for claiming a deduction on account of


"bad-debts? (03)

Question # 33
Spring 2005 Q.3

106
Tariq Hussain Tunio

Describe any five types of expenses that are not allowed to be deducted under the head
"income from business". (05)

Question # 34
Spring 2005 Q.4(a)

Describe the assets that are not eligible for the purpose of claiming initial depreciation
allowance. (04)

Question # 35
Spring 2006 Q.3

a. Explain with reasons, as to whether or not the following expenses are admissible
business expenditures:
(i) Penalty paid by a banking company on contravention of State Bank of
Pakistan's regulations. (01)
(ii) Freight charges to forwarding agent amounting to Rs.60,000 paid in
cash.(01)
(iii) Payment of salary to an employee from which tax was not deducted by
the employer. However the employee paid the tax himself. (01)
b. What is the basis of stock-in-trade computation under the Income Tax Ordinance
when the taxpayer follows the cash basis of accounting? (02)
c. Explain the provisions of section 29 with regard to the recovery of bad debts in
subsequent years. (05)

Question # 36
Spring 2006 Q.2(a)

Define the following with reference to the Income Tax Ordinance, 2001:
• Depreciable asset (05)

Question # 37
Autumn 2006 Q.4(a)

A company may account for income chargeable to tax under the head ‘income from
business’ on cash basis or on accrual basis.

Briefly discuss the rules relating to accrual of income and expenditure as explained in the
Income Tax Ordinance,2001.(04)

Question # 38
Autumn 2006 Q.3(b)

One of your clients, Japan and Company, a partnership having three partners, has sent you
its latest financial statements for the year ended June 30,2006. Following items are
appearing under the head ‘other income’.

(i) Accounting profit on disposal of fixed assets.

(ii) Reversal of provision for doubtfull debts pertaining to the year ended June 30,2004.

107
CAF-02 Tax Practices

(iii) Dividend received from a listed company.

You are required to explain with reasons as to how the above items will be treated in the
computation of taxable income. (03)

Question # 39
Spring 2002 Q.5

Explain whether the following are admissible as business expenditure under the Income Tax
Ordinance:

a. Repayment of principal amount of lease rentals of plant & machinery.


b. Sales tax paid on the purchase raw materials to be used in the production of
exempt supply.
c. Dividend
d. Provision in respect of doubtful debts
e. Penalty levied under the Income Tax Ordinance, 2001 for failure to file statement.
(10)

Question # 40
Autumn 2002 Q.5

Dreamland (Pvt.) Ltd. has requested you to advice as regards the important aspects of law
for disallowance of expenses incurred in cash u/s 21 Please write an advisory letter in this
regard explaining the law with suitable examples. (10)

Question # 41
Spring 2003 Q.6

a. What method of accounting is required to be employed by a company deriving


income from business? (02)
b. Briefly state the provisions relating to the change in the method of accounting of
income from business. (03)

Question # 42
Spring 2003 Q.4(a)

1. Mercury & Co. has provided you the following data:

Fair value of leased asset Rs 225,000


Interest rate 20.5%
Security Deposit paid 10% of fair value
Depreciation of leased asset 33% per annum
Term of lease 3 years
Yearly rental in arrears Rs.96,890

You are required to compute amount available for deduction from the taxable income
of Mercury & Co for each year, Please show proper working. (05)

2.Sun & Moon has recently registered as partnership. They have incurred the
following expenditure.

108
Tariq Hussain Tunio

• Fee paid to consultants for preparation of registration deed Rs 50,000


• Preparation of feasibility report Rs. 100,000
• Purchase of office equipment Rs. 150,000
• Purchase of machinery Rs. 1,000,000
• Trial run cost Rs.200,000
• Installation cost Rs.50,000
You are required to explain the tax treatment by computing the amount allowable as
deduction in accordance with file provisions of Income Tax Ordinance, 2001. (05)

Question # 43
Spring 2004 Q.4

a. Briefly explain when the expenditure is considered as incurred with reference to


accrual basis of accounting defined in the Income Tax Ordinance, 2001. (03)
b. Please mention the costing method(s) and stock valuation method(s) to be applied by
a person following accrual basis of accounting to account for income chargeable to
tax under the head 'income from business. (03)

Question # 44
Autumn 1998 Q.4

Mr. XYZ, Deputy Managing Director of a public limited company attended a seminar on the
subject of taxation.
One of the speaker of the seminar in his speech said that it is necessary to determine
a. tax status of the taxpayer to calculate the correct taxable income and tax liability
b. he also said that expenditure incurred by a taxpayer may be of two types i.e,
capital expenditure or revenue expenditure and
c. he further said that the income tax Statute while taxing the income also provides
relief from taxes as well under the provisions section 53 of the Income Tax
ordinance, 2001.

The Deputy Managing Director has asked you as a Chief Accountant to explain to him in a
write-up:

(i) How does tax liability differ according to the tax status of the person? (06)

(ii) Does the tax treatment differ in respect of capital and revenue expenditure? If so
explain with examples (4)

(iii) How and to what exemptions from tax are provided under the provisions of section 53
of the income Tax Ordinance, 2001. (4)

Question # 45
Spring 1999 Q.3

Mr. B is Managing Director (MD) of a XYZ public limited company. He is master of


science in petroleum engineering with a little background of accounting and tax. He has
been informed that the assessing officer may question the method of accounting of the XYZ
public limited company in the context of determination of its total income for the purpose of
charge of tax u's 4. Whilst section 20 of the Income Tax Ordinance, 2001 specifies

109
CAF-02 Tax Practices

admissible deductions, section 21 of the Ordinance stipulate certain deductions which are
not admissible under the law.

MD is not clear about certain things stated above and has asked you, as chief accountant
to explain him the following in a write up:

a. What types of income are chargeable under the head income from business u/s 18?
(03)

b. What are the six inadmissible deductions u/s 21 of the Ordinance for computing
income from business? (3)

c. How many types of method of accounting exist and under what circumstances taxation
officer may not accept the taxpayer's method of accounting (03)

Question # 46
Autumn 1999 Q.2

What are the conditions in the law for admissibility of the following expenditure:

(i) lease payments (02)


(ii) interest (02)
(iii) salary and wages (02)
(iv) gratuity and provident fund (02)
(v) perquisites (02)

Question # 47
Spring 1999 Q.5(b)

Unpaid trading liability attracts income tax. Illustrate the provisions of income tax law
covering this aspect. (06)

Question # 48
Spring 2001 Q.3

Whether or not the following are admissible under the Income Tax Ordinance, 2001:
a. Borrowing cost on loans obtained to purchase fixed assets (01)
b. Bonus shares issue expenses (01)
c. Preliminary expenses (01)
d. Bad debts (01)

Question # 49
Autumn 2001 Q.2

Discuss briefly the legal position with respect to the admissibility or otherwise of the
following as business expenditure under the Income Tax Ordinance, 2001:

a. Amount paid as income tax (02)


b. Capital expenditure incurred on scientific research in Pakistan (02)
c. Share of profit paid to a bank under a scheme of musharika (02)
d. Interest paid by a firm to partner of the firm. (02)

110
Tariq Hussain Tunio

e. Salary paid otherwise than through a crossed cheque etc.(02)

111
CAF-02 Tax Practices

112
Tariq Hussain Tunio

Chapter

06

CAPITAL GAINS
CHAPTER SYNOPSIS
TOPIC SECTION
Structure of taxation of capital gains General
Capital gains on disposal of capital
37
asset
Capital gain on disposal of immovable
37
property
Deduction of losses in computing
38
capital gains
Capital gain on sale of securities 37A
Taxation on Deemed Income 7E
ICAP Past Papers Questions

113
CAF-02 Tax Practices

1. STRUCTURE OF TAXATION OF CAPITAL GAINS

1. The head of income ‘Capital Gains’ deals with taxation of capital gain arising on
disposal of (i) capital asset, and (ii) securities.

COMPUTATION OF CAPITAL GAINS


(A) Capital Gain on Capital Asset Other than Immovable Property
Cost of Capital Asset xx
L: Consideration Received on disposal of Capital Asset (xx)
Gain Arising on Disposal of Capital Asset xx
(B) Capital Gain on Capital Asset Being Immovable Property
Cost of Immovable Property xx
L: Consideration Received on Disposal Immovable Property (xx)
Gain Arising on Disposal of Immovable Property xx
(C) Capital Gain on Securities
Cost of Securities xx
L: Consideration Received on Disposal of Securities (xx)
Gain Arising on Disposal of Securities xx
CAPITAL GAIN xx

2. CAPITAL GAINS ON DISPOSAL OF CAPITAL ASSET OTHER THAN


IMMOVABLE PROPERTY

Amounts Chargeable under ‘Capital Gains’.

1. A gain arising on disposal of a capital asset by a person in a tax year is chargeable to


tax in that year under the head “Capital Gains”, except when it is exempt under the
Income Tax Ordinance, 2001.

Definition of Capital Asset

2. Capital Asset

▪ means property of any kind


o held by a person,
o whether or not connected with business

▪ but does not include

o any stock-in-trade, consumable stores or raw materials held for


the purpose of business

o any property depreciable assets and Intangibles

o any movable property (excluding capital asset specified in


§38(5)) held for personal use by the person or any member of the
person’s family dependent on the person.

3. Following movable properties held for personal use are treated as capital
assets as per §. 38(5)

114
Tariq Hussain Tunio

▪ a painting, sculpture, drawing or other work of art


▪ jewellery
▪ a rare manuscript, folio or book
▪ a postage stamp or first day cover
▪ a coin or medallion
▪ an antique

Computation of Capital Gains

4. The gain arising on the disposal of a capital asset by a person is computed in


accordance with the following formula:

Consideration Received on Disposal of Capital Asset


L: Cost of the Capital Asset
Capital Gain on Disposal of Capital Asset

Amounts Not to be Included in the Cost of Capital Asset


5. For the purposes of determining cost of a capital asset, no amount is included in the
cost of a capital asset for any expenditure incurred by a person:

▪ that is or may be deducted under another head of income, or


▪ that is not allowed as deduction u/s 21.

3. CAPITAL GAIN ON DISPOSAL OF IMMOVABLE PROPERTY

1. Immovable property such as land or building is capital asset except when


depreciable asset, situated in Pakistan.

2. Gain arising on disposal of immovable property situated in Pakistan is chargeable to


tax at following rates.

RATE OF TAX
S. HOLDING PERIOD Open Constructed Flats
No
Plots Property
1 Where holding period of Immovable 15% 15% 15%
Property does not exceed one year
2 Where holding period of Immovable 12.5% 10% 7.5%
Property exceeds one year but does not
exceed two year
3 Where holding period of Immovable 10% 7.5% 0
Property exceeds two but does not exceed
three years
4 Where holding period of Immovable 7.5% 5% -
Property exceeds three years but does not
exceed three year
5 Where holding period of Immovable 5% 0 -
Property exceeds four years but does not
exceed five years
6 Where holding period of Immovable 2.5% - -
Property exceeds five year but does not
exceed six year

115
CAF-02 Tax Practices

7 Where holding period of Immovable 0% - -


Property exceeds six years

4. DEDUCTION OF LOSSES IN COMPUTING CAPITAL GAINS

1. In computing capital gain on disposal of a capital asset of a person, a deduction is


allowed for any loss on the disposal of a capital asset.

Explanation for Students


If a person derives gain on disposal of one capital asset and sustains a loss
on the disposal of another, the loss (referred to as ‘capital loss’) is deducted
from the capital gain.

How to Compute Capital Loss

2. The loss arising on disposal of a capital asset is computed in accordance with the
following formula:

C o s t o f t h e C a p i t a l A s s e t
L:Consideration Received on Disposal of Capital Asset
Capital Loss on Disposal of Capital Asset

Amounts Not to be Included in the Cost

3. For the purposes of determining cost of a capital asset for determination of capital
loss, no amount is included in the cost of a capital asset for any expenditure incurred
by a person:

▪ that is or may be deducted under another head of income, or


▪ that is not allowed as deduction u/s 21.

Loss Not Deductible Where Gain is not Chargeable

4. In case gain on disposal of a capital asset is not chargeable under Capital


Gains, loss on the disposal of such capital asset is not deductible.

Non-Recognition of Losses on Disposal of Declared Capital Assets

5. No loss is recognized on the disposal of the following capital assets:

▪ painting, sculpture, drawing or other work of art


▪ jewellery
▪ a rare manuscript, folio or book
▪ a postage stamp or first day cover
▪ a coin or medallion
▪ an antique

116
Tariq Hussain Tunio

5. CAPITAL GAIN ON DISPOSAL OF SECURITEIS

1. The capital gain arising on disposal of securities is chargeable to tax subject to


following conditions:

▪ Securities are held for a period of less than a year. (Gain arising on securities
held for a period of more than one year is not chargeable to tax.)

▪ Capital gain arises on or after the first day of July 2010

Explanation for Students


Capital gain on disposal of securities is treated as ‘separate block of income’.
Hence, gain on sale of securities is computed separately from gain on
disposal of capital assets. It is included under the head but is subtracted from
taxable income for application of separate/ special tax rates.

Computation of Capital Gains

2. The gain arising on the disposal of a security is computed in accordance with the
following formula:

Consideration Received on Disposal of Security


L: Cost of the Capital Asset
Capital Gain on Disposal of Secur ity

Definition of Security

3. Security means:

▪ share of a public company,


▪ voucher of Pakistan Telecommunication Corporation,
▪ Modaraba Certificate,
▪ an instrument of redeemable capital
▪ Debt securities,
▪ Unit of exchange traded fund,
▪ derivative products.

Definition of Debt Securities

Debt Securities

• Means

a. Corporate Debt Securities such as Term Finance Certificates (TFCs),


Sukuk Certificates (Sharia Compliant Bonds), Registered Bonds,
Commercial Papers, Participation Term Certificates (PTCs) and all kinds of
debt instruments issued by any Pakistani or foreign company or
corporation registered in Pakistan; and

b. Government Debt Securities such as Treasury Bills (T-bills), Federal


Investment Bonds (FIBs), Pakistan Investment Bonds (PIBs), Foreign
Currency Bonds, Government Papers, Municipal Bonds, Infrastructure

117
CAF-02 Tax Practices

Bonds and all kinds of debt instruments issued by Federal Government,


Provincial Governments, Local Authorities and other statutory bodies.

Derivative products include future commodity contracts entered into by the member of
Pakistan Mercantile Exchange whether or not settled by physical delivery.

Holding Period

4. The holding period of a security is to be reckoned from the original date of acquisition
(whether before, on or after 30.06.2010) to the date of its disposal.

Setting off and Carry Forward of Loss on Securities

5. Where a person sustains a loss on disposal of securities in a tax year, the loss is set
off only against the gain of the person from any other securities chargeable to tax
under this head.

Loss sustained on disposal of securities in tax year 2019 and onwards that has not
been set off against the gain of the person from disposal of securities chargeable to
tax under this section shall be carried forward to the following tax year and set off
only against the gain of the person from disposal of securities chargeable to tax, but
no such loss shall be carried forward to more than three tax years immediately
succeeding the tax year for which the loss was first computed.

Tax Rates
6. Capital gain arising on disposal of securities is chargeable to tax at the following tax
rates:

Rate of Tax for


S# HOLDING PERIOD Tax Year 2023
and onwards
1 Where holding period of a security does 15%
not exceed one year.

2 Where holding period of a security 12.5%


exceeds one year but does not exceed
two years.
3 Where holding period of a security 10%
exceeds two years but does not exceed
three years.
4 Where holding period of a security 7.5%
exceeds three years but does not
exceed four years.
5 Where holding period of a security 5%
exceeds four years but does not exceed
five years.
6 Where holding period of a security 2.5%
exceeds five years but does not exceed
six years.
7 Where holding period of a security 0%
exceeds six years.
8 Future commodity contracts entered 5%
into by members of Pakistan Mercantile
Exchange

118
Tariq Hussain Tunio

6. TAX ON DEEMED INCOME 7E

1. A resident person owning capital assets in Pakistan will be taxed on deemed


income arising from capital assets for tax year 2022 and onwards.

2. An exclusive definition of ‘capital asset’ has been provided, which effectively means
that such tax is leviable only in respect of ‘immovable property’ (e.g. house, any
building, manufacturing plant etc.) situated in Pakistan owned by resident persons.

3. Deemed income shall be computed as 5% of the Fair Market Value (as determined
by the FBR under section 68 i.e. FBR Value or DC Rate) of capital assets situated
in Pakistan held on the last day of the tax year.

4. The rate of tax on such income is prescribed as 20%. This translates into an
effective tax at 1% of Fair Market Value of capital assets.

5. For the purposes of such tax; however, following immovable properties shall stand
excluded from the scope of such tax:

(i) One immovable property owned by the resident person;

(ii) Any property from which income is chargeable to tax under the
Ordinance and tax leviable is paid thereon (for example property
subject to rental income):

(iii) Immovable property in the first tax year of acquisition where


withholding tax under section 236K has been paid at the time of
purchase;

(iv) Where the fair market value of the capital assets in aggregate
excluding the capital assets mentioned in clauses (i) to (iii) above and
(v) to (ix) below does not exceed Rs 25 million;

Provided that the exclusions mentioned at clauses (i) to (iv) above


shall not apply in case of a person not appearing in the active
taxpayers' list, other than persons who are not required to file return
covered in rule 2 of the Tenth Schedule;

(v) Self-owned business premises from where the business is carried out
by the persons appearing on the active taxpayers’ list at any time
during the year

(vi) self-owned agriculture land where agriculture activity is carried out by


person excluding farmhouse and land annexed thereto;

Note: Farmhouse means a house constructed on a total minimum


area of 2000 square yards with a minimum covered area of 5000
square feet used as single dwelling unit.

(vii) immovable property allotted to:

a. a shaheed or dependents of a shaheed belonging to Pakistan


Armed Forces.

119
CAF-02 Tax Practices

b. a person or dependents of the person who dies while in the


service of Pakistan armed forces or Federal or provincial
government.

c. a war wounded person while in service of Pakistan armed


forces or Federal or provincial government or

d. an ex-serviceman and serving personnel of armed forces or


ex-employees or serving personnel of Federal and provincial
governments, being original allottees of the capital asset duly
certified by the allotment authority;

(viii) Immovable property owned by a provincial government or a local


government; or

(ix) Immovable property owned by a local authority, a development


authority, builders and developers for land development and
construction.

6. If the tax liability under section 7E is not discharged, then the registrar or the
person registering the transfer is required not to register the transfer of the subject
property.

120
Tariq Hussain Tunio

PAST EXAMINATION QUESTIONS


Question # 1
Spring 2010 Q.6

(a) Explain the term Capital Assets as referred to In the Income Tax Ordinance, 2001.
(05)

(b) Mr. Shahbaz, a resident individual earned Rs. 700,000 from the sale of assets as
shown below:

Purchase Purchase Sale Sale Gain/loss


(Rs.)
Date Price Date Price
Rupees Rupees
Shares of a listed 10/12/21 350,000 30/06/23 200,000 (150,000)
company
Shares of an unlisted 15/07/21 500,000 30/11/22 900,000 400,000
company
Jewellery 15/05/21 750,000 20/12/22 1,400,000 650,000
Sculpture 01/07/21 400,000 31/01/23 300,000 (100,000)
Shares of a private 01/01/23 1,300,000 15/02/23 1,200,000 (100,000)
limited company

Required:

Discuss the treatment and the implications of each of the above transactions under the
Income Tax Ordinance, 2001. Give brief reasons to support your conclusion. (05)

Question # 2
Autumn 2011 Q.5

Mr. Feroz has been the CEO of Aziz Foods Pakistan Limited (AFPL) for several years. He
was given 2000 shares on 1 June 2009 by Aziz AG, Germany (the parent company of AFPL)
at a price of €2.5 per share. The market price on that date was €8.2 per share. The shares
were transferable on completion of one year of service, from the date of issue of shares.

The market price of the shares as on 1 June 2010 was €12.5 per share. On 10 April 2011,
Mr. Feroz sold all shares at €13 per share. He paid a commission of €50 to the brokerage
house.

The relevant exchange rates are as follows:

1 June 2009 €1 = Rs. 118.10


1 June 2010 €1 = Rs. 121.40
10 April 2011 €l = Rs. 123.90
Required:

Compute the amount to be included in the taxable income of Mr. Feroz for tax years 2009,
2010 and 2011 and specify the head of income under which the income would be classified.
(07)

121
CAF-02 Tax Practices

Question # 3
Autumn 2012 Q.5(a)

In May 2023, Ms. Hameeda sold certain personal assets at the following prices:

Rupees
Plot in DHA Karachi 10,000,000
Paintings 2,000,000
Jewellery 5,000,000

Additional information:

Plot in DHA Karachi was inherited by her from her father in May 2012. It was purchased by
her father for Rs. 4,000,000 and market value at the time of inheritance was Rs. 5,000,000.

Paintings were inherited from her mother in July 2017. These paintings were purchased by
her mother for Rs. 2.350,000 and market value at the time of inheritance was Rs.4,000,000.

Jewellery costing Rs. 3,000,000 was purchased and gifted to her by her husband in March
2015.

Required:

Discuss the taxability of Ms. Hameeda in respect of the above gains/ losses on sale of
assets in the context of Income Tax Ordinance, 2001. (06)

Question # 4
Autumn 2013 Q.3

(b) Explain the term ‘capital asset’ as referred to in the Income Tax Ordinance,2001.
(04)

(c) Gulzar is a Pakistani resident and operates various businesses. He disposed of


the following assets during the tax year 2023:

Plot in DHA Karachi was inherited by her from her father in May 2012. It was
purchased by her father for Rs. 4,000,000 and market value at the time of inheritance
was Rs. 5,000,000. Paintings were inherited from her mother in July 2017. These
paintings were purchased by her mother for Rs. 2.350,000 and market value at the
time of inheritance was Rs.4,000,000.

Jewellery costing Rs. 3,000,000 was purchased and gifted to her by her husband in
March 2015.

Required:

Discuss the taxability of Ms. Hameeda in respect of the above gains/ losses on sale of
assets in the context of Income Tax Ordinance, 2001. (06)

Question # 5
Autumn 2014 Q.3

Zaman is working as the Chief Executive Officer in Yasir Limited (YL). Following are
the details of sale and purchase relating to his capital assets during the tax year 2023.

122
Tariq Hussain Tunio

1. Under an employee share scheme. 25,000 shares of YL were allotted to Zaman, on 1


December 2020forfc 25 each. According to the scheme, he was not allowed to
sell/transfer the shares before completion of years from the date of transfer. The face
value of each share is Rs. 10 per share. Fair market value of the shares was as
follows:

• Rs. 40 per share on 1 December 2020


• Rs, 48 per share on 30 June 2021
• Rs. 55 per share on 30 November 2022
• Rs. 61 per share on 30 June 2023

2. He sold 24,000 shares of HQ (Pvt.) Limited on 30 June 2023 for Rs. 200 per share.
He had aquired these shares as follows:

• 18,000 shares were purchased at Rs. 55 per share on 25 June 2022.


• 6,000 shares were allocated as bonus shares on 28 February 2023.

3. A gain of Rs. 300,000 was realized on the sale of shares of Zeeshan Industries
Limited (ZIL), a public listed company, in June 2023. The shares were acquired on 31
May 2022.

4. Zaman sold a painting to his brother on 23 March 2023 for Rs. 1,800,000. Zaman
had purchased this painitng for his residence, in an auction for Rs. 2,000,000 on 10
July 2020.

5. He sold his old furniture to furqan for Rs. 285,000 on 25 June 2023. The furniture
was purchased in 2021 for Rs. 250,000.

Required:

Compute the amount to be included in the taxable income of Zaman for the tax year 2023
and specify the head of income under which the income would be classified. (10)

Question # 6
Autumn 2015 Q.4

a) What do you understand by the terms “security” and “derivative products” as


provided in the Income Tax Ordinance,2001 and Rules made thereunder?

b) Under the provisions of the Income Tax Ordinance,2001 compute taxable gain or
loss, under the correct head of income, in each of the following cases. Also identify,
giving reasons, whether the company is a public or private company for tax
purposes:

(i) Ashiq has 5,000 shares in Rumi (Pvt) Limited (RPL). 52% of the shares of RPL are
held by Delta Pic. which is owned by the British Government Ashiq inherited these
shares from his father on 1 January 2014. His father had purchased these shares on
31 May 2012 at a price of Rs. 250 per share. The market value of these shares at the
time of inheritance was Rs. 300 per share.

On 30 June 2015 Ashiq sold 2,500 shares in RPL at a price of Rs. 325 per share
when the break-up value of RPL was Rs. 350 per share. (04)

123
CAF-02 Tax Practices

(ii) What would be your answer in (i) above, if 40% of the shares of RPL were held by
the Provincial Government 48% by the British Government and 12% by individual
investors. (03)

Question # 7
Spring 2017 Q.2(b)

Saleha is a resident person. She disposed of the following assets during the tax year 20X7.

(i) A painting which she inherited from her father was sold for Rs. 1,250,000. The market
value of the painting at the time of inheritance was Rs. 1,550,000. The painting was
purchased by her father for Rs. 1,000,000. (02)

(ii) She sold jewellery for Rs. 2,300,000 which was purchased by her husband in March
20X5 for Rs. 1,300,000 and gifted to her on the same date. (02)

(iii) She disposed of her car for Rs. 1,800,000. The car was being used for the purposes
of her business. The tax ' written down value of the car at the beginning of tax year
20X7 was Rs. 1,600,000. The rate of depreciation
for tax purposes is 20%. (02)

(iv) On 20 October 20X6 she sold a dining table to Faheem for Rs. 18,000 which she had
purchased on 15 May 20X5 for Rs. 15,000 for her personal use. (02)

Required:
Under the provisions of the Income Tax Ordinance, 2001, discuss the taxability of each of
the above transactions in the context of capital gain/loss.

Question # 8
Spring 2017 Q. 3(b)

Najam had purchased a house in 20X2 for Rs. 20 million.

On 1 July 20X6, Najam entered into an agreement with Zameer for sale of the
house for Rs. 25 million. As per the terms of the agreement, Najam received Rs. 5
million on the day the contract was signed and balance amount yvas to be paid on 30
September 20X6. However, due to financial difficulties, Zameer failed to pay the
balance amount on the due date and consequently, Najam forfeited the advance in
accordance with the terms of the agreement.

On 15 February 20X7 Najam sold the house to Farid for Rs. 30 million.

Required:
Advise Najam about the taxability of the above transaction under the Income Tax
Ordinance, 2001. (04)

Question # 9
Autumn 2017 Q. 3(b)

Under the provisions of the Income Tax Ordinance, 2001 compute taxable income or
loss, under the correct head of income for tax year 2017, in each of the following
cases:

124
Tariq Hussain Tunio

(d) Zaheer sold a painting to his brother on 10 April 2017 for Rs. 2,000,000. Zaheer had
purchased this painting for his residence, in an auction on 14 August 2013 for Rs. 1,800,000.
(02)

Question # 10
Autumn 2019 Q. 3(b)

Respond to the following independent scenarios, under the provisions of the Income Tax
Ordinance. 2001:

(b) Haris sold two of his personal vehicles dunng the current year and earned profit of Rs.
550,000. Discuss the taxability of profit earned by Haris in the context of capital galn/loss.
(02)

Question # 11
Spring 2020 Q.4(b)

Respond to the following independent situations, under the provisions of the Income Tax
Ordinance, 2001:

(b) On 1 July 2014, Ahmed purchased two sculptures for Rs. 410,000 and Rs. 475,000
respectively. On 30 November 2019, during the shifting of his house, he lost both the
sculptures. On 15 January 2020, he received insurance claim of Rs. 940,000 in a single
transaction against the loss of two sculptures. The fair market value of both the sculptures at
the time of loss was estimated at Rs. 360,000 and Rs. 540,000 respectively. Compute
Ahmed's taxable income or loss for the above transaction. (04)

Question # 12
Autumn 2022 Q. 3

(a) Nargis is a resident filer. During the tax year 2022, she disposed of various assets.
Relevant details of these assets are as follows

Disposal Purchase
Cash Fair market Date of Cost of Date of
Consideration value disposal purchase purchase
— Rs. in million —
Investment in 2.8 3.0 01-Apr-22 2.0 01-Jun-20
shares of a
public
unlisted
company
Investment in 3.5 3.5 01-Jul-21 -2.1 30-Jun-13
shares of a
listed
company
Personal car 5.0 6.0 31-Dec-21 3.8 01-Jan-19
Painting 1.2 1.2 16-Sep-21 1.7 16-Feb-17
Jewelry 8.0 7.6 30-Jun-22 (see note 01-May-16
1)

125
CAF-02 Tax Practices

Note 1: She received the jewelry as a gift from her mother in law at the time of her marriage
when its fair market value was Rs. 4.8 million.

Required:
Under the provisions of the Income Tax Ordinance, 2001 and Rules made thereunder:

1. compute the amount to be chargeable to tax under the head of capital gain.
Also state the reason for ignoring gain / loss, if any. (06)

2. compute the tax liability of Nargis in respect of the capital gain computed in
above assuming that she has no other source of income. (02)

(b) Shahid is a resident filer and has provided following information pertaining to tax year
2022

1. On 16 June 2018, he inherited a bungalow having a fair market value of Rs. 50


million from his father on his death. On 1 January 2022, he decided to sell the
bungalow to Zamin for Rs 60 million and received a deposit of Rs. 6 million. On 14
February 2022, he forfeited the deposit on refusal of Zamin to purchase the
bungalow in accordance with the terms of the contract.

On 31 March 2022, he sold and transferred the bungalow to Kazim for Rs. 54 million.

2. He owns a factory building at Faisalabad. On 1 July 2021, he let out this factory
building along with the plant and machinery at a monthly rent of Rs. 1 million. During
the year, he incurred expenses of Rs. 3.5 million on the repair and maintenance of
the factory.

3. He owns an agricultural land hi Punjab. On 1 January 2022, he rented out the


agriculture landat an annual rent of Rs. 4 million. The fair market value of the annual
rent was Rs. 5 million.

Required:

Under the provisions of the Income Tax Ordinance, 2001 and Rules made thereunder,
compute the taxable income of Shahid under appropriate heads of income for the tax
year 2022. Also compute his tax liability for the tax year 2022. (07)

Note: Show all relevant exemptions, exclusions and disallowances.

Question # 13
Spring 2005 Q.4(b)

Discuss which assets are not considered capital asset for the purpose of determining
income under the head capital gains. (05)

126
Tariq Hussain Tunio

127
CAF-02 Tax Practices

Chapter

07

OTHER SOURCES
INCOME FROM
CHAPTER SYNOPSIS
TOPIC SECTION
Introduction 39
Incomes chargeable under IFOS 39
Deductions in computing IFOS 40
ICAP Past Papers Questions

128
Tariq Hussain Tunio

1. INTRODUCTION

1. Income from Other Sources is a residuary head of income which covers all those
incomes which do not fall under any other head of income.

2. The basis of charge of income from other sources is cash basis i.e. income is
taxed when received, and deduction for an expenditure is allowed when paid.

2. INCOMES CHARGEABLE UNDER IFOS

1. Income of every kind that is not included in any other head of income is chargeable
to tax to a person who receives it in a tax year, except when it is exempt from tax
under the Income Tax Ordinance, 2001.

2. Following incomes are included under Income from Other Sources:

i. Dividend
ii. Royalty
iii. Profit on Debt
iv. Ground Rent

Explanation for Students


Rent that is paid to an owner of a land by a tenant who has constructed a
building over the land is referred to as ‘ground rent’.

v. Any additional payment on delayed refund given to a taxpayer by any


government under any tax law.
Explanation for Students
In case a taxpayer’s refund is duly determined under a tax law, but is
delayed for whatever reason, the tax authorities are liable to be pay
additional payment to the taxpayer under various tax laws. This
payment is treated as income the taxpayer and is chargeable under
IFOS.
vi. Rent from the sub-lease of land or a building

Explanation for Students


In case a land or building is let on rent by a tenant, not being the
owner of the land or building, the rent from that land of building is
chargeable to the tenant under IFOS.

vii. Income from the lease (i.e. letting on rent) of any building together with
plant or machinery (Example: a factory).

viii. Income from provision of amenities, utilities or any other service


connected with renting of building

ix. Any annuity or pension (Example: Pension received by a widow from


employer of deceased husband).

129
CAF-02 Tax Practices

x. Any prize bond, or winnings from a raffle, lottery, prize on winning a quiz,
prize offered by companies for promotion of sale or cross-word puzzle.

xi. Any other amount received as consideration for the provision, use or
exploitation of property, including from the grant of a right to explore for,
or exploit, natural resources.

xii. The fair market value of any benefit, whether convertible to money or not,
received by a person in connection with the provision, use or exploitation
of property.

xiii. Any amount received by a person as consideration for vacating the


possession of a building or part of a building, reduced by any amount paid
by the person to acquire possession of such building or part thereof.

Method of Taxation: This amount is chargeable to tax in the tax year in


which it is received and the following nine tax years in equal proportion.
‹§18(2)›

xiv. Any amount received by a person from Approved Income Payment Plan
or Approved Annuity Plan under Voluntary Pension System Rules, 2005.

xv. Any amount or fair market value of any property received from as gift,
other than gift received from relative as defined in section 85(5).

xvi. Income arising to the shareholder of a company, from the issuance of


bonus shares.

Loan, Advance, Gift & Deposit for Share – Treated Income

3. Any amount of loan, advance (except advance paid for sale of goods or supply of
services), deposit for issuance of shares received by a company, or a gift is treated
as income chargeable to tax under IFOS if:

▪ The amount is received from any person except banking company or financial
institution.
▪ The amount is received otherwise than by a crossed cheque drawn on a bank
or through a banking channel e.g. received in cash.
▪ The amount is received from a person who does not hold a National Tax
Number (NTN).

Profits on Debt Pertaining to Previous Tax Years

4. If an amount of profit on debt pertaining to previous tax years is paid to a


person in current tax year and as a result it is chargeable to tax at higher tax rate, the
person has an option to elect for taxation of that profit on debt at the tax rates of the
tax year to which it relates, subject to following conditions:

▪ Profit on debt is derived from investment in National Savings Deposit


Certificates (including Defense Savings Certificates.

130
Tariq Hussain Tunio

▪ Profit on debt is paid in arrears or the amount received includes profit on debt
chargeable to tax in the tax year or years preceding the tax year in which it is
received.

▪ The person is required to opt for taxation at the rates of relevant tax
year/years by notice in writing to the Commissioner

▪ This option is to be made by the due date for furnishing the person’s return of
income for the tax year in which the amount was received or by such later
date as the Commissioner may allow by an order in writing.

3. DEDUCTIONS IN COMPUTING IFOS

1. A deduction is allowed under IFOS for any expenditure to the extent to which the
expenditure is paid in deriving income chargeable to tax under IFOS.

2. Deduction for an expenditure of capital nature is not allowed, except for depreciation
in relation to a building together with plant and machinery used deriving income
chargeable to tax under IFOS.

3. Expenditure is of a capital nature if it has a normal useful life of more than one year.
§40(6)

4. No deduction is allowed for an expenditure that is deductible under any other head of
income.

5. Any deduction that is not allowed under section 21 (‘Deductions not allowed’) is not
allowed in computing income under IFOS.

Deduction for Zakat from PoD

6. A person receiving profit on debt chargeable to tax under IFOS is allowed a


deduction for zakat, subject to following conditions:

▪ Zakat is paid by the person under the Zakat and Ushr Ordinance, 1980.
▪ Deduction is allowed at the time the profit on debt is paid to the person.

Deduction for Depreciation

7. A person receiving income from lease of a building together with plant or machinery
chargeable to tax under IFOS is allowed a deduction for:

▪ depreciation of building, plant or machinery in accordance with §.22. (Note


that initial allowance u/s 23 is not allowed in relation to the building)
▪ initial allowance for plant or machinery in accordance with §. 23

131
CAF-02 Tax Practices

PAST EXAMINATION QUESTIONS

Question # 1
Spring 2014 Q.4

Bashir and Jameel jointly own a house in Karachi. Bashir has 75% share in the house. On 1
September 20X3, the house was let out at an annual rental value of Rs. 6,500,000. This
amount includes Rs. 186,000 per month for utilities, cleaning and security.

During the tax year 20X4, the owners incurred the following expenditures in relation to the
house:
Rupees
Utilities, cleaning and security 650,000

Repair and maintenance 810,000


Insurance premium 240,000
Collection charges 25,400
Mark-up on amount borrowed for extension of the house 840,000

Bashir and Jameel have no other source of income. All the above expenses were incurred
by them jointly.

Required:
Calculate taxable income of Bashir and Jameel under appropriate heads of income for the
tax year 20X4. (10)

Question # 2
Spring 2017 Q. 3(a)

On 1 June 20X6 Dawood and Dewan jointly purchased a bungalow for Rs. 35 million.
They paid the amount in the ratio of 65:35 respectively. To arrange hinds for the deal,
Dawood borrowed Rs. 3,000,000 in cash from Shameem who is in the business of
lending money. The rate of interest is agreed @ 20% per annum.

On 1 July 20X6, the house was let out to a company at annual rent of Rs. 4,500,000
inclusive of an amount of Rs.75,000 per month for utilities, cleaning and security. For
providing these services Dawood and Dewan paid Rs.35,000 per month. During the
tax year 20X7 they also paid Rs. 10,000 as collection charges and Rs. 230,000 for
administering the property.

Required:
Compute taxable income of Dawood and Dewan under appropriate heads of income
for the tax year 20X7. (08)

132
Tariq Hussain Tunio

Question # 3
Spring 2019 Q.3(c )

On 1 July 20X8, Zahid rented out his properties as follows:

(i) An apartment Was rented to Abdul Qadir at a monthly rent of Rs. 40,000. Zahid
received a non-adjustable security deposit of Rs. 300,000 which was partly used to repay
the non-adjustable security deposit amounting to Rs. 175,000 received from the previous
tenant in July 20X3. He also spent Rs. 20,000 on repairs of the apartment in February 20X9.
(ii) A bungalow was rented to a bank. Zahid and his younger brother are joint owners
of the bungalow in the ratio of 60:40 respectively. The annual rent agreed with the bank was
Rs. 6,000,000 which is inclusive of Rs.100,000 per month for utilities, cleaning and security.
Zahid paid Rs. 35,000 per month for providing these services.

Required:
Under the provisions of Income Tax Ordinance, 2001 compute total and taxable income of
Zahid for the tax year 20X9 under appropriate heads of income. (07)

Question # 4
Autumn 2019 Q. 2(b)

During the tax year 20X9, Amjad carried out the following transactions in respect of his
properties:

(i) On 1 July 20X8, Amjad purchased a factory building in Sukkur along with die
installed machinery at the price of Rs. 9 million and Rs. 3 million respectively. To manage
the shortage of funds of Rs. 2,000,000, he borrowed the same on 1 July 20X8 from his friend
Shamshad through a crossed cheque. The loan carries interest at the rate of 18% per
annum.

On 1 January 20X9, he let out this building along with the machinery to Basit at a monthly
rent of Rs. 500,000 payable in. advance.

(ii) On 1 July 20X8, Amjad let out his residential property situated in DHA Karachi to
Mirza Limited at a monthly rent of Rs. 300,000. Rent for the two years was received in
advance on 1 August 20X8.

(iii) On 1 July 20X8, Amjad also entered into an agreement with Zeeshan for the sale of
his plot situated in Quetta for Rs. 50 million. The plot had been purchased for Rs. 40 million
in 20X4. Under the terms of sale agreement, he received Rs. 5 million at the time of signing
the agreement and the balance was to be received on 30 September 20X8. However, due to
financial difficulties, Zeeshan failed to pay the balance amount on the due date and
consequently, Amjad forfeited the advance in accordance with die terms of the agreement.
On 10 April 20X9, he finally sold the plot to Jamshed for Rs. 65 million.

(iv) Following expenditures were incurred by Amjad in respect of his properties in


Sukkur and Karachi:

133
CAF-02 Tax Practices

Details of Expenditures Property situated in


Sukkur karachi
Repair & maintenance – building 270,000 70,000
- Machinery 50,000 -
Ground rent 50,000 10,000
Insurance – building 150,000 20,000
Total 520,000 100,000

Required:

In view of the provisions of the Income Tax Ordinance, 2001 compute under
appropriate head of income, taxable income of Amjad for the tax year 20X9. (10)

Question # 5
Spring 2020 Q. 4(a)

Respond to the following independent situations, under the provisions of the Income Tax
Ordinance, 2001:

(c) During the tax year 2020, Sadiq received a flat as gift from his uncle, Mumtaz Alvi.
The flat was located in posh area of Lahore and its fair market value at the time of
gift was Rs. 4.5 million. Discuss the tax treatment of the flat received by Sadiq.
(02)

Question # 6
Autumn 2020 Q. 4(a)

Farheen is a resident filer and has provided following information pertaining to tax year 2020:

(i) She owns a bungalow situated in Multan which was given on rent to Abbas under
a agreement of five years which expired on 31 March 2020. Details of payments
received as per the rent agreement are given below.

Rent Rs. 175,000 per month


Security guards-salaries Rs. 50,000
Non-adjustable security Rs. 2,500,000
deposit

On expiry pf the rental agreement, Farheen refunded the security deposit to Abbas
and rented out the bungalow to a new tenant Zafar on the same terms and conditions.
Farheen pays Rs. 40,000 per month to a security services company which provides
security guards at the bungalow.

(ii) She owns a residential plot m Karachi. On 1 March 2020, she decided to sell the
plot to Mehreen for Rs. 2,200,000 and received a deposit of Rs. 220,000. On 1 June.

134
Tariq Hussain Tunio

2020, she forfeited the deposit on refusal of Mehreen to purchase the plot.

(iii) On 1 December 2017, she had acquired a furnished office on monthly rent Of Rs.
5,000 for her own use and had paid a non-re fundable amount of Rs. 2,000,000 to
the previous tenant for vacating the office. During the year, she received an offer of
Rs. 2,400,000 from Shehroz to vacate this office which she accepted and received
the amount on 1 March 2020.

(iv) On 1 October 2019, she inherited a factory with plant and machinery from her father
and let it out on 1 December 2019 at a monthly rent of Rs. 500,000.

(v) Legal and professional charges of Rs. 40,000 were paid for preparation of rental
agreements.

(vi) On 15 November 2019, she received income tax refund of Rs. 180,000 related to tax
year 2017. This amount included Rs. 30,000 being additional payment-on delayed
refund.

Required:

Under the provisions of the Income Tax Ordinance,2001 and Rules made thereunder,
compute the total income of Farheen under appropriate heads of income for the tax year
2020. (07)

Question # 7
Autumn 1999 Q.4(ii)

Mr. Fawad got possession of a shop on 21.07.2004 by paying Rs. 100,000 as pugree to the
outgoing tenant. On 26.05.2005, he vacated the possession of the said shop and received
Rs. 650,000 as consideration for vacating the possession. What will be tax treatment of this
amount in his income for tax year 2005? (04)

Question # 8
Autumn 2006 Q.1( c )

Specify under which head of income, following amounts of rent would be chargeable to tax:

1. Rent in respect of lease of a building together with plant and machinery.


2. Amount included in the rent of a building for the provision of amenities,
utilities or any other service connected with the renting of such building. (02)

Question # 9
Spring 2008 Q.2(a)

Mr. Anil is constructing his house for the purpose of meeting construction expenses, he
intends to take a personal loan of Rs. 500,000 from Mr. Kamran who is in the business of
money lending. He has been advised by one of his friends that such a loan may be included
in his taxable income,under certain circumstances.

You are required to advise Mr. Anil about the circumstances under which the loan may be
included in his taxable income. (04)

135
CAF-02 Tax Practices

Shoo! Shoo! I'm Tax


Exempt
Chapter
TAX
08

EXEMPTIONS & TAX


CONCESSION
CHAPTER SYNOPSIS
TOPIC SECTION
Introduction General & 55
Agricultural income 41
Diplomatic and united nations 42
exemptions
Foreign government officials 43
Exemption under international 44
agreement
Exemptions under Foreign Investments 44A
President’s honors 45
Profit on debt 46
Scholarships 47
Support under agreement to live apart 48
Federal, provincial & local government 49
Foreign source income of short term 50
resident
Foreign source income of returning 51
expatriates
Exemptions and tax concessions in 2nd 53
schedule
Exemptions and tax provisions in other 54
laws
Foreign source salary of resident 102
individual
ICAP Past Papers Questions

136
Tariq Hussain Tunio

1. INTRODUCTION

1. All incomes are taxable unless exempt under Income Tax Ordinance, 2001.

2. If any income is exempt from tax under the Income Tax Ordinance, 2001,

▪ the exemption is limited to the original recipient of that income and

▪ is not extendable to any person receiving any payment out of the exempt
income. ‹§55›

2. AGRICULTURAL INCOME §.41

1. Agricultural income derived by a person

▪ is exempt from tax


▪ under the Income Tax Ordinance, 2001.

Agricultural Income

2. Agricultural Income means:

▪ any rent or revenue derived by a person from land that:

- is situated in Pakistan, and

- is used for agricultural purposes.

▪ any income derived by a person from land situated in Pakistan:

- from agriculture,

- from the performance by a cultivator or receiver of rent-in-kind of any


process ordinarily employed by such person to render the produce raised
or received by the person fit to be taken to market,

Explanation for Students

A landowner may let on rent his agricultural land to a tenant and receive from him rent
in cash or in kind. In some situations, the landowner or the tenant perform certain
processes to make the produce fit to be taken to market.

For example, in case of cultivation of tea or tobacco, the tea or tobacco leaves need to
be dried before they can be sold in the market. This process of drying adds value to
the produce raised or received as rent in kind. Any income that is attributable to the
performance of this process is brought within the ambit of agricultural income; hence,
exempt from tax.

Income attributable to any process, not being a natural process for making the
produce fit to be taken to market, is chargeable to tax. For example, income
attributable to grinding or packing of tea leave is taxable.

137
CAF-02 Tax Practices

▪ from the sale by a cultivator or receiver of rent-in-kind of the produce raised or


received by such person, in respect of which a process of nature has been
performed for making the produce fit to be taken to market.

Explanation for Students


Having performed the natural process to make the produce fit to be taken
to market the landowner or cultivator sells the produce in the market. Any
income that arises on such sale is treated as agricultural income and is
exempt from tax.

▪ any income derived by a person

(i) from any building owned and occupied by the receiver of the rent/
revenue of an agricultural land, or
(ii) any building occupied by the receiver of rent-in-kind or the cultivator of
agricultural land if:

- the building is on, or in the immediate vicinity of the agricultural


land, and

- it is a building which is required as a dwelling-house, a store-house,


or other out-building by the receiver of the rent or revenue, or the
cultivator, or the receiver of the rent-in-kind by reason of the
person’s connection with the land.

Explanation for Students


A landowner may allow the tenant to use his agricultural buildings that
are situated in the vicinity of land and are used for functions connected
with agricultural land. Any income arising from such buildings is also
treated as agricultural income; hence exempt from tax.

Examples

▪ Agricultural Incomes

- Income from land used for grazing


- Income from land used for growing tea
- Income from land used for growing fruits
- Income from land used for growing flowers
- Income from from cultivated forest

▪ Non-Agricultural Incomes

- Income from fee charged for cutting down timber


- Income from profit on sale of timber by a middleman who bought
it from the cultivator
- Income from Income of sugar, tea, rice factories
- Income from Income from sale of water for irrigation
- Income from Income from dairy-farming, and poultry farming

138
Tariq Hussain Tunio

3. DIPLOMATIC AND UNITED NATIONS EXEMPTIONS §.42

1. The income of an individual entitled to privileges under the Diplomatic and Consular
Privileges Act, 1972 is exempt from tax under ITO-2001 to the extent provided for in
that Act.

2. The income of an individual entitled to privileges under the United Nations (Privileges
and Immunities) Act, 1948, is exempt from tax under ITO-2001 to the extent provided
for in that Act.

3. Pension received by an employee of United Nations is also exempt from tax subject
to the following conditions:

▪ Person is a citizen of Pakistan

▪ Pension is received by the person by virtue of the his former employment in


the United Nations or its specialized agencies (including the International
Court of Justice), &

▪ Person’s salary from such employment was exempt under Income Tax
Ordinance, 2001.

4. FOREIGN GOVERNMENT OFFICIALS §.43

1. Any salary received by an employee of a foreign government as remuneration for


services rendered to such government in Pakistan is exempt from tax subject to
following conditions:

▪ The employee is a citizen of the foreign country and not a citizen of


Pakistan,

▪ The services performed by the employee are of a character similar to


those performed by employees of the Government of Pakistan in that
foreign countries, and

▪ The foreign government grants a similar exemption to employees of the


Government of Pakistan performing similar services in that foreign
country.

ILLUSTRATION
X, an employee of Information Ministry of Government of Sri Lanka, is deputed to work as
reporter in Islamabad for the official TV channel. Similarly, Y, an employee of Ministry of
Information and Broadcasting, Government of Pakistan, is sent to Sri Lanka to report for PTV
News. During Tax Year 20X1, X became resident individual in Pakistan and received salary
amounting to Rs. 1,500,000/- from his Government. Salary of X is exempt from tax in
Pakistan if salary of Y is exempted from tax in Sri Lanka.

5. EXEMPTIONS UNDER INTERNATIONAL AGREEMENTS §.44

1. Any Pakistan-source income (PSI) which Pakistan is not permitted to tax under a tax
treaty is exempt from tax under Income Tax Ordinance, 2001.

139
CAF-02 Tax Practices

Exemption of Salary of Individual under Aid Agreement

2. Any salary received by an individual (not being a citizen of Pakistan) is exempt from
tax under Income Tax Ordinance, 2001 to the extent provided for in an Aid
Agreement between the Federal Government and a foreign government or public
international organization, where:

▪ the individual is either a non-resident individual or a resident individual solely


by reason of the performance of services under the Aid Agreement;

▪ if the Aid Agreement is with a foreign country, the individual is a citizen of that
country; and

▪ the salary is paid by the foreign government or public international


organization out of funds or grants released as aid to Pakistan in pursuance
of such Agreement.

Exemption for Foreign Contractor, Consultant, or Expert

3. Any income received by a person (not being a citizen of Pakistan) engaged as a


contractor, consultant, or expert on a project in Pakistan is exempt from tax under the
Income Tax Ordinance, 2001 to the extent provided for in a bilateral or multilateral
agreement between the Federal Government and a foreign government or public
international organization, where:

▪ the project is financed out of grant funds in accordance with the agreement;

▪ the person is either a non-resident person or a resident person solely by


reason of the performance of services under the agreement; and

▪ the income is paid out of the funds of the grant in pursuance of the
agreement.

6. EXEMPTION UNDER FOREIGN INVESTMENT §.44A

1. Taxes on income (including capital gains), advance tax, withholding taxes,


minimum and final taxes under this Ordinance shall, for the period and to the extent
provided in the Second and Third Schedules to the Foreign Investment (Promotion
and Protection) Act, 2022 (XXXV of 2022) in respect of qualified investment as
specified at Sr. No.1 of the First Schedule to the said Act or investors, be exempt or
subject to tax at the rate and in the manner specified under the said Act.

2. All investors and shareholders of the qualified investment, their associates and
companies specified in the Second and Third Schedules to the said Act including third
party lenders on account of any loan shall also be exempt from taxes and other
provisions of this Ordinance or subject to tax at the rate and in the manner specified
under the said Act for the period and to the extent provided in the Second and Third
Schedules to the said Act.

3. Provisions of this Ordinance relating to Anti-Avoidance, for the period and to the
extent specified in the said Act including sections 106, 106A, 108, 109 and 109A, shall
not apply to the persons and amounts mentioned in sub-sections (1) and (2).

140
Tariq Hussain Tunio

4. Rates of depreciation, initial allowance and pre-commencement expenditure under


sections 22, 23 and 25 as on the 20th day of March, 2022 shall continue to be
applicable for thirty years as provided in the Third Schedule to the said Act in respect
of persons mentioned in sub-sections (1) and (2).

5. For the purpose of this section, the terms defined under the Second and Third
Schedules to the said Act shall apply mutatis mutandis to this Ordinance.

7. PRESIDENT'S HONORS §.45

1. Any allowance attached to any Honor, Award, or Medal awarded to a person by the
President of Pakistan is exempt from tax.

2. Any monetary award granted to a person by the President of Pakistan is exempt from
tax.

8. PROFIT ON DEBT §.46

1. Any profit on debt received by a non-resident person on a security issued by a


resident person is exempt from tax if:

▪ the payer and the non-resident recipient of profit on debt are not associates,

▪ the security was widely issued by the resident person outside Pakistan for the
purposes of raising a loan outside Pakistan for use in a business carried on
by the person in Pakistan;

▪ the profit was paid outside Pakistan; and


▪ the security is approved by the FBR for this purpose.

ILLUSTRATION
To expand its business operation, ABC Ltd, a local company having registered office
in Karachi, raised finances from outside Pakistan by issuing bonds in the UAE. The
issuance of bonds was advertised in all the major newspapers of UAE and was
approved by the FBR. X, who was not an associate of ABC Ltd, was one of the
persons who purchased this security. During Tax Year 20X1, he received Rs.
750,000/- from ABC Ltd account of the profit on debt on the bonds. This income of X
is exempt from tax.

9. SCHOLARSHIPS §.47

1. Any amount given as scholarship is exempt from tax subject to the following
conditions:

▪ It is granted to a person to meet the cost of the person’s education

▪ The person to whom scholarship is given is not directly or indirectly an


associate of the payer.

141
CAF-02 Tax Practices

10. SUPPORT UNDER AGREEMENT TO LIVE APART §.48

1. Any income received by a spouse as support payment under an agreement to live


apart is exempt from tax under the Income Tax Ordinance, 2001.

11. FEDERAL, PROVINCIAL AND LOCAL GOVERNMENTS §.49

1. The income of the Federal Government is exempt from tax under the Income Tax
Ordinance, 2001.

2. The income of a Provincial Government is exempt from tax, other than income from
business derived by a Provincial Government from a business carried on in another
province.

3. The income of a Local Government is exempt from tax, other than income
chargeable under the head “Income from Business” derived by a local Government
from a business carried on in the areas of another local government.

4. Any payment received by the Federal Government, a Provincial Government or a


Local Government is not liable to any collection or deduction of advance tax at
source, except for income of Provincial Government or Local Government that is
subject to tax.

5. Regardless of the ultimate destination of such income, the following entities are not
exempt from tax in respect of income derived by them:

▪ A corporation, a company, a regulatory authority, a development authority, other


body or institution established by or under a Federal law or a Provincial law or an
existing law.

▪ A corporation, company, a regulatory authority, a development authority or other


body or institution set up, owned and controlled by the Federal Government or a
Provincial Government either directly or indirectly.

Examples:

- Pakistan International Airlines Corporation


- Pakistan State Oil
- Pakistan industrial Development Corporation
- Sindh Bank

12. FOREIGN SOURCE INCOME OF SHORT TERM RESIDENT INDIVIDUALS


§.50

1. The foreign-source income of an individual is exempt from tax if:

▪ the person is a resident individual solely by reason of the individual’s


employment, and

▪ the person is present in Pakistan for a period or periods not exceeding


three years.

142
Tariq Hussain Tunio

2. This exemption is not applicable to:


▪ any income derived from a business of the person established in Pakistan; or
▪ any foreign-source income brought into or received in Pakistan by the person.

13. FOREIGN SOURCE INCOME OF RETURNING EXPATRIATES §.51

1. Expatriate means somebody who has left his or her homeland to live or work in
another country, usually for a long period of time .

2. Any foreign source income of returning expatriate is exempt from tax for two years
i.e. in the tax year in which the individual became resident individual and in the
following year subject to the conditions that:

▪ The person is a citizen of Pakistan


▪ The person was not a resident individual in any of the four tax years preceding
the tax year in which the individual became a resident

Exemption of Salary of an Individual Leaving Pakistan

3. If a citizen of Pakistan leaves Pakistan during a tax year and remains abroad during
that tax year, any income chargeable under the head “Salary” earned by him outside
Pakistan during that year is exempt from tax.

14. EXEMPTION AND TAX CONCESSION IN SECOND SCHEDULE §.53

1. The income or classes of income, or persons or classes of persons specified in the


Second Schedule to the Income Tax Ordinance, 2001 are:

▪ totally exempt from tax (subject to any conditions and to the extent specified in
the said schedule)

▪ subject to lower rates of tax

▪ allowed a reduction in tax liability (subject to any conditions and to the extent
specified in the said schedule) or

▪ exempted from the operation of any provision of Income Tax Ordinance, 2001
(subject to any conditions and to the extent specified in the said schedule).

Federal Government’s Power to Amend Provisions of 2 nd Schedule

2. The Federal Government or the Board with the approval of the Federal Minister-in-
charge may, from time to time, pursuant to the approval of the Economic
Coordination Committee of the Cabinet whenever circumstances exist to take
immediate action for the purposes of

• national security,
• natural disaster,
• national food security in emergency situations,
• protection of national economic interests in situations arising out of abnormal
fluctuation in international commodity prices,

143
CAF-02 Tax Practices

• implementation of bilateral and multilateral agreements or


• granting an exemption from any tax imposed under this Ordinance

✓ including

➢ a reduction in the rate of tax imposed under the Income Tax


Ordinance, 2001 or

➢ a reduction in tax liability under the Income Tax Ordinance, 2001


or

➢ an exemption from the operation of any provision of the Income


Tax Ordinance, 2001 to any international financial institution or
foreign Government owned financial institution operating under
an agreement, memorandum of understanding or any other
arrangement with the Government of Pakistan, by notification in
the official Gazette, make such amendment in the Second
Schedule by—

▪ adding any clause or condition therein,

▪ omitting any clause or condition therein, or

▪ making any change in any clause or condition therein,

as the Government may think fit, and all such amendments shall have effect
in respect of any tax year beginning on any date before or after the
commencement of the financial year in which the notification is issued.

3. The Federal Government is required to place before the National Assembly all
amendments made by it to the Second Schedule in the financial year.

4. Any notification issued under sub-section (2) after the commencement of the Finance
Act, 2015, shall, if not earlier rescinded, stand rescinded on the expiry of the financial
year in which it was issued

all such notifications, except those earlier rescinded, shall be deemed to have
been in force with effect from the first day of July, 2016 and shall continue to be
in force till the thirtieth day of June, 2018, if not earlier rescinded:

all notifications issued on or after the first day of July, 2016 and placed
before the National Assembly as required under subsection (3) shall
continue to remain in force till the thirtieth day of June, 2018, if not earlier
rescinded by the Federal Government or the National Assembly.

15. EXEMPTION AND TAX PROVISIONS IN OTHER LAWS §.54

1. Any income may be exempted from tax only by the Income Tax Ordinance,
2001.

2. Provision in any other law relating to following types of exemptions shall have
no legal effect, unless also provided for in the Income Tax Ordinance, 2001:

144
Tariq Hussain Tunio

▪ an exemption from any income tax,


▪ a reduction in the rate of tax,
▪ a reduction in tax liability of any person, or
▪ an exemption from the operation/application of any provision of the Income
Tax Ordinance, 2001.

16. FOREIGN SOURCES SALARY OF RESIDENT INDIVIDUAL §.102

1. Any foreign-source salary received by a resident individual is exempt from tax if the
individual has paid foreign income tax in respect of the salary.

When is Foreign Income Tax treated as Paid?

2. A resident individual is treated as having paid foreign income tax in respect of


foreign-source salary if:

▪ tax has been withheld from the salary by the individual’s employer, and
▪ tax has been paid to the revenue authority of the foreign country in which the
employment was exercised.

145
CAF-02 Tax Practices

146
Tariq Hussain Tunio

Chapter

09

LOSSES
CHAPTER SYNOPSIS
TOPIC SECTION
Introduction General
Setting off of losses §.56
Carry forward of business losses §.57
Carry forward of speculation losses §.58
Carry forward of capital losses §.59
Loss setting off/ carry forward – sum up
chart
Foreign losses §.104
Aop - setting off/ carry forward of losses §.59A
ICAP Past Papers Questions

147
CAF-02 Tax Practices

1. INTRODUCTION

1. If deductions allowed under a head of income exceed the amount chargeable under
that head, the excess is referred to as ‘loss’.

2. Loss cannot arise under the heads ‘Salary’ and ‘Income from Property’, for no
deductions are allowed under those heads.

2. SETTING OFF OF LOSSES §.56

1. A person who sustains a loss in a tax year under any head of income is entitled to set
off the loss against income under any other head for that tax year, except income
under the head Salary.

2. Loss under the head Capital Gain (referred to as ‘capital loss’) cannot be set off
against income under any other head, and loss under Speculation Business (referred
to as ‘speculation loss’) may only be set off against any other speculation income of
the person for the year.

Priority in Setting off of Losses

3. If a person sustains losses under Income from Business and under another head of
income that is allowed to be set off, the loss under the head Income from Business is
to be set off last.

Carry Forward of Losses

4. Where loss cannot be set off against income under any other head, the amount not
set off cannot be carried forward to next tax years for set off, except when it is loss
under normal business, speculation business or capital loss

3. CARRY FORWARD OF BUSINESS LOSSES §.57

1. Loss under “Income from Business” (except unabsorbed depreciation &


speculation loss) for tax year is set off against income under other heads of income for
the tax year.

2. But, if the loss cannot be fully set off against income under other heads, so
much of the loss that has not been set off is carried forward to the following tax year
and set off against the person’s income chargeable under the head “Income from
Business” for that year.

Time Period For Carry Forward of Normal Business Loss

3. Loss under “Income from Business” (except speculation loss) can be carried
forward to six tax years immediately succeeding the tax year for which the loss was
first computed and set off against the income under Income from Business.

148
Tariq Hussain Tunio

Business Loss of Banking Companies

4. Where a loss was sustained by a banking company wholly owned by the Federal
Government as on 01/06/ 2002 for assessment year starting on or after 01 July 1995
and ending on the 30 June 2001, such banking company is allowed to carry forward
the loss for a period of ten years.

5. However, the approval is required to be obtained from the State Bank of Pakistan for
the purpose this extended carrying forward of losses.

Priority to Earliest Loss

6. In case a person carries forward loss under a head of income relating to more than
one tax year, the loss of the earliest tax year is set off first, due to the fact that the
time period of six year allowed for carrying forward will expire for the earlier losses
first.

Business Loss and Unabsorbed Depreciation

7. Loss under Income from Business may arise due to deduction of revenue expenses
or depreciation/amortization deductions for a tax year. Income Tax Ordinance, 2001,
provides different tax treatment to each of these types of losses.

8. Therefore, it is necessary to distinguish between business loss (i.e. arising due to


revenue expenses) & unabsorbed depreciation (i.e. arising due to depreciation
deduction).

9. This distinction is done by first deducting revenue expenses from amounts


chargeable under this head, followed by deduction of depreciations/amortization
deductions.

10. The loss attributable to deductions allowed under 22, 23, 23B and 24 that has not
been set off against income, the loss not set off against 50% of the person’s balance
income chargeable under the head ‘’income from business’’ after setting off loss
under sub-section (1), in the following tax year and so on until completely set off:

Such loss shall be set off against 100% of the said balance income if the taxable
income for the year is less than Rs 10 million.

11. There is no time limit for carry forward of depreciation loss. It may be carried forward
until completely set off.

4. CARRY FORWARD OF SPECULATION LOSSES §.58

1. If a person sustains a loss in one speculation business, but has income in another
speculation business, the person is allowed to set off the loss of one speculation loss
from the income of the other speculation business.

Inter-Head Setting Off Of Speculations Loss Not Allowed

2. A person sustaining a speculation loss is not allowed to set it off against incomes of
the other heads of income including the income from normal business for the tax
year.

149
CAF-02 Tax Practices

Carry Forward Of Speculations Loss

3. Speculation loss, not able to be set off from another speculation income, is carried
forward to next six tax years immediately succeeding the tax year for which the loss
was first computed, and set off against income from speculation business of hose tax
years.

Priority To Earliest Speculations Loss

4. Where a person has a loss brought forward speculation loss for more than one tax
year, the loss of the earliest tax year is set off first, due to the fact that the loss of the
earlier year will reach the time limit of six years first.

5. CARRY FORWARD OF CAPITAL LOSSES §.59

1. If a person sustains a capital loss during a tax year, the person is not allowed to set it
off against income from other heads of income for the year.

Capital Losses Are Carried Forward

2. Capital loss is carried forward to the next six years starting from the end of the tax
year in which the loss was first computed and set off against income from capital
gains of those years.

Priority To Earliest Capital Loss

3. Where a person has a loss brought forward capital loss for more than one tax year,
the loss of the earliest tax year shall be set off first, due to the fact that the loss of the
earlier year will reach the time limit of six years first.

150
Tariq Hussain Tunio

6. LOSSES SETTING OFF/CARRY FORWARD - SUM UP CHART

1. The following table graphically represents and sums up the main points of setting off
and carry forward of losses.

Income
not Income available
availabe
Capital
Income Income from Business
Gains
Tax Year

Salary from IFOS


Capital
Property Business Unabsorbed Speculation Assets Securities
Loss Depreciation Loss

No Loss   × × × 
200X
20X1
20X2
20X3
20X4
20X5
20X6

Notes:

- The symbol (×) represents that loss under the head cannot be set off against income
of other heads.

- The symbol () represents that loss under the head can be set off against income of
other heads.

- The downward box represents that loss can be carried forward. If there is no
downwards box, such as in IFOS, the loss under the head cannot be carried forward.

- The downward box that is closed at six years shows that the loss can be carried
forward only for six tax years.

- The downward box that is not closed, such as in Depreciation Loss, shows that the
loss may be carried forward for unlimited period of time.

7. FOREIGN LOSSES §.104

How Does A Foreign Loss Arise?

1. When deductible expenditures incurred by a person in deriving foreign-source


income (FSI) chargeable to tax under a head of income exceed the amount of FSI
chargeable, foreign loss arises.

151
CAF-02 Tax Practices

2. It is to be noted here that only those expenditures are deducted from foreign source
income which were incurred in connection with deriving it.

Setting Off And Carry Forward Rules

3. The foreign loss is not allowed be set off against Pakistan-source Income (PSI) of the
person. It is required to be carried forward to the following six tax years and set off
against the foreign source income chargeable to tax in those years.

4. In case of brought forward foreign losses belonging to more than two tax years, the
loss for the earliest year is set off first, due to the earlier expiry of the time period of
six year for the earliest loss.

Apportionment Of Deductions

5. Expenditure incurred for the purpose of both the FSI and PSI can not be fully
deducted from any one of these. It is required to be apportioned between FSI and
PSI on any reasonable basis, taking account of the relative nature and size of the
activities to which the amount relates.

8. AOP - SETTING OFF/CARRY FORWARD OF LOSSES §.59A

1. In case of association of persons, any loss of the AOP is set off or carried forward and
set off only against the income of the association.

2. It is not allowed to be distributed among the members of the AOP.

152
Tariq Hussain Tunio

PAST EXAMINATION QUESTIONS

Question # 1
Autumn 2002 Q.6

Please write a brief note about the adjustments of loss incurred under any head of income in
the current year. (03)

Question # 2
Spring 2001 Q.6

A taxpayer was dealing in the business of textile garments during the tax year 2005 has
earned income of Rs.199,237. He has assessed brought forward losses for the tax year
2004 as follows:

• Business of steel Rs. 101,356


• Sales of shares of private company Rs. 105,505
You are required to work out his total income for tax year 2005. (05)

Question # 3
Autumn 2003 Q.3(b)

Briefly explain the law relating to set-off and carry forward of foreign losses? (07)

Question # 4
Autumn 2004 Q.8

Describe the provisions relating to set-off and carry forward of foreign losses under the
Income Tax Ordinance.2001? (03)

Question # 5
Spring 2006 Q.7(b)

The records of Mr. A show the following results:

Particulars Rs.
Loss under the head "income from other source" after (20,000)
setting off dividend income of Rs. 30,000
Income from Speculation business 10,000
Capital gain on disposal of shares of private limited 20,000
companies
Loss from textile business after considering tax depreciation (410,000)
of Rs.290,000

You are required to work out the following:


(i) Taxable income;
(ii) Tax liability; and
(iii) Amount of loss that can be:
- Adjusted against any other head of income;
- Carried forward for a maximum of six years;

153
CAF-02 Tax Practices

- Carried forward for indefinite period. (06)


Question # 6
Spring 2007 Q.4(b)

What do you understand by the term ‘’speculation business’’ as referred to in the Income
Tax Ordinance,2001? Briefly discuss the rules relating to set off and carry forward of losses
arising out of speculation business. (04)

Question # 7
Spring 2007 Q.6(a)

Under the Income Tax Ordinance,2001 regarding set off and carry forward of losses under
the heads ‘’Income from Business” and ‘’Capital Gains”. (07)

Question # 8
Spring 2013 Q.3(b)

Mr. Imran Emad (IE) formed his business two years ago. During the latest tax year, IE's
Pakistan source income amounted to Rs 2,500,000.
Following are the details of its foreign source incomes, tax paid thereon and foreign losses
brought forward for the latest tax year:

Foreign income Foreign tax paid Foreign losses


brought forward
Rupees
speculation 500,000 125,000 (250,000)
business
Non-speculation (1,000,000) - -
business
Capital gains 750,000 75,000 (1,500,000)
Other sources 1,250,000 187,500 -

The foreign tax credit relating to income from other sources which remained unadjusted
during the last tax year amounted to Rs. 50,000.

Required:
Calculate total tax payable and foreign tax losses to be carried forward to next year (if
any). (08)

Question # 9
Spring 2014 Q.6(b)

Explain the term ‘foreign losses’. State the provisions relating to set off and carry
forward of foreign losses, under the Income Tax Ordinance,2001. (04)

Question # 10
Spring 2015 Q.6

Aslam is a resident taxpayer who operates his business from Lahore (LHR) and Paris
(PAR). In August 2014, he established a new branch in Berlin (BER).

154
Tariq Hussain Tunio

Following information is available in respect of his business operations for tax year 2015:

LHR PAR BER

-Rs. in million-
Income/(loss) from business 29 40 (15)
Advance taxes paid in respective countries during the year 10 5 3
Income from capital gain (net of income tax of Rs. 3 million) - 27 -
Carried forward losses:
Loss from business - 55 -
Capital loss - 6 -

The following amounts paid by Aslam in respect of BER have been charged to LHR:
(i) salaries for the first three months amounting to Rs.5 million.
(ii) rent expense for the year amounting to Rs.7 million.

Required:
Under the provisions of the Income Tax Ordinance, 2001 calculate the tax payable by Aslam
in the tax year 2015 and foreign tax losses to be carried forward to next year, if any. (09)

Question # 11
Autumn 2016 Q. 8

On 1 July 2015 Mehreen joined a local newspaper as an investigative journalist at a salary


of Rs. 300,000 per month.

Tax deducted u/s 149 from her salary amounted to Rs. 40,000 per month. are the details of
her income received from Germany; tax paid thereon and brought forward foreign losses for
tax year 2016:

Heads of Foreign income/ Foreign Foreign losses


income (loss) tax paid brought
forward
Rupees
Speculation 600,000 110,000 (380,000)
business
Non- 1,480,000 187,600 -
speculation
business
Other sources (1,500,000) - -
Capital gain 950,000 76,000 (1,800,000)

On 1 May 2016 Mehreen resigned from her current job and joined Akhbar Merhaba (AM), an
arabic newspapaer in Dubai, as editor-in-chief on a monthly salary equivalent to PKR
1,200,000. AM paid 50% of her salary in Dubai and remitted the remaining 50% to her bank
account in Pakistan through normal banking channel. Mehreen remained in Dubai during the
rest of the tax year 2016.

Required:
Under the provisions of the Income Tax Ordinance, 2001 and Rules made thereunder,
compute the taxable income, net tax payable by or refundable to Mehreen for tax year 2016
and the amount of foreign losses or foreign tax credit. (10)

155
CAF-02 Tax Practices

Question # 12
Spring 2018 Q.4( c )

Discuss the provisions of the Income Tax Ordinance,2001 regarding set off and carry
forward of losses under the following heads:
(i) income from business (05)
(ii) income from speculation business (04)

Question # 13
Autumn Q.3( c )

Jameel and Company (JC) is the sole trader of a trader of a branded tea in Pakistan. In
addition to the trading business, JC is also engaged in forward purchasing and selling of tea
to reap the benefits of price fluctuations in local and international markets. Following
information has been extracted from the records of JC for the year ended 30 June 20x8:

(i) Detail of trading and speculation business (forward purchase and sale) were as
follows:
Trading Speculation
business business
Rs. in million
Gross revenue 400 200
Gross profit 20 10

(ii) Total administrative anad general expenses for the year amounted to Rs. 7.2 million.
This amount includes a penalty of Rs. 0.4 million paid to the custom authorities.

(iii) Assessed carried forward losses from previous years as follows:

Rs. in million
Losses from trading business 12.8
Losses from speculation business 9.6
Capital losses (incurred in 20x2) 2.0

Under the Income Tax Ordinance,2001 and Rules made under thereunder, compute
JC’s taxable income /(loss) and the amount of loss to be carried forward, if any, for the
tax year 20x8.

156
Tariq Hussain Tunio

How many extra bones


will i get if Govt allows
deductible allowance
Chapter
on purchase of dog
food !!!
10

ALLOWANCE
DEDUCTIBLE
CHAPTER SYNOPSIS
TOPIC SECTION
Introduction General
Zakat 60
Workers' Welfare Fund 60A
Workers' Profit Participation
60B
Fund
Deductible allowance for
60D
education expenses

157
CAF-02 Tax Practices

1. INTRODUCTION

1. Deductible allowance is an amount that is reduced from total income to arrive at


person’s taxable income for a tax year.

2. Deductible allowances cannot reduce total income below zero, which means loss
cannot arise due to deductible allowances.

3. There are four deductible allowances:

(iii) Zakat
(iv) Worker’s Welfare Fund
(v) Workers’ Participation Fund
(vi) Deductible Allowance for Education Expense

2. ZAKAT §.60

1. A person is entitled to a deductible allowance for the amount of any Zakat if:

▪ An amount is paid as Zakat by a person. Deductible allowance is not allowable if


Zakat is payable in a tax year

▪ Zakat is paid in accordance with the Zakat and Ushr Ordinance, 1980.

What if Zakat is Not Able to be Deducted?

2. Amount of Zakat not able to be deducted from total income, for being in excess of the
amount of person’s total income, is not refunded, carried forward to a subsequent tax
year, or carried back to a preceding tax year.

Zakat as Allowance vs. Zakat Deduction

3. Income Tax Ordinance, 2001, provides an option to a person who has paid Zakat
paid in accordance with Zakat and Ushr Ordinance, 1980 to be claimed either as
deductible allowance or as deduction, in case the person has PoD income under
IFOS.

4. As a general rule, amount of Zakat paid is allowed as deductible allowance i.e.


it is reduced from total income. However, in one instance it is also allowed as
deduction under the head ‘Income from Other Source’.

5. A person receiving profit on debt chargeable to tax under the head “Income
from Other Sources” is allowed a deduction for Zakat paid by the person under the
Zakat and Ushr Ordinance, 1980 at the time the profit is paid to the person.

6. Amount of Zakat allowed as deduction under the Income from Other Sources
can not be claimed as deductible allowance.

158
Tariq Hussain Tunio

3. WORKERS' WELFARE FUND §.60A

1. A person is entitled to a deductible allowance for the amount of Workers’ Welfare


Fund paid by the person in tax year under Workers’ Welfare Fund Ordinance, 1971
or under any law relating to the Workers’ Welfare Fund enacted by Provinces after
the eighteenth Constitutional amendment Act, 2010

Deductible allowance is not allowed in respect of any amount of Workers’ Welfare


Fund paid to the Provinces by a trans-provincial establishment.

Explanation for Students


Workers’ Welfare Fund was set up by the Federal Government for providing
residential accommodation and other facilities to workers. Initially Federal
Government contributed Rs. 100 (M) in the fund, after which annual
contribution equal to two percent (2%) of total income is made by certain
taxpayers. These taxpayers are also allowed to deduct the amount paid in
the fund from their total income as deductible allowance.

4. WORKERS' PROFIT PARTICIATION FUND §.60B

1. A person is entitled to a deductible allowance for the amount of Workers’


Participation Fund paid by the person in a tax year in accordance with the provisions
of the Companies Profit (Workers’ Participation) Act, 1968 or under any law relating
to the Workers’ Profit Participation Fund enacted by Provinces after the eighteenth
Constitutional amendment Act, 2010:

Deductible allowance is not allowed in respect of any amount of Workers’ Profit


Participation Fund paid to the province by a trans-provincial establishment.

Explanation for Students


Workers’ Participation Fund was set up under Companies profit (workers'
participation) Act, 1968 by certain companies, which make contribution into it.
Two-third of the amount contributed is distributed among the workers in cash
and the remaining one-third is utilized for the purpose of welfare of the
workers.

5. DEDUCTIBLE ALLOWANCE FOR EDUCATION EXPENSE §.60D

1. Every individual shall be entitled to a deductible allowance in respect of tuition fee


paid by the individual in a tax year provided that the taxable income of the individual
is less than one and a half million rupees.

2.The amount of an individual‘s deductible allowance allowed under sub-section (1) for
a tax year shall not exceed the lesser of —

(a) five per cent of the total tuition fee paid by the individual referred to in sub-
section (1) in the year;
(b) twenty-five per cent of the person’s taxable income for the year; and

159
CAF-02 Tax Practices

(c) an amount computed by multiplying sixty thousand with number of children of


the individual.

3. Any allowance or part of an allowance under this section for a tax year that is not
able to be deducted for the year shall not be carried forward to a subsequent tax
year.
4. Allowance under this section shall be allowed against the tax liability of either of the
parents making payment of the feeon furnishing national tax number (NTN) or name
of the educational institution.
5. Allowance under this section shall not be taken into account for computation of tax
deduction under section 149.

160
Tariq Hussain Tunio

Chapter

11

TAX CREDITS
CHAPTER SYNOPSIS

TOPIC SECTION
Introduction 4(3)(4)
Charitable donations 61
Contribution to an approved pension fund 63
Tax credit for point of sale machine 64D
Miscellaneous provisions relating to tax 65
credits
Tax credit for certain persons 65F
Tax Credit For specified Industrial 65G
Undertaking
Foreign Tax Credit 103

161
CAF-02 Tax Practices

1. INTRODUCTION

1. Any amount that is reduced from tax liability for a tax year to arrive at person's tax
payable is referred to as 'tax credit'.

2. If a taxpayer is allowed more than one tax credits in a tax year, the credits are
allowed in the following order:

(i) Foreign Tax Credit


(ii) Group of Tax Credits
(iii) Credit for advance tax and adjustable withholding tax.

Explanation for Students


The reason for this particular order is that the first two categories are not
refundable in case they are not able to be deducted from tax liability,
whereas the final category is refunded in that case.

2. CHARTITABLE DONATION §.61

1. A tax credit is allowed to a person in respect of any sum paid or any property given
by the person in the tax year as a donation voluntarily contribution or subscription to
any of the following:

(i) board of education,

(ii) university established in Pakistan by, or under, a federal/ provincial law,

(iii) educational institution established or run in Pakistan by FG, PG or LG,

(iv) hospital established or run in Pakistan by FG, PG or LG,

(v) relief fund established or run in Pakistan by FG, PG or LG, or

(vi) non-profit organization

(vii) any person eligible for tax credit u/s 100C

(viii) entities, organization and funds mentioned in 13 th schedule.

Computation of Tax Credit for Donation

2. The amount of tax credit allowed to a person in connection with donation for a tax
year is computed according to the following formula:

(A/B) x C

Where:

A= is the amount of tax assessed to the person for the tax year before
allowance of any tax credit (under Part X of Chapter III of the Ordinance),

162
Tariq Hussain Tunio

B= is the person’s taxable income for the tax year, and

C= is the lesser of:

(a) the total amount of the person’s donations in the year, including the
fair market value of any property given; or
(b) where the person is:

Person Amount
Individual & AOP 30% of Taxable Income
Company 20% of Taxable Income

COMPONENT ‘C’ INCASE OF ASSOCIATES:

If sum is paid or property is given to an associate by a donor, component ‘C’ shall be


as follows:

a) an individual or association of persons, 15% of the taxable income of the person


for the year; or

b) a company, 10% of the taxable income of the person for the year.

Total donation
amount paid by
crossed cheque
/FMV of Property
computed at the time
of donation

Tax Liability for the Year


Taxable Income for the x Lesser of
Year

30% of Taxable
Income for
Individual/AOP and
20% of Taxable
Income for Company

Notes:

▪ Fair market value of any property given shall be determined at the


time it is given.

▪ A cash amount paid by a person as a donation shall be taken into


account of only if it was paid by a crossed cheque drawn on a bank.

163
CAF-02 Tax Practices

3. CONTRIBUTION TO AN APPROVED PENSION FUND §.63

1. An eligible person deriving income chargeable to tax under the head “Salary” or the
head “Income from Business” is entitled to a tax credit for a tax year in respect of any
contribution or premium paid in the year by the person in approved pension fund
under the Voluntary Pension System Rules, 2005.

Computation of Tax Credit

2. The amount of a person’s tax credit allowed for a tax year is computed according to
the following formula:
(A/B) x C
Where

A= is the amount of tax assessed to the person for the tax year, before allowance
of any tax credit (under Part X of Chapter III of the Ordinance),
B= is the person’s taxable income for the tax year, and
C= is the lesser of:

(i) total contribution or premium paid by the person in the year, or

(ii) 20% of the eligible person’s taxable income for the relevant tax year.

Provided that an eligible person joining the pension fund at the


age of forty-one years or above, during the first ten years starting from
July 1, 2006 shall be allowed additional contribution of 2% per annum
for each year of age exceeding forty years. Provided further that the
total contribution allowed to such person shall not exceed 50% of the
total taxable income of the preceding year.
Total contribution or
premium paid
Tax Liability for the Year
Taxable Income for the x Lesser of
Year

20% of Taxable
Income for the Tax
Year

3. The transfer by the members of approved employment pension or annuity


scheme or approved occupational saving scheme of their existing balance to their
individual pension accounts maintained with one or more pension fund managers does
not qualify for this tax credit.

4. TAX CREDT FOR POINT OF SALE MACHINE §.64D

1. Any person who is required to integrate with Board’s computerized system for
real time reporting of sale or receipt, shall be entitled to tax credit in respect of the
amount invested in purchase of point of sale machine.

164
Tariq Hussain Tunio

2. The amount of tax credit allowed under sub-section (1) for a tax year in which
point of sale machine is installed, integrated and configured with the Board’s
computerized system shall be lesser of—

(a) amount actually invested in purchase of point of sale machine; or

(b) rupees one hundred and fifty thousand per machine.

3. For the purpose of this section, the term point of sale machine means a machine
meant for processing and recording the sale transactions for goods or services,
either in cash or through credit and debit cards or online payments in an internet
enabled environment.

5. MISCELLANEOUS PROVISION RELATING TO TAX CREDITS §.65

Member of Association of Person (AOP)

1. The components ‘A’ and ‘B’ in the above formulae for computation of tax credits is
taken to mean as follows in case a person is a member of an AOP and is entitled to a
tax credit on  Charitable donation,  Investment in shares/life insurance, 
Contribution to approved pension fund or  Profit on debt,

A= amount of tax that would be assessed to the individual if any amount


derived in the year that is exempt from tax (i.e. share from AOP) were
chargeable to tax; and

B= the taxable income of the individual for the year if any amount derived in the
year that is exempt from tax (i.e. share from AOP) were chargeable to tax.

Refund, Carry Forward or Carry Back

2. Any of the above tax credit or its part that is not able to be credited in the year is not
refunded, carried forward to a subsequent tax year, or carried back to a preceding
tax year.

Claim of Excess Tax Credit by AOP

3. However, if the excess tax credit pertains to a member of AOP, it may be claimed by
the AOP in that year subject to the condition that the member and the AOP agree in
writing in this regard and the agreement is furnished to the CIR with the return of the
AOP.

6. TAX CREDIT FOR CERTAIN PERSONS §.65F

1. Following persons or incomes shall be allowed a tax credit equal to one hundred per
cent of the tax payable under any provisions of this Ordinance including minimum,
alternate corporate tax and final taxes for the period, to the extent, upon fulfillment of
conditions and subject to limitations detailed as under:-

(a) persons engaged in coal mining projects in Sindh supplying coal exclusively to
power generation projects;

165
CAF-02 Tax Practices

(b) a startup as defined in clause (62A) of section 2 for the tax year in which the
startup is certified by the Pakistan Software Export Board and the next following
two tax years; and

2. The tax credit under sub-section (1) shall be available subject to fulfillment of the
following conditions, where applicable, namely:-

(a) return has been filed;

(b) withholding tax statements for the relevant tax year have been filed in respect of
those provisions of the Ordinance, where the person is a withholding agent; and

(c) sales tax returns for the tax periods corresponding to relevant tax year have been
filed if the person is required to file Sales Tax Return under any of the Federal or
Provincial sales tax laws.

7. TAX CREDIT FOR SPECIFIED INDUSTRIAL UNDERTAKING §.65G

1. When making certain eligible capital investments as specified in sub-section (2), the
eligible taxpayers defined in sub-section (3) shall be allowed to take an investment
tax credit of twenty five percent of the eligible investment amount, against tax
payable under the provisions of this Ordinance including minimum and final taxes.
The tax credit not fully adjusted during the year of investment shall be carried forward
to the subsequent tax year subject to the condition that it may be carried forward for
a period not exceeding two years.

2. For the purposes of this section, the eligible investment means investment made in
purchase and installation of new machinery, buildings, equipment, hardware and
software, except self-created software and used capital goods.

3. For the purpose of this section, eligible person means —


(a) green field industrial undertaking as defined in clause (27A) of section 2
engaged in —

(i) the manufacture of goods or materials or the subjection of goods or


materials to any process which substantially changes their original
condition; or

(ii) ship building:

Provided that the person incorporated between the 30th day


of June, 2019 and the 30th day of June, 2024 and the person is not
formed by the splitting up or reconstitution of an undertaking already
in existence or by transfer of machinery, plant or building from an
undertaking established in Pakistan prior to commencement of the
new business and is not part of an expansion project; and

(b) industrial undertaking set up by the 30th day of June 2023 and engaged in
the manufacture of plant, machinery, equipment and items with dedicated use

166
Tariq Hussain Tunio

(no multiple uses) for generation of renewable energy from sources like solar
and wind, for a period of five years beginning from the date such industrial
undertaking is set up.

8. FOREIGN TAX CREDIT §.103

1. Foreign tax credit is allowed when the following conditions are satisfied:

▪ a resident taxpayer derives foreign source income chargeable to tax in


Pakistan,

▪ the taxpayer has paid foreign income tax in respect of that income, and

▪ the foreign income tax is paid within two years after the end of the tax year in
which the foreign income to which the tax relates was derived.

Computation of Tax Credit

2. Foreign tax credit is allowed equal to an amount that is lesser of:

(a) the foreign income tax paid; or

(b) the Pakistan tax payable in respect of the income.

What is ‘Pakistan Tax Payable’?

3. The Pakistan tax payable in respect of foreign source income is computed

▪ by applying the average rate of Pakistan income tax applicable to the


taxpayer for the year

▪ against his net foreign-source income for the year.

What is ‘Average Rate of Pakistan Income Tax’?

4. Average Rate of Pakistan Income Tax in relation to a taxpayer for a tax year means
the percentage that the Pakistani income tax (before allowance of this tax credit) is of
the taxable income of the taxpayer for the year.

What is taxpayer’s net foreign-source income for the year?

5. Net Foreign-Source Income in relation to a taxpayer for a tax year, means the total
foreign-source income of the taxpayer charged to tax in the year, as reduced by any
deductions allowed to the taxpayer under Income Tax Ordinance,2001 for the year
that:

(a) relate exclusively to the derivation of the foreign-source income, and

(b) are reasonably related to the derivation of foreign-source income in


accordance apportionment of common expenses (§.67).

167
CAF-02 Tax Practices

Foreign Tax Credit in Case of More than One Head of Income

6. In case a taxpayer has foreign income under more than one head of income
(speculation business being treated as separate head as well), the foreign tax credit
is applied separately to each head of income.

168
Tariq Hussain Tunio

Chapter

12

COMMON RULES
CHAPTER SYNOPSIS
TOPIC SECTION
Income of joint owners 66
Apportionment of deductions 67
Fair market value 68
Receipt of income 69
Recouped expenditure 70
Currency conversion 71
Cessation of source of income 72
Double derivation and double
73
deductions
ICAP Past Papers Questions

169
CAF-02 Tax Practices

1. INCOME OF JOINT OWNERS §.66

1. Two or more persons owning a property jointly are not assessed as association of
persons (AOP) in respect of that property if their respective shares in that property
are definite and ascertainable.

2. The share of each person in the income from the property is charged to tax
separately in the computation of each person’s taxable income.

3. This provision does not apply to computing is not applicable to a property that
produces Income from Business.

ILLUSTRATION
X and Y jointly purchased an office building, each having fifty percent share in the property.
The building was rented to Z at annual rent amounting to Rs. 640,000/- during Tax Year
20X1. This amount, being Income from Property jointly earned, is not chargeable to both the
owners as AOP, but rather Rs. 320,000/- is to be charged to tax under Income from Property
in each person’s income for the tax year.

2. APPORTIONMENT OF DEDUCTIONS §.67

1. An expenditure deductions or allowances are apportioned if they relate to derivation


(i.e. earning) of:

i. more than one head of income,

ii. income subject to normal tax (NTR) and subject to final tax (FTR), or

iii. income chargeable to tax under a head of income and to some other purpose
i.e. exempt income.

2. Apportionment of expenditure, deductions or allowances is to be done on any


reasonable basis, taking into account the relative nature and size of the activities to
which the amount of expenditure relates.

3. Income Tax Rules, 2002, (Rule.13) provide for the following method of apportionment
of expenditures.

A x B/C
Where:

A = is the amount of the common expenditure incurred,

B = is the total amount of gross receipts of the particular class of income for
the tax year (without deduction of expenditures), and

C = is the total amount of gross receipts and net gains of all classes of
income for the tax year (without deduction of expenses)

170
Tariq Hussain Tunio

Explanation for Students


▪ An expenditure that is related to a particular head of income or class of
income, the said expenditure is allowed as deduction only under that
head or class of income and is referred to as relatable or allocable
expenses.

▪ An expenditure that is not clearly allocable to any particular class or


classes of income, such as general administrative and other such
allocable expenditures, is referred to as common expenditure. (Rule 13 of
ITR-2002).

▪ Common expenditure is apportioned or prorated among the heads of


income or classes of incomes for which it was incurred, as described
above.

3. FAIR MARKET VALUE §.68

1. The Fair Market Value (FMV) any property, rent, asset, service, benefit or perquisite
at a particular time is the price which is ordinarily fetched by it on sale or supply in the
open market at that time.

2. FMV of any property, rent, asset, service, benefit or perquisite is to be determined


without regard to any restriction on transfer or to the fact the it is not otherwise
convertible to cash.

ILLUSTRATION
Mr. X lives in a house in Lahore. The ownership of the house is disputed between him and his
younger brother Mr. Y in a court, which has imposed a restriction on sale of he property until
the judgment of the court. Mr. X wants to know the FVM of his house. The real estate agent
valued the house at Rs. 8 (m) due to restriction on it transfer. However, if there were no
restriction on the transfer of the property, it would fetch Rs.10 (m). In this case, the FMV of
the house for the purpose of Income Tax Law shall be taken at Rs. 10 (m).

Determination of FMV by the Commissioner

3. Where the price, other than the price of immoveable property, is not ordinarily
ascertainable, such price may be determined by the Commissioner.

4. The Board may, from time to time, by notification in the official Gazette, determine
the fair market value of immovable property of the area or areas as may be specified
in the notification.

5. Where the fair market value of any immovable property of an area or areas has not
been determined by the Board in the notification, the fair market value of such
immovable property shall be deemed to be the value of fixed by the District Officier
(Revenue) or provincial or any other authority authorized in this behalf for the
purposes of stamp duty.

171
CAF-02 Tax Practices

4. RECEIPT OF ICNOME §.69

1. A person is treated as having received an amount, benefit, or perquisite if it is:

▪ actually received by the person,

▪ applied on behalf of the person, at the instruction of the person or under any law,
or

▪ made available to the person.

5. RECOUPTED EXPENDITURE §.70

1. To recoup means ‘to get something back’.

2. In case a person has received back wholly or partly an amount of expenditure or


loss that was previously allowed as deduction in computing income chargeable
under a head of income, the amount thus recouped, whether in cash or in kind, is
chargeable to tax under the head in which it was previously allowed as deduction in
the tax year in which it is received.

ILLUSTRATION
Pure Milk (Pvt.) Ltd incurred advertisement expenditure amounting to Rs. 950,000/- in Tax
Year 2009, which was allowed as deduction in computing its Income from Business for the
said tax year. However, the company received back Rs. 20,000/- from one advertising
company as special discount in Tax Year 2010. The amount received back is chargeable to tax
under the head Income from Business in Tax Year 2010.

6. CURRENCY CONVERSION §.71

1. For the purposes of income tax law, every amount is to be taken into account in Pak
Rupees only.

2. If an amount is in a currency other than rupees, the amount is to be converted


to the Rupee at the State Bank of Pakistan rate applying between the foreign
currency and the Rupee on the date the amount is taken into account for the
purposes of income tax law.

7. CESSATION OF SOURCES OF INCOME §.72

1. Where any income is derived by a person in a tax year from any business,
activity, investment or other source that has ceased either before the commencement
of the year or during the year; and

2. If the income had been derived before the business, activity, investment or
other source ceased it would have been chargeable to tax under ITO-2001, the
Ordinance shall apply to the income on the basis that the business, activity,
investment or other source had not ceased at the time the income was derived.

172
Tariq Hussain Tunio

8. DOUBLE DERIVATION AND DOUBLE DEDUCTION §.73

Rules to Prevent Double Derivation Of Income

1. Any amount is chargeable to tax under ITO-2001 on the basis that it is receivable,
the amount shall not be chargeable again on the basis that it is received.

2. Any amount is chargeable to tax under ITO-2001 on the basis that it is received, the
amount shall not be chargeable again on the basis that it is receivable.

Rules to Prevent Double Deduction


3. any expenditure is deductible under ITO-2001 on the basis that it is payable, the
expenditure shall not be deductible again on the basis that it is paid.

4. any expenditure is deductible under ITO-2001 on the basis that it is paid, the
expenditure shall not be deductible again on the basis that it is payable.

173
CAF-02 Tax Practices

PAST EXAMINATION QUESTIONS

This section includes ICAP past papers of multiple topics listed below:

1. Losses
2. Deductible allowances
3. Tax credits
4. Exemptions

Question # 1
Autumn 2009 Q.2(b)

State the provisions of Income Tax Ordinance, 2001 pertaining to foreign tax credit available
to a resident taxpayer. (06)

Question # 2
Autumn 2009 Q.5

Mr. Abdullah, an employee of a Malaysian based company, has been assigned to work in
Karachi, in its subsidiary company which is registered under the Companies Ordinance,
1984. The initial assignment of two years commenced on March 1, 2009 and would be
extended subject to mutual agreement.
Mr. Abdullah’s remuneration will be paid in Malaysia, details of which are given below:

Nature of Income Amount in Equivalent Rupees


Pakistan source salary income for the tax 5,750,000
year 2009
Pakistan source salary income for the tax 17,250,000
year 2010
Foreign source salary income for the tax 12,000,000
year 2009
Foreign source salary income for the tax 3,000,000
year 2010

Required:
(a) Explain the residential status of Mr. Abdullah under the Income Tax Ordinance,
2001 for the tax year 2009 and 2010. (02)

(b) Compute taxable income of Mr. Abdullah for the tax years 2009 and 2010.
Support your computation with appropriate comments. (08)

Question # 3
Spring 2010 Q.2

Mr. Qamar intends to donate an amount of Rs. 10 million to certain educational and
welfare institutions. In your capacity as his tax consultant, explain the tax relief which

174
Tariq Hussain Tunio

may be available in respect of such donation and the conditions he must fulfill to avail such
relief. (09)

Question # 4
Spring 2012 Q. 2(c)

List the persons or incomes that are allowed a tax credit equal to 100% of the tax payable.
Also specify the conditions/limitations which are required to be fulfilled for availing the said
tax credit. (04)

Question # 5
Autumn 2012 Q. 3(b,c)

Margaret, a German national was employed as a Technical Manager of Faiza Chemicals


Limited, a resident company, on 1 October 2010 for a term of two years. Under the terms of
employment, she was allowed to deliver lectures at various professional organizations.
During tax year 2012.he conducted three workshop sessions, the details of which are as
follows:

• Workshop Session in Lahore: A fee of US$ 15,000 in equivalent Pak Rupees


was received from a local event manager. The fee was credited to her bank
account maintained in Karachi.

• Workshop Session in Munich: A fee of US$ 25,000 was received in Germany


in her Munich bank account.

• Workshop Session in Dubai: A fee of US$ 20,000 was remitted to her bank
account in Karachi.

Required:

Discuss the taxability of the amounts received by Margaret for conducting the workshop
sessions during tax year 2012. (06)

Explain the provisions of the Income Tax Ordinance, 2001 pertaining to foreign tax credit
available to a resident taxpayer. (06)

Question # 6
Spring 2016 Q.4

Lone Traders (LT), a sole proprietorship, is engaged in the business of buying and selling of
Maize and Wheat in bulk quantities. Following information has been extracted from LT's
records for the year ended 31 December 2015:

(i) Wheat sold to food companies in Punjab amounted to Rs. 13,000,000. The sale
was made after allowing discount of Rs. 680,000 to some of the new customers.
The gross profit margin was 25% on gross sales.

(ii) LT paid Rs. 600,000 to a research institute for the development of a formula
which is likely to improve the quality of wheat it purchases from the growers.

(iii) In August 2015, LT signed a future contract with Mubarak Enterprises (ME) for

175
CAF-02 Tax Practices

the purchase of 500 metric tons of maize at Rs. 15,800 per metric ton. The
delivery was expected to be made in October 2015. ME also agreed to
repurchase the entire lot at the price prevailing on the date of sale.

(iv) In October 2015 price of maize increased to Rs. 18,240 per metric ton and LT
sold the entire lot to ME without taking delivery.

(v) LT incurred expenditure of Rs. 25,000 in respect of above future contract.

(vi) Administrative, selling and distribution expenses amounted to Rs. 2,500,000. These
included a penalty of Rs. 45,000 which was imposed due to late payment of sales
tax on wheat.
(vii) Assessed losses brought forward from previous year were as follows:

Rupees
Trading business loss 550,000
Speculation business 300,000
Capital loss 250,000

Required:

Under the provisions of the Income Tax Ordinance, 2001 and Rules made thereunder,
compute LT's taxable income/(loss) and the amount of loss to be, carried forward, if any, for
tax year 2016. (10)

Question # 7
Autumn 2017 Q. 2(b,c)

(a) explain the treatment of foreign source income for tax year 2017 under each of
the following independent situations:

(i) Joseph, a South African cricket coach is working in Pakistan under an


employment, contract since 20 July 2014. During the tax year 2017, he
earned foreign source income from his business established in South Africa
and brought 25% of the income to Pakistan. (04)

(ii) On 15 January 2016 Farhan returned to Pakistan from London after 10


years and has been living in Pakistan since then. During the tax year 2017, he
received GBP 5,000 as return from his investment in London. (02)

(c) Determine the amount of deductible allowance that a resident individual can
claim on account of education expenses, if his taxable income for the year was
Rs. 800,000 and he paid monthly fee of Rs. 6,000 per child for his three children.
(02)

176
Tariq Hussain Tunio

Question # 8
Spring2018 Q. 2(a)

Under the provisions of the Income Tax Ordinance, 2001 compute taxable income or loss
under correct head of income for tax year 20X8, in each of the following cases:

(a) Mrs. Raees separated from her spouse due to certain disagreements. Under an
agreement to live apart, her spouse provided her a house and paid cash of Rs.
150,000 per month as support payment. The fair market rent of the house is Rs.
50,000 per month. (02)

Question # 9
Spring 2018 Q. 4(a)

Nadeem has agricultural land in Thatta which is being used for the cultivation of sugarcane.
During the year, he cultivated 200,000 tonnes of sugarcane. Out of total cultivation, 140,000
tonnes of sugar cane was sold to a sugar mill at a price of Rs. 4,550 per ton whereas the
remaining quantity was utilized in his own sugar mill. During the year, there were no other
purchases of sugar cane by his sugar mill.

The sale of his sugar mill stood at Rs. 310 million whereas total expenses other than the
raw material amounted to Rs. 19 million. There was no opening and closing stock of
sugarcane.

Required:
Under the provisions of the Income Tax Ordinance, 2001 and Rules made thereunder,
compute the taxable income of Nadeem for the tax year 20X8. (04)

Question # 10
Spring 2019 Q. 2(b)

Mohsin has been working at the head office of Lewis Consulting, Inc. (LCI) situated in New
York, USA. On 1 January 20X8, LCI had established its branch office in Pakistan and had
sent Mohsin for two years as Country Manager for looking after the Pakistan operations.

During the tax year 20X9, apart from salary income, Mohsin earned/received the following
amounts:

• On 15 December 20X8, he conducted a seminar in USA for a fee of USD 18,000. On


his request, the event manager transferred the amount (net of tax) directly to his
personal bank account in Islamabad on 10 January 20X9.

• On 31 May 20X9, he earned income from his business established in USA and
brought 40% of the income to Pakistan.

Required:
Under the Income Tax Ordinance, 2001:

(i) State the residential status of Mohsin for the tax year 20X8. (01)

177
CAF-02 Tax Practices

(ii) Discuss the taxability of his foreign source incomes for the tax year 20X9. (04)
Question # 11
Autumn 2020 Q.3(b)

Differentiate between deductible allowances and admissible deductions. Give three


examples of each. (06)

Question # 12
Spring 2021 Q.4

(a) On 1 July 20X1, Mrs. Ahmed separated from her spouse and decided to live apart with
her six years old son. Below are the extracts of clauses from the agreement to live apart:

(i) Mr. Ahmed will pay Rs. 50,000 in cash every month to his spouse.
(ii) Mr. Ahmed will continue to pay his son's monthly school fee Rs. 10,000.
(iii) Mr. Ahmed will transfer the ownership of a shop in her spouse's name. The shop
was already in use by a tenant at a monthly rent of Rs. 88,000. Mrs. Ahmed will be
entitled to receive the rent from the date of transfer of ownership in her name.

On 1 September 20X1, the ownership of the shop was transferred in her name.

Required:

Under the provision of the Income Tax Ordinance, 2001 briefly explain the tax treatment of
the above arrangement in the income tax return of Mrs. Ahmed for the tax year 20X2. Also
specify the head of income under which each of the above receipts will be classified. (04)
(Computation is not required)

(b) Briefly explain the provisions of the Income Tax Ordinance, 2001 and Rules made
thereunder relating to:

(i) order of application of various tax credits if a taxpayer is allowed more than one tax
credit for a tax year. (03)

Question # 13
Autumn 2021 Q.2(b)

Following information pertains to Ms. Ayesha for the tax year 2021:

Required:
Under the Income Tax Ordinance, 2001, discuss how the above losses can be set off
against her aforesaid incomes. Also discuss the amount of losses that can be carried
forward for adjustment against her future incomes. (08)

Question # 14
Spring 2006 Q.4(b)

Mr. Ali, a Pakistani Citizen, returned to Pakistan in November 2004 after completing
his employment contract in Pakistan since then and has been employed by a local
company.

178
Tariq Hussain Tunio

Explain the tax implications on Mr. Ali’s income, earned in UAE and Pakistan, for the tax
year 2005. (04)

This section includes ICAP past papers of foreign source income of a resident
persons:
Question # 1
Autumn 2009 Q. 2(b)

State the provisions of Income Tax Ordinance, 2001 pertaining to foreign tax credit available
to a resident taxpayer. (06)

Question # 2
Autumn 2009 Q. 5

Mr. Abdullah, an employee of a Malaysian based company, has been assigned to work in
Karachi, in its subsidiary company which is registered under the Companies Ordinance,
1984. The initial assignment of two years commenced on March 1,2009 and would be
extended subject to mutual agreement.

Mr. Abdullah's remuneration will be paid in Malaysia, details of which are given below:

Nature of Income Amount in


Equivalent Rupees
Pakistan source salary income for the tax year 2009 5,750,000
Pakistan source salary income for the tax year 2010 17,250,000
Foreign source salary income for the tax year 2009 12,000,000
Foreign source salary income for the tax year 2010 3,000,000

Required:
(a) Explain the residential status of Mr. Abdullah under the Income Tax Ordinance, 2001
for the tax year 2009 and 2010. (02)

(b) Compute taxable income of Mr. Abdullah for the tax years 2009 and 2010. Support
your computation with appropriate comments. (08)

Question # 3
Autumn 2012 Q. 3(b, c)

Margaret, a German national was employed as a Technical Manager of Faiza Chemicals


Limited, a resident company, on 1 October 2010 for a term of two years. Under the terms of
employment, she was allowed to deliver lectures at various professional organizations.
During tax year 2012, she conducted three workshop sessions, the details of which are as
follows:

Question # 4
Spring 2015 Q. 6

Aslam is a resident taxpayer who operates his business from Lahore (LHR) and Paris (PAR).
In August 2014, he established a new branch in Berlin (BER).

Following information is available in respect of his business operations for tax year 2015:

179
CAF-02 Tax Practices

LHR PAR BER


Rs. in million
Income / (loss) from business 29 40 (15)
Advance taxes paid in respective countries during the year 10 5 3
Income from capital gain (net of income tax of Rs. 3 million) - 27 -
Carried forward losses:
Loss from business - 55 -
Capital loss - 6 -

The following amounts paid by Aslam in respect of BER have been charged to LHR:

(i) salaries for the first three months amounting to Rs. 5 million.
(ii) rent expense for the year amounting to Rs. 7 million.

Required:

Under the provisions of the Income Tax Ordinance, ,2001 calculate the tax payable by
Aslam in the tax year 2015 and foreign tax losses to be carried forward to next year, if any.
(09)

Question # 5
Autumn 2016 Q. 8

On 1 July 2015 Mehreen joined a local newspaper as an investigative journalist at a salary of


Rs. 300,000 per month. Tax deducted u'/s 149 from her salary amounted to Rs. 40,000 per
month.

Following are the details of her income received from Germany; tax paid thereon and
brought forward foreign losses for tax year 2016:

Heads of income Foreign Foreign tax paid Foreign losses


income/ (loss) brought forward

Speculation business 600,000 110,000 (380,000)


Non-speculation 1,480,000 187,600 -
business
Other sources (1,500,000) - -
Capital gain 950,000 76,000 (1,800,000)

On 1 May 2016 Mehreen resigned from her current job and joined Akhbar Merhaba
(AM), an Arabic newspaper in Dubai, as editor-in-chief on a monthly salary equivalent
to PKR 1,200,000. AM paid 50% of her salary in Dubai and remitted the remaining
50% to her bank account in Pakistan through normal banking channel. Mehreen
remained in Dubai during the rest of the tax year 2016.

Required:

Under the provisions of the Income Tax Ordinance, 2001 and Rules made thereunder,
compute the taxable income, net tax payable by or refundable to Mehreen for tax year

180
Tariq Hussain Tunio

2016 and the amount of foreign losses or foreign tax credit, if any, to be.carried forward. (10)
Question # 6
Autumn 2017 Q. 2(b)

Explain the treatment of foreign source income for tax year 2017 under each of the following
independent situations:

a. Joseph, a South African cricket coach is working in Pakistan under an employment


contract since 20 July 2014. During the tax year 2017, he earned foreign source
income from his business established in South Africa and brought 25% of the income
to Pakistan. (04)
b. On 15 January 2016 Farhan returned to Pakistan from London after 10 years and
has been living in Pakistan since then. During the tax year 2017, he received GBP
5,000 as return from his investment in London. (02)

Question # 7
Spring 2019 Q. 2(b)

Mohsin has been working at the head office of Lewis Consulting, Inc. (LCI) situated in New
York, USA. On 1 January 20X8, LCI had established its branch office in Pakistan and had
sent Mohsin for two years as Country Manager for looking after the Pakistan operations.

During the tax year 20X9, apart from salary income, Mohsin earned/received the following
amounts:
• On 15 December 20X8, he conducted a seminar in USA for a fee of USD 18,000. On
his request, the event manager transferred the amount (net of tax) directly to his
personal bank account in Islamabad on 10 January 20X9.

• On 31 May 20X9, he earned income from his business established in USA and
brought 40% of the income to Pakistan.
Required:

Under the Income Tax Ordinance, 2001:


(i) state the residential status of Mohsin for the tax year 20X8. (01)
(ii) discuss the taxability of his foreign source incomes for the tax year 20X9. (04)

Question # 8
Autumn 2020 Q.4(b)

Ahmed has completed his MBA from a university in USA. He had been living there since
August 2013 for his education and came to Pakistan only once in 20i7 i.e. from 10 March
2017 to 30 September 2017 and then went back to USA to complete his MBA. Along with his
studies, he was also doing a part time job at a restaurant in USA till November 2019. He
returned to Pakistan on 1 December 2019 and commenced a trading business from 1
January 2020.

Below is the computation for taxable income/loss for the tax year 2020:

Pakistan Foreign Total


source source
income income
Income from Salary Rupees
Salary from restaurant in 840,000 840,000
USA

181
CAF-02 Tax Practices

Income from business


Revenue 4,000,000 4,000,000
Less: Deductions
Cost of goods sold (2,200,000) (2,200,000)
Selling and administrative (2,820,000) (2,820,000)
expenses [Note A]
Donation [Note B] (600,000) (250,000) (850,000)
Taxable income/loss (1,620,000) 590,000 (1,030,000)

Note A: Selling and administrative expenses include the following:

(i) Salaries of Rs. 840,000 paid to two employees equally in cash. Withholding income
tax was deducted as required under Income fax Ordinance, ,2001.
(ii) Rs. 600,000 in respect of the feasibility study which was conducted before
commencement of the business.

Note B: Donation of Rs. 600,000 was paid to a charitable hospital in Pakistan and Rs.
250,000 was paid to a non-profit organization in USA.

Required:

Under the provisions'of the Income Tax Ordinance, 2001 and Rules made thereunder,
comment on the above tax computation for tax year 2020. Give suggestion(s) wherever
required. (08)

Question # 9
Autumn 2022 Q.4

Under the provisions of the Income Tax Ordinance, 2001, discuss the taxability of foreign
source income of the following resident persons for tax year 2022. (Computation of tax
amount, if any, is not required)

a) bi, a Chinese engineer, has been working since March 2020 as a production
manager in a Karachi based company, Karam Limited. During the tax year 2022, his
bank account in China was credited with CNY 40,000 on account of rental income for
his apartment situated in China. He remitted 40% of this amount to his bank account
in Pakistan. (03)

b) Omar, a Pakistani national, came into Pakistan on 1 September 2021 after 20


years of service in UAE. On 1 January 2022, his bank account in UAE Was credited
with AED 50,000 on account of dividend received from a UAE based company. He
remitted 50% of this amount to his bank account in Pakistan. (03)

c) Sidra, a Pakistani national, left Pakistan on 1 July 2021 for employment in a


Singapore based company at a monthly salary of SGD 10,000. However, due to
personal reasons, she returned back to Pakistan on 20 December 2021 and
remained in Pakistan till 30 June 2022. She brought the entire foreign salary amount
to Pakistan. (02)

182
Tariq Hussain Tunio

Chapter

13

INDIVIDUAL AND AOP


TAXATION OF
CHAPTER SYNOPSIS
TOPIC SECTION
Principles of taxation of individuals 86
Deceased individuals 87
Taxation of associations of persons 92
An individual as a member of AOP 88
Authors 89
Change in the constitution of an
98A
association of persons
Discontinuance of business or
dissolution of an association of
98B
persons
Succession to business, otherwise
98C
than on death
ICAP Past Papers Questions

183
CAF-02 Tax Practices

1. PRINCIPLES OF TAXATION OF INDIVIDUALS §.86

1. The taxable income of each individual is to be determined separately. While


computing income chargeable to tax, income of one individual cannot be clubbed
together with the income of another person unless it is expressly provided in the ITO-
2001.

2. An example of clubbing together income of two persons is that of clubbing income of


a minor child with that of its parent with highest income in the tax year.

2. DECEASED INDIVIDUALS §.87

1. Legal representative of a deceased person is liable for

(i) any tax that a deceased person would have become liable for if he had not
died, and

(ii) any tax payable in respect of the income of the deceased person’s estate.

Meaning of Legal representative

2. Legal representative means a person who:

▪ in law represents the estate of a deceased person, and includes

▪ any person who intermeddles with the estate of the deceased, & where a
party sues or is sued in representative character the person on whom the
estate devolves on the death of the party so suing or sued.

Extent of Legal Representative’s Liability

3. The liability of a legal representative for deceased person shall be limited to the
extent to which the deceased’s estate is capable of meeting the liability.

4. Thus, legal representative’s own property shall not be used to meet the tax
liabilities of the deceased person.

Legal Proceedings against Deceased Person

5. Any proceeding taken against the deceased person before his/her death shall
be treated as taken against the legal representative and may be continued against
the legal representative from the stage at which the proceeding stood on the date of
the deceased’s death.

6. Any proceeding which could have been taken against the deceased person
under the Income Tax Ordinance-2001 if the deceased person had not passed away
may be taken against the legal representative of the deceased person.

184
Tariq Hussain Tunio

3. TAXATION OF ASSOCIATION OF PERSONS §.92

1. An association of persons is

▪ liable to tax separately


▪ from the members of the association.

2. Where the association of persons

▪ has paid tax

▪ the amount received by a member of AOP

o in the capacity as member


o out of the income of the association

▪ is exempt from tax in the hands of the members.

If at least one member of the association of persons is a company, the share of


such company or companies shall be excluded for the purpose of computing the
total income of the association of persons and the company or the companies
shall be taxed separately, at the rate applicable to the companies, according to
their share.

Explanation.– For removal of doubt it is clarified that if the income of


association of persons is exempt and no tax is payable under the Ordinance
due to this exemption, the share received in the capacity as member out of
the income of the association shall remain exempt.

4. INDIVIDUAL AS MEMBER OF AOP §.88

1. If a person derives both income from association of persons in the form of share in
the profits (which is exempt) and any other taxable income, the amount of tax
payable on that taxable income is to be computed in accordance with the following
formula:

(A/B) x C
Where:

A= is the amount of tax that would be assessed to the individual for the year if the
share from AOP were chargeable to tax,
B= is the taxable income of the individual for the year if the share from AOP were
chargeable to tax, and
C= is the individual’s actual taxable income for the year.

Explanation for students

Tax payable on taxable income of a member of association of persons is computed


in accordance with the above formula, rather than by applying the tax rates given at
the First Schedule to the Income Tax Ordinance, 2001.

185
CAF-02 Tax Practices

The reason for this is that tax rate computed through this formula yields more tax
than would be payable in case of application of normal tax rates given at the First
Schedule of ITO-2001. Thus, an individual who is a member of AOP is liable to tax at
higher rates than an individual who is not a member of AOP.

5. AUTHORS §.89

1. Where the time taken by an author of a literary or artistic work to complete the work
exceeds twenty-four (24) months, the author may elect to treat any lump sum amount
received by the author in a tax year on account of royalties in respect of the work as
having been received in that tax year and the preceding two tax years in equal
proportions.
2. The amount of royalty received by authors is chargeable to tax under the head
Income from Other Sources.

6. CHANGE IN THE CONSTITUTION OF AN ASSOCIATION OF PERSONS §.98A

Where, during the course of a tax year, a change occurs in the constitution of an
association of persons, liability of filing the return on behalf of the association of
persons for the tax year shall be on the association of persons as constituted at the
time of filing of such return but the income of the association of persons shall be
apportioned among the members who were entitled to receive it and, where the tax
assessed on a member cannot be recovered from him it shall be recovered from the
association of persons as constituted at the time of filing the return.

7. DISCONTINUANCE OF BUSINESS OR DISSOLUTION OF AN ASSOCIATION OF


PERSONS §.98B

Subject to the provisions of section 117, where any business or profession carried on
by an association of persons has been discontinued, or where an association of
persons is dissolved, all the provisions of this Ordinance, shall, so far as may be,
apply as if no such discontinuance or dissolution had taken place.

Every person, who was, at the time of such discontinuance or dissolution, a member
of such association of persons and the legal representative of any such person who
is deceased, shall be jointly and severally liable for the amount of tax payable by the
association of persons.

8. SUCCESSION TO BUSINESS, OTHERWISE THAN ON DEATH §.98C

1.
2. W
Where a person carrying on any business or profession has been succeeded in any
tax year by any other person (hereafter in this section referred to as the
“predecessor” and “successor” respectively), otherwise than on the death of the
predecessor, and the successor continues to carry on that business or profession,-

186
Tariq Hussain Tunio

the predecessor shall be liable to pay tax in respect of the income of the tax
year in which the succession took place upto the date of succession and of
the tax year or years preceding that year; and

the successor shall be liable to pay tax in respect of the income of such tax
year after the date of succession.

Notwithstanding anything contained in sub-section (1), where the predecessor


cannot be found, the tax liability in respect of the tax year in which the succession
took place upto the date of succession and of the tax year or years preceding that
year shall be that of the successor in like manner and to the same extent as it would
have been that of the predecessor, and all the provisions of this Ordinance shall, so
far as may be, apply accordingly.

Where any tax payable under this section in respect of such business or profession
cannot be recovered from the predecessor, it shall be recoverable from the
successor, who shall be entitled to recover it from the predecessor.

187
CAF-02 Tax Practices

PAST EXAMINATION QUESTIONS

Question # 1
Spring 2010 Q.1

Sohail, Khaled and Qazi are members of an association of persons (AOP) and share profit
and loss the ratio of 2:2:1. The principal activity of the AOP is trading of various products.

Following are the details of the annual income / (loss) of the AOP and its members for the
tax year 20X8:

1. The AOP suffered loss before tax amounting to Rs.1,500,000. The loss has been
arrived at after adjusting rental income earned by the AOP, the details of which
are as follows:

RS.
Rental income - 2,000,000
Related Expenses:
Property tax 40,000
Depreciation 475,000 497,500
Net rental Income 1,502,500

No tax was withheld on the rental income or on trading activity.

The expenses debited to profit and loss account include the following amounts paid to the
members of the AOP:
Sohail Khaled Qazi
Salary (Rs.) 900,000 600,000 -
Interest on capital (Rs.) 300,000 300,000 500,000

2. Sohail earned Rs. 1,800,000 from another business, of which he is the sole
proprietor.

3. Khaled received an amount of Rs.255,000 as share of income after tax from


another AOP. He also earned income of Rs.1,900,000 from a sole proprietorship
concern owned by him.

4. Qazi runs a part time business. His gross revenue is Rs.l million whereas total
business expenses are Rs.150,000. He also paid taxes in advance amounting to
Rs.100,000.

Required:

Compute the taxable income and tax liability of the AOP and each of its members. (19)

Question # 2
Spring 2015 Q. 5

a) Under the provisions of the Income Tax Ordinance, 2001 state the rules
relating to residential status of an Association of Person (AOP). Also explain the
taxability of income of AOP, in the hands of the firm and its members. (05)

188
Tariq Hussain Tunio

b) State the rules relating to set-off and carry-forward of losses of AOP and its
members. (02)

Question # 3
Spring 2016 Q. 8

Baqir, Asad and Rahi are members of an association of persons (BAR) and share profits and
losses in the ratio of 5:3:2 respectively. BAR is engaged in the business of trading consumer
electronics and has two independent branches one each in Tehran and Dubai. Following
information has been extracted from BAR’S profit and loss account for the year ended 31
December 2015:

Rupees
Sales 30,000,000
Cost of sales (20,500,000)
Gross profit 9,500,000
Administrative and selling expenses (4,732,000)
Financial charges (980,000)
Other income 1,700,000
Profit before taxation 5,488,000

Additional information:
Cost of sales includes:

a. Closing stock which has been valued at net realizable value of Rs. 1,820,000.
The cost of closing stock under absorption costing was Rs. 1,950,000.

b. Provision of Rs. 75,000 against slow moving stores and spares.

c. Freight charges'of Rs. 260,000. These were paid in cash to Momin Goods
Transport for transporting goods to customers in Multan.

Administrative and selling expenses include:

e. Commission of Rs. 290,000 paid to Baqir, annual performance award of Rs.


310,000 paid to Rahi and Rs. 455.000 paid to AB Bank Limited in final
settlement of a loan obtained by Asad for the construction of his house in
Muree.

f. Provision for bad debts of Rs. 735,000. The opening and closing balances of
provision for bad debts . amounted to. Rs. 1,100,000 and Rs. 1,435,000
respectively. Bad debts written off include a loan of Rs. 285,000 provided to a
supplier.

g. Sales promotion expenses of Rs. 275,000. These expenses were paid by


Rahi through his personal credit card.
h. Rs. 86,000 paid to an institution operated by Federal Government for the
training of industrial workers in Punjab.

Further information:

For the year ended 31 December 2015 Dubai branch made a profit of Rs. 1,500,000 and
Tehran branch made a loss of pis. 1,800,000. These figures are not included in the above
profit and loss account.

189
CAF-02 Tax Practices

Required:
Under, the provisions of the Income Tax Ordinance, 2001 and Rules' made thereunder,
compute the taxable income, net tax payable by. BAR and the amount to be carried forward,
ifany, for tax year 2016. Assume tax and accounting'depreciation is same. (12)

Note: Your computation should commence with the profit before tax figure ofRs.
5,488,000.Show all relevant exemptions, exclusions and disallowances.

Question # 4
Spring 2017 Q. 3(a)

On 1 June 20X6 Dawood and Dewan jointly purchased a bungalow for Rs. 35 million. They
paid the amount in the ratio of65:35 respectively. To arrange funds for the deal, Dawood
borrowed Rs. 3.000,000 in cash from Shameem who is in the business of lending money.
The rate of interest is agreed @ 20% per annum.

On 1 July 20X6, the house was let out to a company at annual rent of Rs. 4,500,000
inclusive of an amount of Rs.75,000 per month for utilities, cleaning and security. For
providing these services Dawood and Dewan paid Rs. 35,000 per month. During the tax
year 20X7 they also paid Rs. 10,000 as collection charges and Rs. 230,000 for administering
the property.

Required:

Compute taxable income tff Dawood and Dewan under appropriate heads of income for the
tax year,20X7. (08)

Question # 5
Spring 2019 Q.1

Mustafa, Ali and Zain are partners of a resident firm in Pakistan, under the name and style
MAZ Enterprises (MAZE) which is engaged in manufacturing and local supply of auto spare
parts. All partners have equal share of profits and losses in the firm.

Following information has been extracted from accounting records of MAZE for the tax
year 20X9:
Rupees
Sales 140,400
Cost of goods sold (91,260)
Gross profit 49,140
Administrative and selling expenses (21,430)
Financial charges (15,740)
(37,170)
Other income 1,900
Profit before tax 13,870

Additional information:

1. The above accounts have been prepared bn cash basis and stock-in-trade has
been Valued on the prime-cost method. However, the partners Want to change

190
Tariq Hussain Tunio

the method of accounting from cash basis to accrual basis. In this respect
following information has been gathered:

Opening Closing
balances balances
Rs. in '000’
Stock-in-trade using prime-cost method 5,200 7,500
Stock-in-trade using absorption-cost method 5,900 8,800

2. Cost of goods sold includes cost of used machinery imported from China on 31
July 2 0X8 amounting to Rs. 2,110,000. The cost includes payment of custom duty
of Rs. 90,000 and income tax of Rs. 110,000 to the Collector of Customs.

3. Administrative and selling expenses include:

• payment of Rs. 380,000 to a local hotel for holding annual eid-milan party for
the employees, key customers.and their families.

• payment of a fixed monthly remuneration of Rs. 150,000 to each partner.

• payment of Rs. 180,000 for purchase of accounting software on 1 January


20X9. The software is expected to be used for fifteen years.

4. Financial charges are net of interest income of Rs. 360,000 (net of tax @ 10%
deducted by the bank), earned by the firm on its savings counts.
Required:

Under the provisions of Income Tax Ordinance, 2001 and Rules made thereunder, compute
the total income, taxable income and tax payable by MAZE using accrual basis of
accounting. (10)

Note: Show all the relevant exemptions, exclusions and disallowances.

Question # 6
Spring 2020 Q.2

Farhan, Kamran and Rehan are memuers of an association of persons (AOP) and share its
profit and loss in the ratio of 2:2:1 respectively.
Following information is available with regard to AOP and its members for the tax year 2020:

• During the year, AOP earned a profit before tax.of Rs. 2,000,000 after making
following payment to its members:

Farhan Kamran Rehan


Rupees
Salary 1,666,666 800,000 600,000
interest on capital 566,666 400,000 300,000

• Kamran is running a business as a sole proprietor from which he earned Rs.


800,000. Kamran is also a member of another AOP where his share of profit or loss
is 60%. During the year, the other AOP incurred a loss after tax of Rs. 350,000 and
paid Rs. 150,000 on account of income tax.

191
CAF-02 Tax Practices

• Rehan received net dividend of Rs. 102,000 from a listed company after deduction of
withholding tax @ 15%.

• Farhan has no other source of income.

Required:

Under the provisions of the Income Tax Ordinance, 2001 compute taxable income and tax
liability of AOP and each of its members for the tax year 2020. (11)

Question # 7
Autumn 2020 Q. 2

Libas & Co. is an association of persons (AOP) with three members, Saba, Junaid and
Akram, sharing profit and loss in the ratio of 1:1:2 respectively.

During the year, AOP earned profit before tax of Rs. 8,500,000 from its principal business
i.e. trading of garments. In addition, AOP is also involved in purchase and sale of following
securities listed on the Pakistan Stock Exchange:

Note A: Sale proceed from disposal of these shares was credited to the AOP's bank account

Details of purchase Details of sale


Name of Price
investee Date No. of Price per Date No. of per
company shares share sharesshare
(Rs.) (Rs.)
XOK 1 Oct 2016 200,000 200 29 June 2020 200,000 225
Limited [Note A1]
PBB 18 Aug 2017 55,000 145 20 Dec 2019 100,000 180
Limited 10 Jan 2018 100,000 150
OOI 15 Feb 2020 150,000 86 15 March 150,000 78
Limited [Note B] 2020
on 2 July 2020.

Note B: Due to shortage of funds for making this purchase, AOP borrowed Rs.
5,000,000 in cash from Imran, who is in the business of lending money at 15% per
annum.

Other information related to Saba:

During the year, she earned Rs. 1,500,000 by working as a freelance photographer.

On 1 April 2020, Saba received Rs. 1,100,000 from Zafar in full settlement of a loan.
The loan was provided on 1 April 2019 at 10% per annum interest through proper
banking channel.
Required:
Under the provisions of the Income Tax Ordinance, 2001, compute taxable income
and tax liability of AOP and Saba for the tax year 2020. (13)

192
Tariq Hussain Tunio

Question # 8
Autumn 2021 Q.5

Kamkaj & Co. is an association of persons (AOP) with three members namely Baqir, Omer
and Sadabahar(Pvt) Limited (SPL), sharing.profit and loss in the ratio of 20:30:50
respectively.

Following information is available with regard to AOP and its members for the tax years 2020
and 2021:

(i) AOP's income for tax years 2020 and 2021:

2020 2021
Rupees
Income from business* (18,000,000) 25,000,000
Dividend income - 4,000,000

*Net of annual fixed commission of Rs. 7,000,000 to SPL

(ii) On 1 February 2021, Baqir earned capital gain of Rs. 5,200,000 on sale of his
property which.was purchased on 1 January 2018.

(iii) Omer also operates a sole proprietor business from which he earned profits of Rs.
6,000,000 and Rs. 2,500,000 in tax years 2020 and 2021 respectively.

Required:

Under the provisions of the Income Tax Ordinance, 2001 compute the following for the tax
years 2020 and 2021:
• Taxable ineome of AOP
• Taxable income and tax liability of Baqir and Omer (10)

193
CAF-02 Tax Practices

Chapter

14

RETURNS
CHAPTER SYNOPSIS
TOPIC SECTION
Return of income 114
Business Bank Account 114A
Powers to enforce filing of
114B
returns
Persons not required to file
115
return
Wealth statement 116
Foreign Income and Assets
116A
Statements
Notice of discontinued business 117
Due date of filing of return &
118
other documents
Extention of time 119
ICAP Past Papers Questions

194
Tariq Hussain Tunio

1. RETURN OF INCOME §.114

Persons Liable to File a Return

1. Persons Required to Furnish a Return for a Tax Year –§114(1)

(i) Every Company


(ii) Every person (other than a company) whose taxable income for the year
exceeds the maximum amount that is not chargeable to tax under the Income
Tax Ordinance, 2001, for the year.
(iii) Any Non-profit Organization (NPO)
(iv) Every person whose income for the year is subject to final taxation
(v) Any person who has been charged to tax in any of the two preceding tax
years,
(vi) Any person who claims a loss carried forward under the Ordinance for a tax
year,
(vii) Any person who owns: (a) immovable property with a land area of 500 square
yards or more, or (b) owns any flat, located in areas falling within the
municipal limits existing immediately before the commencement of Local
Government laws in the provinces; or areas in a Cantonment; or the
Islamabad Capital Territory
(viii) Any person who owns immoveable property with a land area of five hundred
square yards or more located in a rating area
(ix) Any person who owns a flat having covered area of two thousand square feet
or more located in a rating area;
(x) Any person who owns a motor vehicle having engine capacity above 1000
CC;
(xi) Any person who has obtained National Tax Number.
(xii) Any person who is the holder of commercial or industrial connection of
electricity where the amount of annual bill exceeds Rs. 500,000/; or
(xiii) Any person is registered with any chamber of commerce and industry or any
trade or business association or any market committee or any professional
body including Pakistan Engineering Council, Pakistan Medical and Dental
Council, Pakistan Bar Council or any Provincial Bar Council, Institute of
Chartered Accountants of Pakistan or Institute of Cost and Management
Accountants of Pakistan.
(xiv) Resident person being an individual required to file foreign income and assets
statement
(xv) Persons or classes of persons notified by the Board with the approval of the
Minister in-charge.

NOTE:
Every individual whose income under the head ‘Income from business’
exceeds rupees three hundred thousand but does not exceed rupees four
hundred thousand in a tax year is also required to furnish return of income
from the tax year

Features of Complete Return of Income –§114(2)

2. A return of income must fulfill all of the following conditions/ requirements:

▪ prescribed form and accompanied by all prescribed annexures, statements or


documents;

195
CAF-02 Tax Practices

▪ state all the relevant particulars or information as specified in the form of return,
including a declaration of the records kept by the taxpayer;
▪ signed by the person when the person is an individual, or it shall be signed by the
person’s representative wherever the provisions of Representative are applicable
(as per §172);
▪ evidence of payment of due tax as per return of income (e.g. Challan or CPR i.e.
Computerized Payment Receipt); and
▪ wealth statement (as required under section 116)
▪ accompanied with a foreign income and assets statement (as required under
section 116A).

Electronic Filing of Return –§114(2A)

3. A return of income filed electronically on the web or any magnetic media or any other
computer readable media as may be specified by the Board shall also be deemed to
be a return for the purpose of ITO-2001.

4. The Board may, by notification in the official Gazette, make rules for

▪ determining eligibility of the data of such returns and


▪ e-intermediaries who will digitise the data of such returns and transmit the
same electronically to the Income Tax Department under their digital
signatures and
▪ other matters relating to electronic filing of returns, statements or documents,
etc. §114(2A)

Filing a Return for a Period of Less than 12 Months –§114(3)

5. The Commissioner is authorized to require a person, or his representative, to furnish


a return of income through a notice for a period of less than twelve months in case of
following conditions:

▪ the person has passed away,


▪ the person has become bankrupt or gone into liquidation,
▪ the person is about to leave Pakistan permanently,
▪ the Commissioner otherwise considers it appropriate to require such a return to
be furnished

The due date for filling of return these cases is to be mentioned on the notice.

Notice of Filing of Return to Non-Filers of Return –§114(4) & (5)

6. Where in a tax year a person is required, in the opinion of a Commissioner, to


file a return of income but who has failed to do so, the Commissioner may issue a
notice in writing to the non-filer of return requiring him to file a return of income for a
tax year.

i) Notice for more than one Tax Years

7. A taxpayer who has failed to discharge his legal obligation of filing a return for
the past several tax years may also be required to file returns for those tax years.
However, the notice for filing of return for more than one tax year may be issued in
respect of only last five completed tax years.

196
Tariq Hussain Tunio

Note:
Also, it is not necessary to issue different notices for different tax years; a person
may be required to file a return for more than one tax year through a single notice.

8. in case of a person who has not filed return for any of the last five completed tax
years, notice under sub-section (4) may be issued in respect of one or more of the
last ten completed tax years.

9. the time-limitation shall not apply if the Commissioner is satisfied on the basis of
reasons to be recorded in writing that a person who failed to furnish his return has
foreign income or owns foreign assets.

ii) Time Period for Filing Return in Response to Notice

10. Return in response to this notice is required to be furnished within thirty days from
the date of service of such notice or within such longer or shorter period as may be
specified in such notice or within the time as the Commissioner may allow.

Revision of a Return –§114(6) & (6A)

11. A return may be revised by a taxpayer in case of discovery of any omission or wrong
statement in it, subject to the following conditions:

(a) it is accompanied by the revised accounts or revised audited


accounts;

Commissioner may waive this condition if the Commissioner is


satisfied that filing of revised accounts or audited accounts is not
necessary

(b) the reasons for revision of return, in writing, duly signed, by the
taxpayers are filed with the return;

(c) it is accompanied by approval of the Commissioner in writing for


revision of return; and

this condition is not applicable if revised return is filed within 60 days


of filing of return.

(d) taxable income declared is not less than and loss declared is not more
than income or loss determined by an order.

12. If any of these conditions is not fulfilled, the return furnished shall be treated as an
invalid return as if it had not been furnished.

13. condition in above point c shall not apply and the approval required thereunder shall
be deemed to have been granted by the Commissioner, if-

a. the Commissioner has not made an order of approval in writing, for


revision of return, before the expiration of sixty days from the date when
the revision of return was sought; or

b. taxable income declared is more than or the loss declared is less than the
income or loss, as the case may be, determined under section 120.

197
CAF-02 Tax Practices

14. the Commissioner shall grant approval in case of a bonafide omission or wrong
statement

(i) Penalties/Default Surcharge in Case of Revision of Return

15. If a taxpayer files a revised return voluntarily along with deposit of the amount of tax
short paid or amount of tax sought to be evaded along with the default surcharge,
whenever it comes to his notice before receipt of audit notice or show cause notice
before amendment, no penalty shall be recovered from him.

16. In case the taxpayer deposits the amount of tax as pointed out by the Commissioner
during the audit or before the issuance of show cause notice (u/s.122(9)), he shall
deposit the amount of tax sought to be evaded, the default surcharge and twenty-five
percent of the penalties leviable under ITO-2001 along with the revised return

17. In case the taxpayer revises the return after the issuance of a show cause notice
(u/s.122(9)), he shall deposit the amount of tax sought to be evaded, default
surcharge and fifty percent of the leviable penalties under ITO-2001 along with the
revised return and thereafter, the show cause notice shall stand abated.

Chart# 2► REVISION OF RETURN & REDUCTION IN PENALTY

REVISION OF
RETURN

A return may be revised in case of discovery of any omission or


Penalties
wrong statement in it, subject to the following conditions that it is

S Penalty/
Situation
# Default Surcharge
accompanied accompanied 1 Revision of return No penalty is imposed if the
with reasons accompanied by taxable income
by the revised by taxpayer taxpayer deposits the amount of
for revision of approval of CIR declared is not
accounts or before issuance of tax short paid or sought to be
return in in writing for < and loss
revised audited audit notice. evaded along with default
writing, duly revision of declared is not
accounts surcharge.
signed by the return > income or
2 Revision of return Penalty equal to 25% of the
taxpayers loss
by taxpayer amount of actual leviable
determined by
before issuance of penalty is imposed if the
an assessment
showcase notice taxpayer deposits the amount of
order
for amendment of tax sought to be evaded along
assessment. with default surcharge.
3 Revision of return Penalty equal to 50% of the
by taxpayer after amount of actual leviable
issuance of a penalty is imposed if taxpayer
show cause deposits the amount of tax
notice for sought to be evaded along with
amendment of default surcharge.
assessment.

Is Purported Return a Valid Return?

18. Every return purporting to be made or signed by a taxpayer, or by an


authorized representative on behalf of a person, is treated as having been duly made
by the person or with the person’s authority until the person proves the contrary.

198
Tariq Hussain Tunio

19. The burden of proof for proving that the return does not belong to a taxpayer and that
it was filed without his authority is on the taxpayer.

ILLUSTRATION

Q: Suppose a return for one Mr. X is submitted for Tax Year 2010 to the tax department on
30th September 2010. The return declares taxable income at Rs. 8 (M), whereas previously
Mr. X has been declaring taxable income around Rs. 400,000/ - for many years. When
contacted by the tax department, Mr. X states that he has not filed the said return. Will this
return be considered as valid by the tax department?

Ans: Yes, unless Mr. X proves that the return was not submitted by him or that it was made by
someone else without his authority.

2. BUSINESS BANK ACCOUNT §.114A

1. Every taxpayer shall declare to the Commissioner the bank account utilized by the
taxpayer for business transactions.

2. Business bank account shall be declared through original or modified registration


form prescribed under section 181.

3. POWERS TO ENFORCE FILING OF RETURNS §.114B

• Notwithstanding anything contained in any other law for the time being in
force, the Board shall have the powers to issue income tax general order in
respect of persons who are not appearing on active taxpayers’ list but are
liable to file return under the provisions of the Ordinance.

• The income tax general order issued under sub-section (1) may entail any or
all of the following consequences for the persons mentioned therein, namely:–

(a) disabling of mobile phones or mobile phone SIMS;

(b) discontinuance of electricity connection; or

(c) discontinuance of gas connection.

• The Board or the Commissioner having jurisdiction over the person mentioned
in the income tax general order may order restoration of mobile phones,
mobile phone SIMS and connections of electricity and gas, in cases where he
is satisfied that —

(a) the return has been filed; or

(b) person was not liable to file return under the provisions of the Ordinance.

• No person shall be included in the general order under sub-section (1) unless
following conditions have been met with, namely:–

a. notice under sub-section (4) of section 114 has been issued;

199
CAF-02 Tax Practices

b. date of compliance of the notice under sub-section (4) of section 114 has
elapsed; and

c. the person has not filed the return.

• The action under this section shall not preclude any other action provided
under the provisions of the Ordinance.

4. PERSONS NOT REQUIRED TO FILE A RETURN §.115

1. A person who is required to file a return solely because of the ownership of


immovable property with a land area of 500 sq yards or more, or a flat (as stated
above) is not required to file a return in case he/she is:

i. A widow
ii. An orphan below the age of 25 years
iii. A disabled person
iv. in the case of ownership of immovable property, a Non-resident person

5. WEALTH STATEMENT §.116

1. Following particulars are required to be furnished on the wealth statement:

▪ the person’s total assets and liabilities on a specified date or dates,

▪ the total assets and liabilities of the person’s spouse, minor children, and other
dependents on a specified date or dates,

▪ any assets transferred by the person to any other person during the period and
the consideration for the transfer,

▪ the total expenditures incurred by the person, and the person’s spouse, minor
children, and other dependents during the period or periods specified, and the
details of such expenditures, and

▪ the reconciliation statement of wealth.

Persons Required to File Wealth Statement

2. Following persons are required to file wealth statement:

(i) Any person, being an individual, is required by the Commissioner by a


notice to file Wealth Statement

(ii) Every resident taxpayer being an individual filing a return of income for
any tax year shall furnish a wealth statement and wealth reconciliation
statement for that year along with such return

(iii) Every member of an association of persons shall also furnish wealth


statement and wealth reconciliation statement for the year along with return of
income of the association.

200
Tariq Hussain Tunio

Revision of Wealth Statement

3. In case a person, who has furnished a wealth statement, discovers any omission
or wrong statement in the wealth statement, he may:

• without prejudice to any liability incurred by him under any provision of this
Ordinance,

• furnish a revised wealth statement along with the revised wealth reconciliation
and the reasons for filing revised wealth statement

under intimation to the Commissioner in the prescribed form and manner,at any time before
the receipt of notice under sub-section (9) of section 122, for the tax year to which it relates.

Where the Commissioner is of the opinion that the revision under this sub-section is not for
the purpose of correcting a bona fide omission or wrong statement, he may declare such
revision as void through an order in writing after providing an opportunity of being heard.

Explanation.- For the removal of doubt it is clarified that wealth statement cannot be
revised after the expiry of five years from the due date of filing of return of income
for that tax year.

6. FOREIGN INCOME AND ASSETS STATEMNET §.116A

1. Every resident taxpayer being an individual having foreign income of not less than
ten thousand United States dollars or having foreign assets with a value of not
less than one hundred thousand United States dollars shall furnish a statement,
hereinafter referred to as the foreign income and assets statement, in the
prescribed form and verified in the prescribed manner giving particulars of—

a) the person’s total foreign assets and liabilities as on the last day of the tax
year;

b) any foreign assets transferred by the person to any other person during the
tax year and the consideration for the said transfer; and

c) complete particulars of foreign income, the expenditure derived during the


tax year and the expenditure wholly and necessarily for the purposes of
deriving the said income.

2. The Commissioner may by a notice in writing require any person being an


individual who, in the opinion of the Commissioner on the basis of reasons to be
recorded in writing, was required to furnish a foreign income and assets statement
under sub-section (1) but who has failed to do so to furnish the foreign income and
assets statement on the date specified in the notice.

7. NOTICE OF DISCONTINUED BUSINESS §.117

1. Any person discontinuing a business is required to give a notice to that effect to the
Commissioner within fifteen days of the discontinuance.

201
CAF-02 Tax Practices

2. The purpose of this notice is to bring the fact of discontinuance to the knowledge of
the Commissioner, who may then require the person to file a return for the period
during which the business was carried on during the year before it discontinuance.

Filing of Return

3. The person discontinuing a business is required to furnish a return of income in


response to the notice by Commissioner, or as required under the other provisions of
the Ordinance, as the case may be.

4. The period for filing of this return is treated as a ‘separate tax year’. This separate tax
year spans over a period commencing on the first day of the tax year in which the
discontinuance occurred and ending on the date of discontinuance.

5. A return furnished in this manner is treated for all purposes of this Ordinance as a
return of income, including the provisions of application self-assessment.

What if the Person Fail to Give Notice

2. If a taxpayer fails to give the notice of discontinuance of business to the


Commissioner and the Commissioner has reasonable grounds to believe that the
business has discontinued or is likely to discontinue, the Commissioner may serve a
notice on the person to furnish the aforesaid return of income within the time
specified in the notice.

202
Tariq Hussain Tunio

8. DUE DATE OF FILING OF RETURN /OTHER DOCUMENTS §.118

1. The due dates of filing of return and other documents are given in the following table.

S.# Particulars Due Date


1 Return to be filed by Company whose:
- T.Y ends b/w 1 Jan to 30 June 31st December next following the
end of the tax year to which the
return relates.
- T.Y ends b/w 1 July to31 Dec 30th September next following the
end of the tax year to which the
return relates.
Salaried individual 30th September next following the
2 end of the tax year to which the
return relates.
Any person otherthan company 30th September next following the
3 end of the tax year to which the
return relates.
4 Wealth Statement:
- filed mandatorily Date of furnishing of return
- filed under notice by CIR. Date specified on the notice
Return in case of discontinuance of Date specified on the notice
5
business

2. Where salary income for the tax year is Rs. 500,000/- or more, the taxpayer shall

▪ file return of income electronically in the prescribed form and

▪ it shall be accompanied by the proof of deduction or payment of tax and

▪ wealth statement (as required u/s116) or a foreign income and asset


statement u/s 116A.

the board may amend the condition specified in this sub-section or direct that the
said condition shall not apply for a tax year.

3. If a taxpayer is not borne on the National Tax Number Register and fails to file an
application for allotment of NTN in the prescribed form and manner with the
taxpayer’s return of income, such return is not be treated as a return furnished under
the ITO-2001.

9. EXTENTIN OF TIME FOR OF RETURN/OTHER DOCUMENTS §.119

Extension of Time for Filing of Return & Other Documents

1. Returns and other documents are required to be furnished to the Commissioner


within the above prescribed time limits.

2. If a person is unable to meet the deadline stipulated for the filing of return or other
documents, he may apply to the Commissioner, before the expiry of due date, for
extension of the date for filing of the same.

3. Extension of time is available for filing of following documents:

203
CAF-02 Tax Practices

▪ Return of Income
▪ Return in case of Discountinued of business
▪ Wealth Statement

Any person who is required to file above may apply in writing to CIR for extension of
time.

Grounds for Granting Extension of Time

4. When an application for extension of date has been received by the Commissioner,
he may grant an extension of time for furnishing the return or other document if he is
satisfied that the taxpayer is unable to furnish the return of income or other document
by the due date because of:

▪ Absence from Pakistan

▪ Sickness or other misadventure

▪ Any other reasonable cause

Maximum Period of Extension of Time

5. The extension of time granted by Commissioner should not exceed fifteen days. It
may, however, exceed the period longer than fifteen days is only allowed in case of
‘exceptional circumstance’ like floods, earthquake, fire, curfew, etc.

The Chief Commissioner may on an application made by the taxpayer for extension
or further extension, as the case may be, grant extension or further extension for a
period not exceeding fifteen days unless there are exceptional circumstances
justifying a longer extension of time.

Extension and Payment of Tax

6. The due date for ‘payment of tax’ is the same as the due date for ‘filing of return’, as
payment of tax is to be made on or before the date of filing of return.

7. An extension of time granted for filing of return, however, does not apply to the
payment of due tax. If a person who has received extension of time for falling of
return does not pay tax on the original date of payment, he is liable for punitive action
in the form of default surcharge for late payment.

ILLUSTRATION

Q: Mr. Rameez was required to file a return for Tax Year 2009 on 30.09.2009. The tax
payable for the year along with return was Rs. 25,000/-. Extension for filing of return for 10
days was granted to Mr. Rameez, for he was outside Pakistan at that time. Thinking that the
date for payment of tax is also extended, Mr. Rameez made payment of tax on 10.10.2010.
What are the consequences?

Ans: Mr. Rameez’s contention is wrong. The extension of time was only granted for filing of
return; it did not relate to the payment of tax. Hence, default surcharge will be impos

204
Tariq Hussain Tunio

PAST EXAMINATION QUESTIONS

Question # 1
Spring 2015 Q. 2

(a) List the persons who are required to furnish a return of income for a tax year under
the Income Tax Ordinance, 2001. (06)

(b) Specify the circumstances under which the Commissioner has powers to issue
notice demanding a return of income from certain person(s) for less than one year.
(03)

(c) State the powers of the Commissioner if a taxpayer fails to furnish return as
required under part (b) above, within the specified time. (04)

Question # 2
Autumn 2016 Q. 3(c)

List the persons who may be granted immunity from filing of tax return u/s 114 of the Income
Tax Ordinance, 2001 solely by reason of owning immovable property with a land area of two
hundred and fifty square yards or more or any flat located in areas falling within the
municipal limits. (03)

Question # 3
Spring 2017 Q.5

(a) List the persons who are required to file a tax return under the provisions of the
Income Tax Ordinance, 2001. (06)
(b) In the light of the provisions of the Income Tax Ordinance, 2001:
(i) Identify the circumstances under which the Commissioner of Income Tax may
require a person to furnish a return of income for a period of less than twelve
months. (03)
(ii) State the consequences if a person fails to furnish the return as required in (i)
above. (03)

Question # 4
Spring 2017 Q.6

Zahid, the sole proprietor of FG and company, is a resident individual and is in the process
of filing his wealth statement for the tax year 20X7. The relevant information is as under:

(i) Assets and liabilities disclosed in the wealth statement for the tax year 20X6
were as follows:

Assets Rupees
Agriculture land in Hyderabad 5,000,000
Residential property in DHA Karachi 3,000,000
Investments shares of listed companies 1,100,000
Business capital – FG & Co. 4,000,000
Motor vehicle 1,540,000
Cash at bank 600,000

205
CAF-02 Tax Practices

Cash in hand 300,000


15,540,000
Liabilities
Bank loan (1,500,000)
Net assets 14,040,000

(ii) Details relating to FG & Co. are as follows:

Rupees
Income from business for the tax year 20X7 2,540,000
Drawings during the year 450,000

(iii) Balance of cash in hand and at bank, as on 30 June 20X7 amounted to Rs.
157,500 and Rs. 730,000 respectively.

(iv) Transactions carried out by Zahid during the year were as follows:

o Paid an advance ofRs. 1,000,000 against purchase of a bungalow for Rs.


10,000,000.

o Sold shares of a listed company for Rs. 350,000. The shares were purchased
on 1 May 20X6 for Rs. 50,000. Capital gain tax collected by NCCPL
amounted to Rs. 37,500.

o Gifted shares of a listed company to his brother. The shares were purchased
by Zahid in 20X2 at a cost ofRs. 100,000 whereas market value of the shares
at the time of gift was Rs. 150,000.

o Paid Rs. 200,000 towards principal amount of the bank loan.

o Personal expenses amounted to Rs. 2,075,000.

o Net receipts against agricultural income amounted to Rs. 2,500,000.

Required:

Prepare Zahid's wealth statement and wealth reconciliation statement for the tax year
20X7. (07)

Questton # 5
Spring 2018 Q. 4(b)

Identify due date of filing of tax return in each of the following cases, under the
provisions of the Income Tax Ordinance, 2001:
(i) An individual who’s entire income falls under final tax regime (0.5)
(ii) An individual who derives his income from business which falls under normal
tax regime. (0.5)
(iii) An individual filing return in response to a notice received from the
Commissioner who believes that he is likely to discontinue his business. (01)
(iv) An individual filing return in response to a notice received from the
Commissioner for not filing return of income of the previous tax year. (01)

206
Tariq Hussain Tunio

(v) A company. (01)

Question # 6
Spring 2019 Q. 3(a)

Imran, a resident person, is filing the return of his business income for the first time.

He has been informed by his friend that he will also be required to file a wealth statement In
this respect, he seeks your advice about the particulars which he should disclose in his
wealth statement (04)

Question # 7
Spring 2020 Q. 3(b)

Under the provisions of the Income Tax Ordinance, 2001 and Rules made thereunder,
discuss:

(b) Who is required to file the foreign income’ and assets statement? Also state the
particulars to be included in such statement (05)

Question # 8
Autumn 2021Q. 2(a)

a) Mukhtar, a resident individual, is in process of finalization of his wealth statement for the
tax year 2021. He has provided you the following information:

(i) During the tax year 2021, Mukhtar received share of profit of Rs. 1,400,000 from
an AOP As on 30 June 2020, his total investment in the AOP was Rs.
5,300,000. He. was also provided a car worth Rs. 2,500,000 by the AOP for
office use only.

(ii) In 2014, he had purchased 10 tola gold for Rs. 500,000. At 30 June 2021, the
market value of the gold was Rs. 107,000 per tola.

(iii) During the tax year 2021, he sold his personal car for Rs. 1,876,000. The car
was purchased in 2019 for Rs. 1,700,000.

(iv) During the tax year 2021, he paid Rs. 600,000 against outstanding interest free
loan of Rs. 1,000,000. The loan was obtained in tax year 2020.

Required:

Under the provisions of the Income Tax Ordinance, 2001 advise Mukhtar that how the above
matters would be dealt with in his wealth statement and its reconciliation for the tax year
2021. (04)

Question # 9
Autumn 2021 Q. 3(b)

207
CAF-02 Tax Practices

(c) Aoun has discovered an error in his annual income tax return which was submitted
on the due date. Now he intends to file a revised return voluntarily.
Required:

Under the provisions of Income Tax Ordinance, 2001 state the conditions which
Aoun must comply with for filing valid revised return. (04)

208
Tariq Hussain Tunio

Chapter

15

ASSESSMENT
CHAPTER SYNOPSIS
TOPIC SECTION
Introduction General
Assessment 120
Best judgment assessment 121
Amendment of assessment 122
Revision by the Commissioner 122A
Revision by the Regional Commissioner 122B
Agreed Assessment in certain cases 122D
Provisional assessment in certain cases 123
Assessment giving effect to an order 124
Powers of tax authorities to modify 124A
orders
Assessment in relation to disputed 125
property
Evidence of assessment 126
Restriction of proceeding 120B
ICAP Past Papers Questions

209
CAF-02 Tax Practices

1. INTRODUCTION

Definition of Assessment

“Assessment” includes 'provisional assessment', 're-assessment' and 'amended


assessment' and the cognate expressions shall be construed accordingly. §.2(5)

Explanation for Students


Assessment is a process through which taxable income and tax liability of a
person are determined every year. The above definition of ITO 2001 is a
limited purpose definition, as, being inclusive in type, it merely includes
‘provisional assessment’, 're-assessment' and 'amended assessment' and
related expressions in ambit of the term ‘assessment’. This is done for clarity
and removal of doubts. Dictionary meaning of ‘assessment’ also applies.

Assessment = Dictionary Meaning of Assessment + (Provisional Assessment,


Re-assessment' and Amended assessment and cognate expressions)

2. ASSESSMENT §.120

What is Assessment?

Where a taxpayer has filed a complete return (other than revised return) for any tax
year ending on or after July 01, 2002, the taxable income and tax liability of the
person is deemed to be assessed in the following manner:

a. CIR is deemed to have made an assessment of taxable income for TY, and tax
due thereon.

b. Return shall be taken for all purposes of ITO 2001 to be an assessment order
issued to taxpayer by CIR on the day return was furnished.

Explanation for Students


Following are key points of assessment:
▪ Assessment is also referred to as ‘self-assessment’.
▪ Assessment is deemed to be done by Commissioner.
▪ Results of deemed assessment are finalization of taxable income and tax
due by CIR equal to amounts declared on the Return by the taxpayer
(except when adjusted by Online Automated System).
▪ Return becomes a ‘deemed Assessment Order’ issued by the
Commissioner to the taxpayer on the day it was furnished (no separate
‘assessment order’ is passed).
▪ This self-assessment system is applicable for all tax years from tax year
2003 and onwards.

210
Tariq Hussain Tunio

Audit Under Self-assesment System

CIR may conduct audit of the income tax affairs of a person and all provisions of
section 177 shall apply accordingly. (Section 177 deals with audit of income tax
affairs of taxpayers.)

Explanation for Students


Self-assessment is not a blank cheque to taxpayers for tax evasion. Effective
methods of check and balance on self-assessment have to be kept in place so
that due taxes may be collected from the taxpayers. It is due to this that the
Commissioner is authorized to conduct audit of taxpayers’ income tax affairs.

Self-assessment does not prevent the Commissioner from conducting audit and
re-computing taxpayer’s taxable income and tax liability through amendment of
assessment if the taxpayer has not worked out his tax liability in accordance with
Income Tax Ordinance, 2001.

Completeness of Return – A Perquisite for Self-Assessment

The provisions of self-assessment apply to a taxpayer only when he has filed a


complete return of income.

When is a Return Treated as Complete?

As has already by described at Para 18.1.5, a return of income becomes complete


when it fulfills the following conditions:

A return of income shall:

▪ be in the prescribed form and shall be accompanied by prescribed annexures,


statements or documents,
▪ state all the relevant particulars or information as specified in the form of return,
including a declaration of the records kept by the taxpayer,
▪ be signed by the individual, or his authorized representative.
▪ be accompanied with evidence of payment of due tax as per return of income
▪ be accompanied with a wealth statement as required under section, and
▪ be accompanied with a foreign income and assets statement.

Notice of Deficiencies in case of Incomplete Return –§.120(3)

If a taxpayer has filed an incomplete return of income, the Commissioner issues a notice to
the taxpayer informing him of the deficiencies in his return and directing him to provide the
missing information, particulars, statement or documents.

The taxpayer is required to complete the return by removing the deficiencies pointed out by
the Commissioner on or before the date specified by the Commissioner in the notice.

Deficiencies to be Pointed out by CIR in the Notice

The Commissioner is authorized to point out through the notice of deficiencies only those
deficiencies which make the return incomplete.

211
CAF-02 Tax Practices

He is not allowed to point out any deficiencies relating to incorrect amount of tax
payable on taxable income or short payment of tax payable through this notice,
because in that case the sanctity of self-assessment will be eroded.

The Commissioner, however, using powers of amendment of assessment correct


these deficiencies.

Response by Taxpayer to Notice of Deficiencies

Full Compliance by the Taxpayer: If the taxpayer complies fully with the requirements of
the notice and all the missing information, particulars, statement or documents are
provided to the Commissioner by the due date specified on the notice of deficiencies,
the return is treated as complete on the day it was originally furnished. Moreover, the
provisions of universal self-assessment shall also apply to the return completed in
this manner.

Partial or No Compliance by the Taxpayer: If the taxpayer does not comply with the
requirements of the notice or comply partially by the due date specified on the notice,
the return furnished by the taxpayer is treated as an invalid return, meaning thereby
that the return was not filed in the first place.

Time Limits for Issuance of Notice Deficiencies

A notice of deficiencies may be issued by the Commissioner before the end of the
financial year in which return was furnished. If no notice is issued in this period, the
return is covered by the provisions of universal self-assessment.

Notice of deficiencies shall not be issued after the expiry of 180 days from the end of
financial year in which return was furnished.

212
Tariq Hussain Tunio

Chart# 1► ASSESSMENT §.120

RETURN
(Other than Revised
Return)

Complete Return of
Incomplete
income is furnished for TY Return is not
ending on/before furnished informing him of
30.06.2002 the deficiencies in
his return
taxable income and tax
liability of the person is
deemed to be assessed CIR issues NOTICE
by CIR (within 180 days of end of
Deeming of FY in which return was
furnished) directing him to
Assessment provide the
missing
information,
Return is treated as an particulars,
assessment order issued statement or
to taxpayer by CIR on the Compliance by documents, except
day it was furnished incorrect amount
CIR may conduct audit Taxpayer of tax payable on
of income tax affairs of taxable income or
the taxpayer short payment of
tax payable

Return is Full Non/Partial


treated as Compliance by Compliance by
complete on Taxpayer Taxpayer
the day it was
filed.

Return is
treated as
invalid, as if
not filed.

213
CAF-02 Tax Practices

3. BEST JUDGMENT ASSESSMENT §.121

Conditions in Which Best Judgment Assessment is Done?

1. The Commissioner is authorized to make best judgment assessment (also called ‘ex-
parte’ assessment) of the taxable income and the tax due thereon where a person
failes to:

(i) file return of income in response to notice [issued by CIR to non-filers for filing the
return]
(ii) file return [in case of a non-resident ship and aircraft owner or charterer]
(iii) file wealth statemtn
(iv) produce books of accounts, documents and records maintained by taxpayer
under ITO-2001, or any other relevant document or evidence required to be
produced, during the course of audit, for the purpose of making assessment of
his income and determination of tax due thereon, before CIR or any person
employed by firm of chartered accountants or cost & management accountants
appointed by FBR to conduct audit of taxpayer.

How is the Best Judgment Assessment Done?

▪ The Commissioner is required to make best judgment assessment on the basis of 


any available information or material and  to the best of his judgment. Taxable
income and tax due thereon is determined by the Commissioner by passing ‘best
judgment assessment order’.

▪ Best judgment assessment order is required to state the following particulars:


o the taxable income
o the amount of tax due
o the amount of tax paid, if any, and
o the time, place and manner of appealing the assessment order.

Time Limits for Best Judgment Assessment

The best judgment assessment order may be passed within six years after the end of
the tax year to which it relates.

ILLUSTRATION

Q. Mr. Javaid failed to file his wealth statement for the Tax Year 2015. He received a notice
for best judgment assessment for the said tax year on July 20, 2023. Being his tax consultant,
you are required to advise him whether or not best judgment assessment in his case can be
done.

Ans. Best judgment assessment for Tax Year 2015 can be done only before 30.06.2021. Since,
the time limit of six years from the end of the tax year to which it relates has expired, best
judgment assessment in this case cannot be done.

214
Tariq Hussain Tunio

Chart# 3► BEST JUDGEMENT ASSESSMENT §.121

If a person
fails to

furnish a return in furnish a return furnish wealth produce accounts, documents & records
response to notice before CIR or CA/ICMA firm appointed by
(aircraft/ship owner or charterer) statement FBR for tax audit

CIR makes best judgment based on


assessment of taxable ▪ available information/
income of the person material and
▪ to the best of his judgment
stating:
▪ the taxable income;
▪ the amount of tax due; CIR issues
▪ the amount of tax paid, if any; best judgment assessment
order
and
▪ the time, place and manner of
appealing the assessment order

Best judgment assessment order


shall only be issued within 6
years after the end of the tax
year or the income year to
which it relates.

4. AMENDEDMENT OF ASSESSMENT §.122

Amendment of Assessment

By making ‘alteration’ or ‘addition’, commissioner may amend following orginal


assessment orders’:

▪ [Self] Assessment Order (u/s 120)


▪ Best Judgment Assessment Order (u/s 121)

Time Limits for Amendment

No assessment order shall be amended by Commissioner

▪ after expiry of five years


▪ from the end of financial year
▪ in which Commissioner has issued or treated to have issued assessment
order to taxpayer

215
CAF-02 Tax Practices

Revised Return is Deemed Amended Assessment Order

A revised return is treated as deemed ‘amended assessment order’ issued to the


taxpayer by the Commissioner. On filing a revised return by the taxpayer, the
Commissioner is treated as having made an amended assessment of the taxable
income and tax payable thereon equal to the amounts specified on the revised
return.

Further Amendment §122(4)

In case an original assessment order has been amended by the Commissioner or it


has been treated as amended by Commissioner through filing of a revised return by
the taxpayer, the Commissioner is authorized to further amend it as many times as
necessary.

Time Limits for Further Amendments

An original assessment order may be amended as many times as necessary within


the later of:

(i) five years from the end of the financial year in which the Commissioner has
issued or is treated as having issued the original assessment order to the
taxpayer; or

(ii) one year from the end of the financial year in which the Commissioner has
issued or is treated as having issued the amended assessment order to the
taxpayer.

Pre-Conditions for Amendment and Further Amendment

An assessment order may be amended and an amended assessment may be further


amended, only on the basis of audit or definite information, when the Commissioner
is satisfied that:

i) Any income chargeable to tax has escaped assessment


ii) Total income has been underassessed
iii) Total income has been assessed at too low a rate
iv) Total income has been subject to excessive relief
v) Total income has been subject to excessive refund
vi) Any amount under a head of income has been misclassified

What is Definite Information?

For the purposes of amendment of assessment, “definite information” includes


information on

▪ sales or purchases of any goods made by taxpayer


▪ receipts of taxpayer from services rendered
▪ any other receipts that may be chargeable to tax under ITO 2001, and
▪ acquisition, possession or disposal of any money, asset, valuable
article or investment made or expenditure incurred by the taxpayer

Amendment of Assessment in Cases of ‘Erroneous’ Assessment Order


§.122(5A)

216
Tariq Hussain Tunio

1. In addition to amendment and further amendment discussed above, there is another


type of amendment and further amendment. In this case the Commissioner is
authorized to amend or further amend an assessment order if he considers that the
assessment order is erroneous in so far as it is prejudicial to the interest of revenue.
It is commonly reffered to as amendment/further amendment u/s 122(5A).

2. The difference between this type of amendment/further amendment and the


amendment/further amendment discussed before is that here the basis for
amendment is the evidence acquired from the assessment order itself, in the form of
an error in it that causes loss to the revenue, whereas in the previous
amdendement/further amendment ‘audit’ or ‘definite information’ were required as a
precondition for amendment/further amendment, which required detailed
inquiry/scrutiny. However, there is no scope for detailed scrutiny or investigation in
amended assessment u/s 122(5A).

3. With respect to the time limits, there is no difference in both the types of amendment/
further amendment. The time limits for amendment is five years from the end of the
financial year in which the Commissioner is treated as having issued an assessment
order under the universal self-assessment or has issued a best judgment order to the
taxpayer.

4. The time limit for further amendment is later of:

(i) Five years from the end of the financial year in which the Commissioner has
issued or is treated as having issued the original assessment order to the
taxpayer; or

(ii) One year from the end of the financial year in which the Commissioner has
issued or is treated as having issued the amended assessment order to the
taxpayer.

Particulars of Amended Assessment Order

12. As soon as possible after making an amended assessment, the Commissioner is


required to issue an amended assessment order to the taxpayer, which should state
the following particulars:

▪ the amended taxable income of the taxpayer;


▪ the amended amount of tax due;
▪ the amount of tax paid, if any; and
▪ the time, place, and manner of appealing the amended assessment.

Opportunity of Being Heard

▪ No assessment may be amended or further amended by the Commissioner unless


the taxpayer has been provided an opportunity of being heard.

Explanation for Students


This opportunity embodies the basic principle of natural justice i.e. Audi Altrem
Partem (which means that ‘nobody can be condemned/punished unheard’). A
person must be given an opportunity of explaining his position before any
amended assessment order is passed against him.

217
CAF-02 Tax Practices

▪ Amended assessment order shall be made within 180 days of issuance of show
cause notice or within maximum periof od 90 days as may be extended by
Commissioner, for reasons to be recorded in writing.

▪ Any period during which proceedings are adjourned on account of

o stay order
o Alternative Dispute Resolution proceedings
o agreed assessment proceedings
o time taken through adjournment by taxpayer not exceeding 60 days

shall be excluded from the computation of the period.

5. REVISION BY THE COMMISSIONER §.122A

Orders that may be Revised

The Commissioner is authorized to revise any order passed by any Officer of Inland
Revenue, except the order passed by Commissioner (Appeals).

This authority of revision by the Commissioner is exercisable suo moto i.e. on his
own initiative.

Procedure of Revision by the Commissioner

The procedure to be followed in the process of revision is given below:

▪ The Commissioner calls for the record of any proceeding carried out under
Income Tax Ordinance,2001, in which an order has been passed by an
Officer of Inland Revenue.

▪ Commissioner makes an inquiry, if necessary.

▪ If the Commissioner considers that the order of the Officer of Inland Revenue
requires revision, he may revise it as he deems fit.

Revision to be only in Favor of Taxpayer

The order of revision by the Commissioner cannot be prejudicial to the taxpayer,


meaning thereby that the revision may only be done in favor of the taxpayer and not
against him.

Restriction on Revision
The Commissioner cannot revise any order of an Officer of Inland Revenue if:

▪ an appeal against the order of taxation officer has been filed by the
taxpayer and it lies to the Commissioner (Appeals) or to the Appellate Tribunal,
▪ the time within which appeal to the Commissioner (Appeals) or
Appellate Tribunal may be made has not expired,
▪ the order in appeal by the Commissioner (Appeals) is pending,
▪ the order by the Commissioner (Appeals) has been made the subject of
an appeal to the Appellate Tribunal.

218
Tariq Hussain Tunio

Chart# 7► REVISION BY COMMISSIONER §.122A

CIR may, suo


moto, call for by an Officer of
record of Inland Revenue
▪ any proceeding except
under ITO-2001 Commissioner
▪ in which an order (Appeals)
has been passed

If, after making


necessary inquiry,
Commissioner
considers that the
order requires
revision

CIR may make the


revision to the
order as he deems An order of revision
fit shall not be
prejudicial to the
person to whom the
order relates.
The Commissioner
shall not revise any
order if

▪ an appeal against the order the order is


lies to CIR (Appeals) or to the ▪ pending in appeal before the
Appellate Tribunal or Commissioner (Appeals) or
▪ the time within which such ▪ has been made subject of an
appeal may be made has not appeal to the Appellate
expired Tribunal

6. REVISION BY THE CHEIF COMMISSIONER §.122B

Chief Commissioner may, either of his own motion or on an application made by the
taxpayer for revision, call for the record of any proceedings relating to issuance of an
exemption or lower rate certificate with regard to collection or deduction of tax at
source under this Ordinance, in which an order has been passed by any authority
subordinate to him.

219
CAF-02 Tax Practices

Where, after making such inquiry as is necessary, Chief Commissioner considers


that the order requires revision, the Chief Commissioner may, after providing
reasonable opportunity of being heard to the taxpayer, make such order as he may
deem fit in the circumstances of the case.

7. AGREED ASSESSMENT IN CERTAIN CASES §.122D

1. Where a taxpayer, in response to Show Cause Notice [u/s 122 (9)], intends to
settle his case, he may file offer of settlement in the prescribed form before the
assessment oversight committee, in addition to filing reply to the Commissioner.

2. The Committee after examining the aforesaid offer may call for the record of the
case and after affording opportunity of being heard to the taxpayer, may decide to
accept or modify the offer of the taxpayer through consensus and communicate its
decision to the taxpayer.

3. Where the taxpayer is stratified with the decision of the Committee,-

(a) the taxpayer shall deposit the amount of tax payable including any amount
of penalty and default surcharge as per decision of the Committee;

(b) the Commissioner shall amend assessment in accordance with the decision
of the Committee after tax payable including any amount of penalty and
default surcharge as per decision of the Committee has been paid;

(c) the taxpayer shall waive the right to prefer appeal against such amended
assessment; and

(d) no further proceedings shall be undertaken under this Ordinance in respect


of issues decided by the Committee unless the tax as decided by committee
has not been deposited by the taxpayer.

4. Where the Committee has not been able to arrive at a consensus or where the
taxpayer is not satisfied with the decision of the Committee, the case shall be
referred back to the Commissioner for decision on basis of reply of taxpayer in
response to show cause notice u/s 122(9). And the proceedings or decision by
committee will have not effect in this case.

5. The Committee shall comprise the following income tax authorities having
jurisdiction over the taxpayer, namely:-

(a) the Chief Commissioner Inland Revenue;

(b) the Commissioner Inland Revenue; and

(c) the Additional Commissioner Inland Revenue.

6. This section shall not apply in cases involving concealment of income or where
interpretation of question of law is involved having effect on other cases.

220
Tariq Hussain Tunio

6. PROVISIONAL ASSESSMENT IN CERTAIN CASES §.123

1. CASE 1

▪ Where concealed asset of person


▪ is impounded by department or agency of Federal or Provincial Government

Commissioner may

▪ at any time before issuing any best judgement assessment order or amended
assessment order
▪ issue to person provisional assessment order or provisional amended
assessment order
▪ for last completed tax year of person taking into account the concealed asset.

2. CASE 2

▪ Where offshore asset of any person, not declared earlier,


▪ is discovered by Commissioner or any department or agency of Federal or
Provincial Government,

Commissioner may

▪ at any time before issuing any best judgement assessment order or amended
assessment order, issue to person provisional assessment order or
provisional amended assessment order
▪ for last completed tax year of person taking into account offshore asset
discovered.

3. Commissioner shall finalise provisional assessment order or provisional amended


assessment order as soon as practicable

4. “Concealed asset” means any property or asset which, in the opinion of the
Commissioner, was acquired from any income subject to tax under ITO 2001.

ILLUSTRATION
Q: Dr. X was arrested at Karachi airport with gold worth Rs. 10 (M) on October 15, 2009 by
Federal Investigation Agency (FIA), which also impounded his gold. Dr. X’s past record at
the tax department was examined to see if he had declared the gold in his wealth statement
and it was discovered that he had not disclosed the aforesaid property in his wealth
statements. His return of income for Tax Year 2009 declared taxable income at Rs. 620,000/-.
Describe the method through which this concealed asset may be charged to tax?

A: The amount of Rs.10 (M) is chargeable to tax, for being concealed asset, through passing a
provisional amended assessment order for Tax Year 2009.

221
CAF-02 Tax Practices

Chart# 5► PROVIONSAL ASSESSMENT IN CERTAIN CASES §.123

Concealed asset
Definition: Concealed
of a person is asset means any property
impounded/ or asset which, in the
opinion of CIR, was
Offshore asset acquired from any income
discovered subject to tax ITO-2001.

by any department or
agency of Fed Govt or
a Provincial
Government

▪ for the last completed


tax year of the person
CIR may issue to the ▪ taking into account the
person a concealed asset.
▪ provisional
assessment order or
▪ provisional amended
assessment order
▪ at any time before
issuing assessment
order or amended
assessment order

CIR finalizes a
provisional assessment
order or a provisional
amended assessment
order as soon as
practicable.

7. ASSESSMENT GIVING EFFECT TO AN ORDER §.124

Where in consequence of or to give effect to any finding or direction in order made


by CIR (Appeals), ATIR, High Court, or Supreme Court, an assessment order or
amended assessment order is to be issued to any person, the Commissioner shall
issue the order within 2 years from end of financial year in which order of
CIR(Appeals), ATIR, High Court or Supreme Court, was served on Commissioner.

222
Tariq Hussain Tunio

▪ Where, by an order made by ATIR, High Court, or Supreme Court, an assessment


order is set aside wholly or partly, and Commissioner or CIR (Appeals) is directed
to pass a new assessment order, CIR or CIR (Appeals) shall pass the new order
within 1 year from end of financial year in which CIR or CIR (Appeals) is served
with the order.

▪ This limitation shall not apply, if an appeal or reference has been preferred,
against the order, passed by ATIR or High Court.

▪ Where assessment order has been set aside or modified, proceedings may
commence from stage next preceding the stage at which such setting aside or
modification took place and nothing contained in ITO 2001Ordinance shall render
necessary re-issue of any notice which had already been issued or re-furnishing or
re-filing of any return, statement, or other particulars which had already been
furnished or filed.

▪ Where direct relief is provided in an order by CIR (Appeals) or ATIR, the


Commissioner shall issue appeal effect orders within 2 months of date the
Commissioner is served with the order.

▪ The provisions of this section shall in like manner apply to any order issued by any
High Court or the Supreme Court in exercise of original or appellate jurisdiction.

8. ASSESSMENT GIVING EFFECT TO AN ORDER §.124A

▪ Where question of law has been decided by High Court or ATIR in case of
taxpayer, the Commissioner may, notwithstanding that he has preferred an appeal
against decision of High Court or made application for reference against order of
ATIR, follow said decision in case of said taxpayer in so far as it applies to said
question of law arising in any assessment pending before Commissioner until the
decision of High Court or of ATIR is reversed or modified.

▪ In case decision of High Court or ATIR, is reversed or modified, Commissioner


may, notwithstanding expiry of period of limitation prescribed for making any
assessment or order, within a period of 1 year from date of receipt of decision,
modify assessment or order in which the said decision was applied so that it
conforms to the final decision.

10. EVIDENCE OF ASSESSMENT §.126

1. The production of an assessment order or a certified copy of an assessment order is


conclusive evidence that the assessment of the taxable income and tax due of a
taxpayer has been duly made for a tax year.

Particulars Of Assessment Order To Be Taken As Correct

2. The amounts and all particulars relating to assessment as specified in the


assessment order should be taken to be correct except in the proceedings of
appeals. In the appeal proceedings, the Commissioner (Appeals), Appellate Tribunal
etc may bring about changes in the amounts and other particulars of an assessment
order if necessary.

223
CAF-02 Tax Practices

Substance over Form

3. Any assessment order or other document purporting to be made, issued, or executed


under Income Tax Ordinance, 2001, may not be quashed or deemed to be void or
voidable for want of form; or may not be affected by reason of any mistake, defect, or
omission therein, if: (i) it is, in substance and effect, in conformity with ITO-2001 and
(ii) the person assessed, or intended to be assessed or affected by the document, is
designated in it according to common understanding.

9. ASSESSMENT IN RELATION TO DISPUTED PROPERTY §.125

Where the ownership of any property the income from which is chargeable to tax
is in dispute in any Civil Court in Pakistan, an assessment order or amended
assessment order in respect of such income may be issued at any time within
one year after the end of the financial year in which the decision of the Court is
made.

Chart# 6► ASSESSMENT IN RELATION TO A DISPUTED


PROPERTY §.125

Where ownership of a
property producing
income chargeable to
tax is in dispute in any
Civil Court in Pakistan

an assessment order or
amended assessment
order in respect of such
income may be issued

at any time within


▪ one year after the end
of the FY
▪ in which the decision
of Court is made

224
Tariq Hussain Tunio

10. RESTRICTION OF PROCEEDINGS §.120B

▪ Where any person entitled to declare


o undisclosed assets
o expenditure and
o undisclosed sales
o under the Assets Declaration Act, 2019

▪ declares such assets, expenditures or sales to pay tax,

▪ no proceedings shall be undertaken under ITO 2001 in respect of such


declaration

▪ Particulars of person making declaration under Assets Declaration Act, 2019 or


any information received in any declaration made under said Act shall be
confidential.

225
CAF-02 Tax Practices

PAST EXAMINATION QUESTION

Question # 1
Autumn 2016 Q.2

Maroof filed his return of income for tax year 2015 on 30 September 2015. On 15 August
2016 he received a show Cause notice from the Commissioner Inland Revenue u/s 122- for
amendment of the assessment order issued on self-assessment basis.

Required:
Under the provisions of the Income Tax Ordinance, 2001 briefly describe:

1. the circumstances under which an assessment order treated as issued on self-


assessment basis'may be amended by the Commissioner. (04)

2. the situations in which the Commissioner may be barred from amending the original
assessment order. (04)

Question # 2
Autumn 2017 Q.4(b)

b) List the situations under which an original assessment can be amended or an


amended assessment can be further amended by the Commissioner of Income Tax.
Also state the time period within which the original or the previously amended
assessment order can further be amended. (07)

Questlon # 3
Spring 2018 Q. 3 (a, b, d)

(a) Under the provisions of the Income Tax Ordinance, 2001 briefly discuss the
following:

(i) the term 'Concealed assets’. (02)


(ii) The powers of Commissioner relating to the concealed assets of any person
when these are impounded by the Federal Government (03)

(b) Anwar had filed his return of income for the tax year 2013 on 31 August 2013.
Discuss the following in the light of provisions of the Income Tax Ordinance, 2001:

(i) By which date the Commissioner of Income Tax could make the first
amendment of the assessment, if required. (02)

(ii) By which date any further amendment can be made if the first amendment was
made on 15 February 2017. (02)

d. Under the provisions of the Income Tax Rules, 2002 list the records to be kept
by a taxpayer in respect of his income from:
(i) Salary (01)
(ii) Property (1.5)
(iii) Capital gain (1.5)

226
Tariq Hussain Tunio

Question # 4
Autumn 2019 Q. 4

Under the provisions of the Income Tax Ordinance, 2001 and Rules made thereunder, briefly
explain the following:

(a) Requirement of books of account to be maintained by a taxpayer who has business


income up to Rs. 500,000. (04)

(b) General provisions/rules which may apply to income subject to Final Tax Regime. (06)

(c) Provisions regarding Special Audit Panel. (05)

Required:

Under the provisions of the Income Tax Ordinance, 2001:

Question # 5
Spring 2020 Q.3(c )

( c ) the concept of ‘concealed asset’ and state the powers of the Commissioner relating to
concealed asset of any person when it is impounded by the Federal Government.

Question # 6
Spring 2021 Q.5

Star garments Limited (SGL) had filed its tax return for the tax year 2015 on 30 September
2015.
On 25 February 2021, the Commissioner of Income Tax, on the basis of definite information,
issued a notice u/s 122(5) to SGL for the audit of the books of account for the tax year 2015.

The accountant informed the chief executive officer that tax audit for the tax year 2015 had
already been conducted in 2019 and an amended assessment order u/s 122(5A) was issued
by the Commissioner on 24 February 2020.

Required:
Under the provisions of the Income Tax Ordinance, 2001:

(a) Explain the term ‘Definite information'. (02)


(b) Discuss whether the Commissioner is empowered to make further amendment in the
assessment order issued on 24 February 2020. (07)

Question # 7
Spring 2022 Q.4

(a) Briefly explain the term ‘Sectoral benchmark ratios’. Also, explain the circumstances
in which a Commissioner shall determine taxable income on the basis of sectoral benchmark
ratios. (03)
(b)

227
CAF-02 Tax Practices

(i) Riaasat Limited (RL) is a manufacturing company. With effect from 1 July 2022, RL is
considering to change its tax year from the normal to the special tax year ending on 31
December.

Identify the due/last date of filing of RL’s tax return in respect of the following:
■ Filing of tax return for the year ended 3 0 June 2022.
■ Filing of tax return for the transitional period.
■ Filing of first tax return for the special tax year. (03)

Question #8
Autumn 2022 Q. 5(a)

Under the provisions of the Income Tax Ordinance, 2001 and Rules made thereunder:

(i) briefly discuss the terms 'Normal assessment’ and 'Best judgement assessment’.
(03)

(ii) state the requirements that should be complied with by a sole proprietor on
discontinuance of business.(02)

(iii) list the additional records which are required to be kept by a sole proprietor whose
business income exceeds Rs. 500,000 as compared to a sole proprietor whose
business income is upto Rs. 500,000. (02)

228
Tariq Hussain Tunio

229
CAF-02 Tax Practices

Chapter

16

APPEALS
CHAPTER SYNOPSIS
TOPIC SECTION
Appeal to Commissioner (Appeals) 127-129
Appeal to Appellate Tribunal 130-132
Reference to High Court 133
Alternative dispute resolution 134A
Burden of proof 136
ICAP Past Papers Questions

230
Tariq Hussain Tunio

1. APPEAL TO COMMISSIONER (APPEALS) §.127-129

Appealable Orders

1. Any person dissatisfied with the following orders passed by a Commissioner or an


Officer of Inland Revenue may file an appeal with the Commissioner (Appeals)
against that order.

S# ORDERS AGAINST WHICH APPEAL MAY BE FILED


1 Online automatice check of accuracy u/s 120(2A)
2 Best Judgment Assessment
3 Amendment of Assessment
4 Non-resident ship owner or charterer or Non-resident
aircraft owner or charterer
5 Recovery of tax from the person from whom tax was not
collected or deducted
6 Refund
7 Penalties
8 an order holding a person to be personally liable to pay
an amount of tax
9 an order declaring a person to be the representative of a
non-resident person
10 an order relating to rectification of mistakes refusing to
rectify the mistake, either in full or in part, as claimed by
the taxpayer
an order having the effect of enhancing the assessment
or reducing a refund or otherwise increasing the liability
of the person

Requirements of Appeal to Commissioner (A)

2. An appeal to be filed with the Commissioner (A) is required to fulfill the following
conditions:

▪ It has to be in the prescribed form,


▪ It has to be verified in the prescribed manner,
▪ It has to state precisely the grounds upon which the appeal is made,
▪ It has to be accompanied by the prescribed fee, and
▪ It has to be lodged with the Commissioner (Appeals) within the stipulate time.
▪ In case of an appeal against assessment order, the appeal cannot be filed unless
the appellant has paid the amount of tax computed by himself on his return and
which he was required to pay along with return of income but failed to pay.

Fee for Filing Appeal

3. Appeal filed with Commissioner (A) is required to be accompanied by the following


amount of fee:

SR. NO TYPE OF APPEAL FEE in Rs.


1 Appeal against an Assessment Company Rs. 5000
Order Others Rs. 2500

231
CAF-02 Tax Practices

2 Appeal against an order other Company Rs. 5000


than assessment Order Others Rs. 1000

Time Limits for Filing Appeal to Commissioner (A)

4. An appeal against the orders of a Commissioner or Officer of Inland Revenue has to


be filed within thirty (30) days of the date of service of the order against which the
appeal is to be filed.

Computation of Thirty Days

5. In case the appeal is against an assessment order or a penalty order, the time limit of
thirty days starts from the date of service of the ‘notice of demand’.

6. Notice of demand is issued along with assessment order or penalty order and
requires a taxpayer to pay the amount of tax or penalty imposed.

7. In case of appeal against any other order, the time limit of thirty days start from the
date the order appealed against is served on the taxpayer.

Condonation of Time

8. If a taxpayer has failed to file an appeal in the given time, he may apply in writing to
the Commissioner (A) to allow him to file an appeal after the expiry of the time limits.
9. The Commissioner (Appeals) is authorized to admit an appeal after the expiration of
the stipulated period if he is satisfied that the appellant was prevented by sufficient
cause from lodging the appeal within time limits provided for the filing of appeal.

Procedure to be Followed in Appeal

Date of Hearing

10. The Commissioner (A) gives a notice of the day fixed for the hearing of the appeal to
both the parties i.e. the appellant and to the Commissioner.

Power of Adjournment of Hearings

11. The Commissioner (A) may adjourn the hearing of the appeal from time to time

Filing New Grounds of Appeal

12. Before the hearing of an appeal, the Commissioner (A) may allow an appellant
to file any new ground of appeal not specified in the grounds of appeal already filed
by the appellant if the Commissioner (A) is satisfied that the omission of the ground
from the form of the appeal was not willful or unreasonable.

Record Requisition & Inquiries

13. During the hearing of appeal, the Commissioner (A) may call for necessary
particulars relating to the matters arising in the appeal or he may also cause further
enquiry to be made by the Commissioner.

232
Tariq Hussain Tunio

Evidence not Produced Before the CIR is not Admissible

14. The Commissioner (Appeals) cannot admit any documentary material or evidence
which was not produced before the Commissioner.

15. This material or documents may be admitted only when Commissioner (A) is satisfied
that the appellant was prevented by sufficient cause from producing such material or
evidence before the Commissioner.

Decision by Commissioner (A)

Types of Decisions

16. As soon as practicable after deciding an appeal, the Commissioner (A) shall specify
in the order the amount of tax upheld and to serve the order on both the appellant
and the Commissioner.

17. In disposing of an appeal, the Commissioner (A) may make following types of
decisions:

▪ In case of appeal against an assessment order, the Commissioner (A) may make
an order to confirm, modify or annul the assessment order,

▪ In case of appeals against an order other than assessment order, Commissioner


(A) may make such order as he thinks fit.

Opportunity of Being Heard

18. The Commissioner (A) cannot increase the amount of any assessment order or
decrease the amount of any refund unless the appellant has been given a
reasonable opportunity of showing cause against such increase or decrease.

Time Limits for Decision by Commissioner (A)

19. The Commissioner (A) is required to dispose of an appeal within four months from
the end of the month in which appeal was filed. If the Commissioner (A) fails to make
an order on an appeal within the given time, the relief sought by the appellant in the
appeal is treated as having been given and all the provisions of ITO-2001 have effect
accordingly.

20. In computing the four months after the end of the month in which appeal was lodged,
any period during which the hearing of an appeal is adjourned on the request of the
appellant are excluded in the computation.

2. APPELLATE TRIBUNAL §.130

▪ There shall be established Appellate Tribunal to be called Appellate Tribunal Inland


Revenue (ATIR) to exercise the powers and perform the functions conferred on
Appellate Tribunal Inland Revenue by ITO 2001.

▪ ATIR shall consist of chairman and such other judicial and accountant members
who shall be appointed in such numbers and in such manner as Prime Minister may
prescribe by rules.

233
CAF-02 Tax Practices

▪ No person shall be appointed as judicial member of ATIR unless he

(a) has been a Judge of a High Court


(b) is or has been a District Judge
(c) is an advocate of a High Court with a standing of not less than ten years
(d) possesses such other qualification as may be prescribed under sub-section (2) of
this section.d

▪ No person shall be appointed as an accountant member of ATIR unless he

(a) is an officer of IRS equivalent in rank to that of Chief Commissioner IR


(b) is a CIR or CIR (Appeals) having not less than 3 years experience as CIR
(c) has for a period of not less than 10 years practiced professionally as CA
(d) has for a period of not less than 10 years practiced professionally as CMA (cost
and management accountant)

▪ Constitution, functioning of benches and procedure of ATIR shall be regulated by


rules which PM may prescribe. [Prior ruels to continue in force unless amended or
repealed.]

2. APPEAL TO APPELLATE TRIBUNAL §.131

Filing Second Appeal With Appellate Tribunal

If the taxpayer or Commissioner objects to an order passed by the Commissioner (A),


the taxpayer or Commissioner may file a second appeal to the Appellate Tribunal against
order of the Commissioner (A).

Requirements of Appeal to Appellate Tribunal

An appeal against the orders of Commissioner (A) should fulfill the following conditions:

▪ It has to be in the prescribed form,

▪ It has to be verified in the prescribed manner,

▪ It has to be accompanied by the prescribed fee, except in case of an


appeal preferred by the Commissioner,

▪ It has to be lodged with the Appellate Tribunal within the stipulated time
limit

Fee for Filing Appeal to the Appellate Tribunal

▪ For company Rs. 5000/-


▪ Other cases Rs. 2500/

Time Limits for Filing Appeal to Tribunal

An appeal against the decision of Commissioner (A) is required to be preferred to the


Appellate Tribunal within 60 days of the date of service of order of the Commissioner
(A) on the taxpayer or the Commissioner.

Condonation of Time

234
Tariq Hussain Tunio

In case of failure to file second appeal within sixty days of the date of service of order
of the Commissioner (A), the Tribunal is authorized to admit an appeal if it is satisfied
that the appellant was prevented by sufficient cause from lodging the appeal within
the time limits provided for the filing of appeal.

Stay Against Recovery Of Tax

▪ Notwithstanding an appeal has been filed, tax shall, unless its recovery has been
stayed by ATIR, be payable in accordance with assessment.

▪ On filing of application, if ATIR is of the opinion that the recovery of tax levied
under ITO 2001 and upheld by CIR Appeals), shall cause undue hardship to
taxpayer, the Tribunal, after affording opportunity of being heard to CIR, may stay
recovery of such tax for a period not exceeding 180 days in aggregate.

▪ Where recovery of tax has been stayed, such stay order shall cease to have
effect on expiration of the said period of 180 days following the date on which the
stay order was made and CIR shall proceed to recover the said tax

▪ In computing the aforesaid period of 180 days, period, if any, for which recovery
of tax was stayed by a High Court, shall be excluded.

2. DISPOSAL OF APPEAL BY APPELLATE TRIBUNAL §.132

Disposal of Appeal by Appellate Tribunal

The Appellate Tribunal is authorized to call for necessary particulars relating to the
matters arising on the appeal or cause further enquiry to be made by the Commissioner
during the hearing of the appeal.

Moreover, during the proceedings, it is the obligation of the Tribunal to give an


opportunity of being heard to the parties to the appeal.

Non Appearance on the Date of Hearing

In case a party fails to appear on the date of hearing before the Tribunal, the Tribunal
may proceed ex parte to decide the appeal on the basis of the available record.

Time Limit for Decision by Appellate Tribunal

The Appellate Tribunal is required to make decision in appeal in six months from the
date of its filing.

Types of Decision By Appellate Tribunal

Appeal against Assessment Orders

In case of an appeal against an assessment order, the Appellate Tribunal may make any
of the following order:

▪ affirm, modify or annul the assessment order, or

235
CAF-02 Tax Practices

▪ remand the case to the Commissioner or the Commissioner (Appeals) for making
such enquiry or taking such action as the Tribunal may direct. This type of
decision is also called ‘set aside decision’.

Appeal against Non-Assessment Orders

If the appeal filed relates to a decision other than the one relating to assessment, the
Appellate Tribunal may make an order to:

▪ affirm, vary or annul the decision, and

▪ issue such consequential directions as the case may require.

Communication of the Order by the Tribunal

Having issued a decision, the Appellate Tribunal is required to communicate its order to
the taxpayer and the Commissioner.

Opportunity of Being Heard

The Appellate Tribunal cannot (i) increase the amount of any assessment or (ii) increase
the amount of any penalty or (iii) decrease the amount of any refund unless the taxpayer
has been given a reasonable opportunity of showing cause against such increase or
decrease.

Tribunal’s Decision is Final in Question of Fact

The decision of the Appellate Tribunal is final on the ‘Question of Fact’, meaning
thereby that no further appeal/reference may be filed against it in the High Court.
However, a reference may be filed in the High Court against the order of Appellate
Tribunal if it relates to Question of Law.

Explanation for Students


The appeals may be categorized into two types: those related to question of
fact and those related to question of law. The issues or matters to be judged in
‘Question of Fact’ are those that relate to ‘facts’, rather than interpretation of
law.

For example, whether or not auto parts for automobiles are products of iron
and steel, whether or not tax was deducted on the amount of salary received
by the taxpayer, or whether or not the taxpayer has filed the return on due
date, etc.

‘Question of Law’ relates to the interpretation or application of the provisions of


the Income Tax Ordinance, 2001.

For example, whether or not an amount given after the retirement of a person
is taxable as part of salary, whether or not an expense incurred on business
tour is deductable under the head ‘Income from Business’, or whether or not
an unsigned return becomes deemed assessment order, etc.

236
Tariq Hussain Tunio

3. REFERENCE TO HIGH COURT §.133

Filing Reference to High Court

The Commissioner or the taxpayer may file a reference to the High Court against the
decision of the Appellate Tribunal within ninety days of the communication of the
order of the Appellate Tribunal.

The person filing the reference is required to submit following document to the High Court:

▪ A application in the prescribed, and


▪ A statement of the case in which following particulars are stated:

- facts of the case,


- decision of the Appellate Tribunal, and
- the question of law which has arisen out of the decision of the Tribunal.

Fee for Filing the Reference to High Court

A taxpayer is required to pay a fee of Rs. 100/- along with the application for
reference to the High Court. This fee is not required to be paid in case the reference
is filed by the Commissioner.

Procedure for Disposal of Reference by the HC

Upon application by the taxpayer or Commissioner, the High Court first decides
whether or not a question of law has arisen from the decision of the Tribunal. If the
High Court is satisfied that a question of law has arisen out of the order of the
Tribunal, then the Court may proceeds to hear the case. If the court is not satisfied
that a question of law has arisen, then it rejects the application for reference.

A reference to the High Court is heard by a Bench of not less than two judges of the
High Court. After hearing the reference, the High Court decides the question of law
raised by the reference and passes judgment. The judgment of the High Court is
required to specify the grounds on which it is based.

The Court serves a copy of the judgment under its seal on the Appellate Tribunal,
wherefrom it is served on the parities to the case.

Postponement of Refund

The High Court is authorized to decide the reference by issuing any type of order.
Among the orders that the High Court may pass is the reduction of tax liability of the
taxpayer.

If as a result of this, a refund is created, and the Commissioner wants to prefer a


petition for leave to appeal to the Supreme Court challenging the order of the High
Court, the High Court, upon application by the Commissioner, may allow the
Commissioner to postpone the refund so created until the disposal of the appeal by
the Supreme Court.

The application by the Commissioner is required to be made within thirty days of the
receipt of the judgment of the High Court.

237
CAF-02 Tax Practices

Stay Against Recovery Of Tax

Despite filing a reference to High Court, a taxpayer is required to pay the tax in
accordance with the order of the Appellate Tribunal. The recovery of the tax may,
however, by stayed by the High Court by an order. The stay order of the High Court
may be granted for period of six months.

The order of High Court staying the recovery of tax ceases to have effect before the
expiry of six months if the appeal is decided or the stay order has been withdrawn by
the High Court earlier than that period.

4. ALTERNATIVE DISPUTE RESOLUTION §.134A

▪ An aggrieved person in connection with any dispute pertaining to

1. liability of tax of Rs. 100 million or above against aggrieved person or


admissibility of refund

2. extent of waiver of default surcharge and penalty

3. any other specific relief required to resolve the dispute

may apply to Board for appointment of committee for resolution of any hardship
or dispute mentioned in detail in application, which is under litigation in any court
of law or an appellate authority, except where criminal proceedings have been
initiated.

▪ Application for dispute resolution shall be accompanied by an initial proposition


for resolution of the dispute, including an offer of tax payment.

▪ Board may, after examination of application of aggrieved person, appoint a


committee, within 15 days of receipt of application in the Board, comprising, –

(i) a retired judge not below the rank of a judge of a High Court, who shall also
be the Chairperson of the Committee, to be nominated by the Board from a
panel notified by the Law and Justice Division for such purpose;

(ii) the Chief Commissioner Inland Revenue having jurisdiction over


the case; and

(iii) a person to be nominated by the taxpayer from a panel notified


by the Board comprising

a. chartered accountants, cost and management accountants and


advocates having a minimum of ten years’ experience in the field of
taxation;

b. officers of the Inland Revenue Service who stood retired in BS


21 or above; or

c. reputable businessmen as nominated by the Chambers of


Commerce and Industry: Provided that the taxpayer shall not
nominate a chartered accountant or an advocate if the said chartered

238
Tariq Hussain Tunio

accountant or the advocate is or has been an auditor or an authorized


representative of the taxpayer.

▪ Board shall communicate the order of appointment of Committee to the aggrieved


person, court of law or the appellate authority where the dispute is pending and to
the concerned Commissioner.

▪ Committee appointed shall examine the issue and may, if it deems necessary,
conduct inquiry, seek expert opinion, direct any officer of the Inland Revenue or
any other person to conduct an audit and shall decide the dispute by majority,
within forty-five days of its appointment extendable by another fifteen days for the
reasons to be recorded in writing.

▪ Decision by Committee shall not be cited or taken as a precedent in any other


case or in the same case for a different tax year.

▪ The recovery of tax payable by a taxpayer in connection with any dispute for
which a Committee has been appointed shall be deemed to have been stayed on
the constitution of Committee till the final decision or dissolution of the
Committee, whichever is earlier.

▪ The decision of the Committee hall be binding on the Commissioner when the
aggrieved person, being satisfied with the decision, has withdrawn the appeal
pending before the court of law or any appellate authority in respect of dispute
and has communicated the order of withdrawal to the Commissioner:

▪ If order of withdrawal is not communicated to CIR within 60 of service of decision


of the Committee upon the aggrieved person, the decision of Committee shall not
be binding on the Commissioner.

▪ Commissioner shall also withdraw the appeal, if any, pending before any court of
law or an appellate authority in respect of dispute within 30 days of the
communication of the order of withdrawal by the aggrieved person to the
Commissioner.

▪ The aggrieved person shall make the payment of income tax and other taxes and
within time decided by Committee and all decisions and orders made or passed
shall stand modified to that extent.

▪ If the Committee fails to decide within the period of 60 days, the Board shall
dissolve the Committee by an order in writing and the matter shall be decided by
the court of law or the appellate authority where the dispute is pending under
litigation.

▪ Board shall communicate the order of dissolution to the aggrieved person, court
of law or the appellate authority and to the CIR.

▪ On receipt of the order of dissolution, the court of law or the appellate authority
shall decide the appeal within 6 months of the communication of the said order.

▪ Board may prescribe amount to be paid as remuneration for the services of the
members of the Committee, other than chief commisiuoner.

239
CAF-02 Tax Practices

5. BURDEN OF PROOF §.136

1. In any appeal filed by a taxpayer before an appellate forum, the burden of


proof lies with the taxpayer in the following matters:

▪ in the case of an assessment order, the extent to which the order does not
correctly reflect the taxpayer’s tax liability for the tax year, or

▪ in the case of any other decision, that the decision is erroneous.

240
Tariq Hussain Tunio

PAST EXAMINATION QUESTION


Question # 1
Autumn 2015 Q. 3(b)

Under the provisions of the Income Tax Ordinance, 2001 determine the date by which
appeal can be filed with the Commissioner (Appeals) in the following cases:
(i) Assessment order for tax year 2014 was made on 31 December 2014. Demand
notice was served on 1. January 2015. (02)

Question # 2
Spring 2019 Q. 4(a,b)

a) Under the Income Tax Ordinance, 2001 identify four situations under which an
appeal may be filed with the Commissioner (Appeals). (04)

b) Sadiq Ali has received an ex-parte assessment order from the income tax
department under which he is required to pay Rs. 5.2 million on account of tax not
withheld from certain payments. He does not agree with the contention of the income
tax department and would like to hie an appeal to the Commissioner (Appeals).
Required:

State the procedure that he should follow for filing of appeal to the Commissioner (Appeals).
(03)

Question # 3
Autumn 2020 Q. 3(a)

On 2 July 2019, Rubina received a show cause notice u/s 122 from the Commissioner Inland
Revenue (CIR) for amendment of the assessment order for tax year 2018. Due to lack of
knowledge about tax matters, she.did not respond to it.

On 1 August 2019, she received a demand notice under which she was required to pay Rs.
610,000 within 30 days on account of undeclared income and an amended assessment
order for tax year 2018 under section 122 from the CIR.

Rubina is dissatisfied with the order issued by the CIR and wants to file an appeal to the
Commissioner (Appeals) because payment of this amount will cause hardship to her.

Required:
Under the provisions of the Income Tax Ordinance, 2001:

ii. state the time period within which an appeal may be filed by Rubina to the
Commissioner (Appeals). (01)

iii. discuss different types of orders that the Commissioner (Appeals) may make for
disposing of an appeal; (02)

iv. explain what action(s) the Commissioner (Appeals) may take for ensuring that no
undue hardship will be ' caused to Rubina because of the payment of this demand.
(03)

241
CAF-02 Tax Practices

v. discuss the option(s) available to Rubina for defending her case, if the Commissioner
(Appeals) issues an order confirming the amended assessment order issued by the
CIR. (02)

242
Tariq Hussain Tunio

Chapter

17

AUDIT & RECORD


CHAPTER SYNOPSIS

TOPIC SECTION
Records 174
Audit 177
ICAP Past Papers Questions

243
CAF-02 Tax Practices

1. RECORDS §.174

1. Unless otherwise authorised by the Commissioner, every taxpayer is required to


maintain in Pakistan the prescribed

▪ accounts,
▪ documents and
▪ records.

2. The Commissioner may disallow or reduce a taxpayer’s claim for a deduction if the
taxpayer is

▪ unable, without reasonable cause, to provide

o a receipt,
o other record or
o evidence of the
- `transaction or
- circumstances
- on the basis of which the deduction is claimed

3. The accounts and documents are required to be maintained

▪ for six years


▪ after the end of the tax year to which they relate.

4. Where any proceeding is pending before any authority or court

▪ the taxpayer shall maintain the record


▪ till final decision of the proceedings.

5. Pending proceedings include proceedings for

▪ assessment or amendment of assessment,


▪ appeal,
▪ revision,
▪ reference,
▪ petition or
▪ prosecution and
▪ any proceedings before an Alternative Dispute Resolution Committee.

6. “Deduction” means any amount debited to

▪ trading account,
▪ manufacturing account,
▪ receipts and expenses account or
▪ profit and loss account.

7. The Commissioner may require any person

▪ to install and use an Electronic Tax Register


▪ of such type and description as may be prescribed
▪ for the purpose of storing and accessing information

244
Tariq Hussain Tunio

▪ regarding any transaction that has a bearing on the tax liability of such
person.

2. AUDIT §.177

▪ CIR may call for any record or documents including books of accounts maintained
under ITO 2001 or any there law for the time being in force for conducting audit of
income tax affairs of the person and where such record or documents have been kept
on electronic data, the person shall allow access to CIR or officer authorized by CIR
for use of machine and software on which such data is kept and CIR or officer may
have access to required information and data and duly attested hard copies of such
information or data for the purpose of investigation and proceedings under this
Ordinance in respect of such person or any other person.

▪ Provided that

(a) CIR may, after recording reasons in writing call for record or documents
including books of accounts of the taxpayer; and
(b) the reasons shall be communicated to the taxpayer while calling record or
documents including books of accounts of the taxpayer:

▪ Provided further that CIR shall not call for record or documents of the taxpayer after
expiry of six years from the end of the tax year to which they relate.

▪ After obtaining record of a person or where necessary record is not maintained, CIR
shall conduct audit of income tax affairs (including examination of accounts and
records, enquiry into expenditure, assets and liabilities) of that person or any other
person and may call for such other information and documents as he may deem
appropriate.

▪ CIR may conduct audit proceedings electronically through video links, or any other
facility as prescribed by the Board.

▪ Where a taxpayer—

(a) has not furnished record or documents including books of accounts;


(b) has furnished incomplete record or books of accounts; or
(c) is unable provide sufficient explanation regarding the defects in records,
documents or books of accounts, it shall be construed that taxable income has
not been correctly declared and the Commissioner shall determine taxable
income on the basis of sectoral benchmark ratios prescribed by the Board.

“Sectoral Benchmark Ratios” means standard business sector ratios notified by the
Board on the basis of comparative cases and includes financial ratios, production
ratios, gross profit ratio, not profit ratio, recovery ratio, wastage ratio and such other
ratios in respect of such sectors as may be prescribed.

▪ After compilation of audit, CIR shall, after obtaining taxpayer’s explanation on all
issues raised in audit, issue an audit report containing audit observations and finding.

▪ After issuing audit report, CIR may, if considered necessary, amend assessment after
providing an opportunity of being heard to the taxpayer through show casue notice
[u/s 122 (9)]

245
CAF-02 Tax Practices

▪ The fact that a person has been audited in a year shall not preclude the person from
being audited again in the next and following years where there are reasonable
grounds for such audits.

▪ Board may appoint a firm of CA or firm of CMA to conduct audit of income tax affairs
of any person or classes of persons and scope of such audit shall be as determined
by the Board or the Commissioner on a case to case basis.

▪ Any person employed by a firm may be authorized by CIR, in writing, to exercise the
powers in sections 175 and 176 for the purposes of conducting an audit.

▪ Where a person fails to produce before CIR or firm of CA or CMA appointed by


Board or CIR to conduct audit, any accounts, documents and records, required to be
maintained under ITO-2001 [section 174] or any other relevant document,
electronically kept record, electronic machine or any other evidence that may be
required by CIR or firm of CA or CMA for the purpose of audit or determination of
income and tax due thereon, CIR may proceed to make best judgment assessment
and assessment treated to have been made on the basis of return or revised return
filed by the taxpayer shall be of no legal effect.

▪ Explanation.— For the removal of doubt, it is declared that the powers of CIR (u/s
177) are independent of powers of Board (u/s 214C) and nothing contained in section
214C restricts powers of CIR to call for record or documents including books of
accounts of a taxpayer for audit and to conduct audit.

▪ Board may appoint as many special audit panels as may be necessary, comprising
two or more members from the following:—

(a) an officer or officers of Inland Revenue


(b) a firm of CA
(c) a firm of CAM
(d) any other person including a foreign expert or specialist as directed by the
Board, to conduct an audit, including a forensic audit, of the income tax affairs
of any person or classes of persons and the scope of such audit shall be as
determined by Board or CIR on case-to-case basis.
(e) tax audit expert deployed under an audit assistance programme of an
international tax organization or a tax authority outside Pakistan.

In case the member is not an officer of Inland Revenue, the person shall only
be included as a member in the special audit panel if an agreement of
confidentiality has been entered into between the Board and the person,
international tax organization or a tax authority, as the case may be.

▪ Special audit panel shall be headed by a Chairman who shall be an officer of


Inland Revenue.

▪ Powers for the purposes of conducting an audit shall only be exercised by an


officer or officers of Inland Revenue, who are member or members of the special
audit panel, and authorized by the Commissioner.

▪ Where a person fails to produce before CIR or special audit panel to conduct
audit, any accounts, documents and records, required to be maintained or any other
relevant document, electronically kept record, electronic machine or any other

246
Tariq Hussain Tunio

evidence that may be required by CIR or panel, CIR may proceed to make best
judgment assessment and the assessment treated to have been made on the basis
of return or revised return filed by the taxpayer shall be of no legal effect.

▪ If any one member of special audit panel, other than Chairman, is absent from
conducting an audit, the proceedings of the audit may continue, and the audit
conducted by the special audit panel shall not be invalid or be called in question
merely on the ground of such absence.

▪ Functions performed by an officer or officers of Inland Revenue as members of the


special audit panel, for conducting audit, shall be treated to have been performed by
special audit panel.

▪ Board may prescribe the mode and manner of constitution, procedure and working of
the special audit panel.

247
CAF-02 Tax Practices

PAST EXAMINATION QUESTION


Question # 1
Autumn 2009 Q.4(b)

Ayub Industries Limited has been selected for the audit of its income tax affairs. The
management is of the opinion that since their tax affairs were audited last year also, they
should not have been selected for audit this year.

Required:
Discuss the management’s point of view in the light of Income Tax Ordinance,2001. (02)

Question # 2
Spring 2018 Q.3(d)

Under the provisions of the Income Tax Rules,2002. List the records to be kept by a
taxpayer In respect of his income from:
(i) salary (01)
(ii) property (1.5)
(iii) capital gain (1.5)

Question # 3
Autumn 2019 Q.4(a),( c )

Under the provisions of the Income Tax Ordinance,2001 and Rules made thereunder, briefly
explain the following:
a) requirement of books of account to be maintained by a taxpayer who has business
income upto Rs. 500,000.(04)
b) provisions regarding Special Audit Panel. (05)

Question # 4
Spring 2021 Q.4(b)(ii)

Briefly explain the provisions of the Income Tax Ordinance,2001 and Rules made
thereunder relating to requirement of books of account to be maintained by a
manufacturer having turnover exceeding Rs. 2.5 million. (04)

Question # 5
Spring 2022 Q.4(a)

Briefly explain the term “Sectoral benchmark ratio”. Also, explain the circumstances in
which a commissioner shall determine taxable income on the basis of sectoral
benchmark ratios. (03)

248
Tariq Hussain Tunio

Chapter

18

DEFINITIONS
CHAPTER SYNOPSIS

TOPIC SECTION
Introduction 1
Definitions 2
Associate 85
nd
Women Enterprise 2 schedule
ICAP Past Papers Questions

249
CAF-02 Tax Practices

1. INTRODUCTION

(ii) The law

▪ relating to taxation of income


▪ is entitled as the Income Tax Ordinance, 2001 ('ITO 2001'). §.1(1)

(ii) ITO 2001 extends to the whole of Pakistan. §.1(2)

Explanation for Students


 Pakistan comprises of:
a) Balochistan, KPK, Sindh & the Punjab,
b) Islamabad Capital Territory, and
c) Such States and territories as are or may be included in Pakistan, whether by
accession or otherwise. (Article 1(2) the Constitution)
 ITO-2001 does not apply to
o Azad Jammu and Kashmir (AJ&K) and, Gilgit-Baltistan not being part
of Pakistan

 ITO,2001 is applicable to erstwhile Tribal Areas (FATA/PATA), however, any


income drive therein is exempt from tax from 01-06-2018 to 30-06-2023 (both
days inclusive) [Cl. (145A) of Part I of 2nd Schedule].

(iii) ITO 2001 came into force on 1st July 2002.

This date was appointed by the Federal Government by notification in the official
Gazette (i.e. SRO 381(I)/2002 dated 15.06.2002) §.1(3)

2. DEFINITIONS

4. ACCUMULATED PROFITS §.2(1)

Accumulated profits in relation to distribution or payment of a dividend include:

(a) any reserve

▪ made up wholly or partly of


▪ any allowance, deduction, or exemption
▪ admissible under ITO 2001

(b) all profits of the company

▪ including income and gains


▪ of a trust
▪ up to the date of such distribution or such payment
▪ (for the purposes of clause (19) (a), (d) and (e)); and

(c) includes all profits of the company

▪ including income and gains of


▪ a trust up to the date of its liquidation

250
Tariq Hussain Tunio

(for the purposes clause (19) (c)).

5. APPROVED GRATUITY FUND §.2(3)

Approved gratuity fund means

a gratuity fund approved by the Commissioner in accordance with Part III of the
Sixth Schedule.

6. APPROVED PENSION FUND §.2(3C)

Approved Pension Fund means

Pension Fund
▪ approved by Securities and Exchange Commission of Pakistan (SECP) under
Voluntary Pension System Rules, 2005, and
▪ managed by a Pension Fund Manager registered with the SECP under
Voluntary Pension System Rules, 2005.

7. ASSESSMENT §.2(5)

Assessment includes

▪ provisional assessment, re-assessment, amended assessment


▪ and the cognate expressions shall be construed accordingly.

8. ASSESSMENT YEAR §.2(5A)

Assessment year means

▪ assessment year as defined in the repealed Ordinance.

9. ASSOCIATION OF PERSONS §.2(6)

Association of persons means

▪ an association of persons as defined in section 80.

According to §. 80 “association of persons” includes

▪ a firm,
▪ a Hindu undivided family (HUF),
▪ any artificial juridical person and anybody of persons
o formed under a foreign law,
o but does not include a company. §80(2)(a)

10. BOARD §.2(8)

Board means

▪ the Central Board of Revenue established under the Central Board of


Revenue Act, 1924 (IV of 1924), and

251
CAF-02 Tax Practices

▪ on the commencement of Federal Board of Revenue Act, 2007, the Federal


Board of Revenue established under section 3 thereof.

11. BONUS SHARES §.2(9)

Bonus shares includes bonus units in a unit trust.

12. BUSINESS §.2(10)

Business

▪ includes any

o trade,
o commerce,
o manufacture,
o profession,
o vocation or
o adventure or concern in the nature of
- trade,
- commerce,
- manufacture,
- profession or
- vocation,

▪ but does not include

o employment.

13. BUSINESS BANK ACCOUNT §.2(10A)

Business bank account means

A bank account utilized by the taxpayer for business transaction declared to the
Commissioner through original or modified registration form prescribed under section
181.

14. CAPITAL ASSET §.2(11)

Capital asset means

▪ a capital asset
▪ as defined in section 37 of ITO 2001.

15. CHARITABLE PURPOSE §.2(11A)

Charitable purpose includes

▪ relief of the poor,


▪ education,
▪ medical relief and
▪ the advancement of any other object of general public utility.

16. CONCEALMENT OF INCOME §.2(13AA)

252
Tariq Hussain Tunio

Concealment of income includes –

(a) the suppression of any item of receipt liable to tax in whole or in part, or
failure to disclose income chargeable to tax;
(b) claiming any deduction or any expenditure not actually incurred;
(c) any act referred to in sub-section (1) of section 111; and
(d) claiming of any income or receipt as exempt which is otherwise taxable.

Explanation.- For removal of doubt it is clarified that none of the


aforementioned acts would constitute concealment of income unless it is
proved that taxpayer has knowingly and willfully committed these acts.

17. DEDUCTIBLE ALLOWANCE §.2(16)

Deductible allowance means

an allowance that is deductible from total income under Part IX of Chapter III.

18. DIVIDEND §.2(19)

Dividend includes:

(a) any distribution of accumulated profits,

▪ whether capitalised or not,


▪ by a company to its shareholders

if such distribution

▪ entails the release of


▪ all or any part of the assets of the company
▪ including its money.

(b) any distribution

▪ of debentures, debenture-stock or deposit certificate


▪ in any form,
▪ whether with or without profit,
▪ by a company to its shareholders

to the extent to which

▪ the company possesses


▪ accumulated profits
▪ whether capitalised or not.

(c) any distribution

▪ made to the shareholders of a company


▪ on its liquidation,

to the extent to which

253
CAF-02 Tax Practices

▪ the distribution is attributable


▪ to the accumulated profits of the company,
▪ whether capitalised or not
▪ immediately before its liquidation.

(d) any distribution

▪ by a company to its shareholders


▪ on the reduction of its capital,

to the extent to which

▪ the company possesses accumulated profits,


▪ whether capitalised or not.

(e) payment of any sum

▪ by

o a private company (as defined in Companies Ordinance, 1984) or


o a trust

▪ whether the payment represents

o part of the assets of company or trust, or


o otherwise,

▪ by way of

o advance or
o loan to a shareholder or
o any payment on behalf, or for the individual benefit, of any such
shareholder,

▪ to the extent to which

o the company or trust


o possesses accumulated profits.

(f) remittance of

▪ after tax profit


▪ of a branch
▪ of a foreign company operating in Pakistan.

But does not include:

(i) a distribution made

▪ in accordance with sub-clause (c) or (d)


▪ in respect of
o any share for full cash consideration, or
o redemption of debentures or debenture stock,

254
Tariq Hussain Tunio

▪ where the holder of the share or debenture

o is not entitled to participate in the surplus assets


o in the event of liquidation.

(ii) any advance or loan made

▪ to a shareholder by a company
▪ in the ordinary course of its business,
▪ where the lending of money
o is a substantial part of
o the business of the company.

(iii) any dividend paid

▪ by a company
▪ which is set off by the company
▪ against the whole or any part of
▪ any sum

o previously paid by it and


o treated as a dividend
o within the meaning of sub-clause (e)

▪ to the extent to which it is so set off; and

(iv) remittance of

▪ after tax profit


▪ by a branch of
▪ Petroleum Exploration and Production (E&P) foreign company,
▪ operating in Pakistan.

19. ELIGIBLE PERSON §.2(19A)

Eligible Person, for the purpose of Voluntary Pension System Rules, 2005,
means

an individual Pakistani who holds a valid National Tax Number [or Computerized
National Identity Card [or National Identity Card for Overseas Pakistanis] issued by
the National Database and Registration Authority

the total tax credit available for the contribution made to approved
employment pension or annuity scheme and approved pension fund under Voluntary
Pension System Rules, 2005, should not exceed the limit prescribed or specified in
section 63.

20. EMPLOYEE §.2(20)

Employee means

▪ any individual
▪ engaged in employment

255
CAF-02 Tax Practices

21. EMPLOYER §.2(21)

Employer means

▪ any person
▪ who engages and remunerates an employee.

22. EMPLOYMENT §.2(22)

Employment includes

▪ a directorship or any other office involved in the management of a company;


▪ a position entitling the holder to a fixed or ascertainable remuneration; or
▪ the holding or acting in any public office.

Explanation for Students


Persons holding public office such as the President or the Prime Minister, as
well as the directors of a company are not ordinarily considered as
‘employees’. However, for the purposes of ITO 2001, they are treated as
‘employee’.

23. FEE FOR TECHNICAL SERVICES §.2(23)

Fee for technical services means

▪ any consideration

o whether periodical or lump sum,


o for rendering of

▪ managerial,
▪ technical or
▪ consultancy services
▪ including services of technical or other personnel,

▪ but does not include

o consideration for services

▪ rendered in relation to
▪ a construction, assembly or like project
▪ undertaken by the recipient; or

o consideration

▪ which would be income of the recipient


▪ chargeable under the head “Salary”.

256
Tariq Hussain Tunio

24. INCOME §.2(29)

Income

▪ includes

o any amount chargeable to tax under ITO-2001


o any amount subject to collection or deduction of final tax under the
provisions of ITO-2001 under section 148,150, 152(1), 153, 154, 156,
156A, 233, 234(5), 236Z
o any amount treated as income under any provision of ITO-2001, and
o any loss of income

25. INCOME YEAR §.2(29A)

Income year means

▪ income year
▪ as defined in the repealed Ordinance (i.e. ITO-1979)

26. INDUSTRIAL UNDERTAKING §.2(29C)

Industrial undertaking means

(a) an undertaking which is

▪ set up in Pakistan and


▪ which employs

o 10 or more persons in Pakistan and


o involves the use of electrical energy or any other form of energy
which is mechanically transmitted and is not generated by human
or animal energy; or

o 20 or more persons in Pakistan and


o does not involve the use of electrical energy or any other form of
energy which is mechanically transmitted and is not generated by
human or animal energy:

▪ and which is engaged in

o the manufacture of goods or materials or the subjection of goods


or materials to any process which substantially changes their
original condition; or

o ship-building; or

o generation, conversion, transmission or distribution of electrical


energy, or the supply of hydraulic power; or

o the working of any mine, oil-well or any other source of mineral


deposits; and

257
CAF-02 Tax Practices

(b) any other industrial undertaking which the Board may by notification in the
official gazette, specify.

27. IRIS §.2(30AC)

Iris means

▪ a web based computer programme for operation and


▪ management of Inland Revenue taxes and laws administered by the Board.

28. IT ENABLED SERVICES §.2(30AE)

IT enabled services include

▪ but not limited to inbound or outbound call centres,


▪ medical transcription,
▪ remote monitoring,
▪ graphics design,
▪ accounting services,
▪ Human Resource (HR) services,
▪ telemedicine centers,
▪ data entry operations,
▪ cloud computing services,
▪ data storage services,
▪ locally produced television programs and
▪ insurance claims processing.

29. MEMBER §.2(32)

member in relation to an association of persons, includes


▪ a partner in a firm.

30. MINOR CHILD §.2(33)

Minor child means


an individual who is under the age of eighteen years at the end of a tax year.

31. NON-PROFIT ORGANIZATION §.2(36)

Non-profit organization means

▪ any person other than an individual which is

o established for

- religious,
- educational,
- charitable,
- welfare or
- development purposes, or
- for the promotion of an amateur sport;

o formed and registered

258
Tariq Hussain Tunio

- under any law


- as a non-profit organization

o approved by the Commissioner for specified period

- on an application made by the person in prescribed form and


manner,
- accompanied by prescribed documents and such other documents
as may be required by the Commissioner

▪ and none of the assets of such person

o confers or may confer


o a private benefit to any other person.

32. OFFICER OF INLAND REVENUE §.2(38A)

Officer of Inland Revenue means

▪ any Additional Commissioner Inland Revenue,


▪ Deputy Commissioner Inland Revenue,
▪ Assistant Commissioner Inland Revenue,
▪ Inland Revenue Officer,
▪ Inland Revenue Audit Officer,
▪ District Taxation Officer Inland Revenue,
▪ Assistant Director Audit, or
▪ any other officer
however designated or appointed by the Board for the purposes of this Ordinance.

33. NON-RESIDENT PERSON §.2(37)

Non-resident person means

▪ a non-resident person
▪ as defined in §.81.

34. NON-RESIDENT TAXPAYER §.2(38)

Non-resident taxpayer means

▪ a taxpayer
▪ who is a non-resident person.

35. PERMENANT ESTABLISHMENT §.2(41)

Permanent establishment in relation to a person,

o means

o a fixed place of business


o through which the business of the person is wholly or partly carried on

o and includes

259
CAF-02 Tax Practices

o a place of management,

- branch,
- office,
- factory or workshop,
- premises for soliciting orders,
- warehouse,
- permanent sales exhibition or sales outlet, other than a liaison
office except where the office engages in the negotiation of
contracts other than contracts of purchase;

o a mine,

- oil or gas well,


- quarry or
- any other place of extraction of natural resources;

o an agricultural, pastoral or forestry property;

o a building site,
- a construction, assembly or installation project or
- supervisory activities connected with such site or project
- but only where such site, project and its connected supervisory
activities continue for a period or periods aggregating more than
90 days within any 12 months period

o the furnishing of services, including consultancy services,


- by any person
- through employees or other personnel engaged by the person for
such purpose

o a agent, except when he is independent agent who acts independently in


the ordinary course of business, if the agent

- has and habitually exercises an authority to conclude


contracts on behalf of the other person

- has no such authority, but habitually maintains a stock-


in-trade or other merchandise from which the agent regularly
delivers goods or merchandise on behalf of the other person; or

▪ any substantial equipment installed, or other asset or property capable


of activity giving rise to income.

36. PRINCIPLE OFFICER §.2(44A)

Principal officer used with reference to a company or association of persons includes

(a) a director, a manager, secretary, agent, accountant or any similar


officer; and

260
Tariq Hussain Tunio

(b) any person connected with the management or administration of the


company or AOP upon whom the Commissioner has served a notice of
treating him as the principal officer thereof.

37. PROFIT ON DEBT §.2(46)

Profit on a debt, whether payable or receivable, means

(a) any profit, yield, interest, discount, premium or other amount, owing under a
debt, other than a return of capital; or
(b) any service fee or other charge in respect of a debt, including any fee or
charge incurred in respect of a credit facility which has not been utilized;

38. PRIVATE COMPANY §.2(45)

Private Company means

▪ a company that is not a public company;

39. PUBLIC COMPANY §.2(47)

Public company means

▪ a company

o in which not less than 50% of the shares


o are held by

- Federal Government or
- Provincial Government

▪ a company

o in which not less than 50% of the shares


o are held by
- a foreign Government, or
- a foreign company owned by a foreign Government

▪ a company

o whose shares were traded on a registered stock exchange in Pakistan at


any time in the tax year and
o which remained listed on that exchange at the end of that year; or

▪ a unit trust

o whose units are widely available to the public and


o any other trust as defined in the Trusts Act, 1882.

40. RENT §.2(49)

Rent means

261
CAF-02 Tax Practices

▪ rent as defined in sub-section (2) of section 15 and


▪ includes an amount treated as rent under section 16.

41. RESIDENT COMPANY §.2(50)

Resident company means

▪ a resident company
▪ as defined in section 83.

42. RESIDENT INDIVIDUAL §.2(51)

Resident individual means

▪ a resident individual
▪ as defined in section 82.

43. RESIDENT PERSON §.2(52)

Resident person means

▪ a resident person
▪ as defined in section 81.

44. RESIDENT TAXPAYER §.2(53)

Resident taxpayer means

▪ a taxpayer who is a resident person.

45. ROYALTY §.2(54)

Royalty means any amount paid or payable, however described or computed,


whether periodical or a lump sum, as consideration for —

(a) the use of, or right to use any patent, invention, design or model, secret
formula or process, trademark or other like property or right;

(b) the use of, or right to use any copyright of a literary, artistic or scientific
work, including films or video tapes for use in connection with television or
tapes in connection with radio broadcasting, but shall not include
consideration for the sale, distribution or exhibition of cinematograph films;

(c) the receipt of, or right to receive, any visual images or sounds, or both,
transmitted by satellite, cable, optic fibre or similar technology in connection
with television, radio or internet broadcasting;

(d) the supply of any technical, industrial, commercial or scientific


knowledge, experience or skill;
(e) the use of or right to use any industrial, commercial or scientific
equipment;

262
Tariq Hussain Tunio

(f) the supply of any assistance that is ancillary and subsidiary to, and is
furnished as a means of enabling the application or enjoyment of, any such
property or right as mentioned in subclauses (a) through (e); and

(g) the disposal of any property or right referred to in sub-clauses (a) through (e).

46. TAX §.2(63)

Tax
▪ means

any tax imposed under Chapter II,

▪ and includes
o any penalty,
o fee or
o other charge or
o any sum or
o amount leviable or payable
under this Ordinance.

47. TAXPAYER §.2(66)

Taxpayer
▪ means

o any person who derives an amount chargeable to tax under this


Ordinance,

▪ and includes

o any representative of a person who derives an amount chargeable to tax


under ITO-2001

o any person who is required to deduct or collect tax under ITO-2001, or

o any person required to furnish a return of income or pay tax under ITO-
2001.

48. TAX YEAR §.2(68)

Tax year

▪ Means
o the tax year as defined in sub-section (1) of section 74

▪ and, in relation to a person, includes


o a special year or
o a transitional year

that the person is permitted to use under section 74

263
CAF-02 Tax Practices

Definition of Association of Persons

2. Association of Persons includes

▪ a firm
▪ a Hindu undivided family (HUF)
▪ any artificial juridical person and anybody of persons formed under a foreign
law but does not include a company. §.80(2) (a)

Definition of Firm

3. Firm means:

▪ the relation between persons


▪ who have agreed to share the profits of a business
▪ carried on by all or any of them acting for all. §.80(2)(c)
Definition of Company

4. Company means:

(ii) a company
▪ as defined in Companies Ordinance, 1984

(iii) a body corporate


▪ formed by/ under
▪ any law in force in Pakistan

(iv) a modaraba

(v) a body
▪ incorporated by/ under a law of a foreign country
▪ relating to incorporation of companies (i.e. foreign company)

(v) ▪ a co-operative society,


▪ a finance society or
▪ any other society
o established or constituted
o by/ under any law

(vi) a non-profit organization

(vii) ▪ a trust,
▪ an entity or a body of persons
o established or constituted
o by or under any law for the time being in force

(viii) a foreign association


▪ incorporated or not,
▪ that is declared by the FBR
o by general or special order
▪ to be a company for the purposes of ITO 2001

(ix) a Provincial Government

264
Tariq Hussain Tunio

(x) a Local Government in Pakistan, or

(xi) a small company (as defined in §. 2 of ITO-2001) §.80(2)(b)

Definition of Trust

5. Trust means:

▪ an obligation annexed to the ownership of property and


▪ arising out of the confidence reposed in and accepted by the owner, or
▪ declared and accepted by the owner for the benefit of another, or
▪ of another and the owner, and
▪ includes a unit trust. §.80(2)(d)
Definition of Unit Trust

6. Unit trust means any trust under which:

▪ beneficial interests are divided into units


▪ such that the entitlements of the beneficiaries to income or capital
▪ are determined by the number of units held. §.80(2)(e)

3. ASSOCIATE §.85

Associates or associated persons mean:

(i) two persons who are associated with each other in a way that their relationship is
such that

▪ one may reasonably be expected to act in accordance with the intentions of


the other, or
▪ both persons may reasonably be expected to act in accordance with the
intentions of a third person.

Chart# 1► ASSOCIATE

A A
Associates

X Y

X
Associates

➢ Two persons shall not be associates solely by reason of the fact that

▪ one person is an employee of the other or


▪ both persons are employees of a third person.

(ii) an individual and a relative of the individual.

265
CAF-02 Tax Practices

An individual and a relative shall not be associates if the Commissioner is


satisfied that neither person may reasonably be expected to act in accordance
with the intentions of the other.

“Relative” means:

(a) ▪ an ancestor,
▪ a descendant of any of the grandparents, or
▪ an adopted child

o of the individual, or
o of the individual's spouse; or

(b) a spouse

▪ of the individual or
▪ of any of the above persons.

(iii) members of an association of persons

Members of an AOP shall not be associates if the Commissioner is satisfied that


neither person may reasonably be expected to act in accordance with the
intentions of the other.

Chart# 2► ASSOCIATE

AOP

Member 'X' Member 'Y'

Associates
``

(iv) a member of an AOP and the association where the member

▪ either alone or together with an associate or associates


▪ under another application,
▪ controls fifty per cent or more of the rights to income or capital of the
association.

266
Tariq Hussain Tunio

Chart# 3► ASSOCIATE

AOP
AOP

Associates
Associates

Member 'X' Member 'X'


Member 'X' with 30% right to with 25% right to
with≥ 50% right to income/capital income/capital
income/capital

Relatives

(v) a trust and any person who benefits or may benefit under the trust

(vi) a shareholder in a company and the company,


▪ where the shareholder,

▪ either alone or together with an associate or associates under another


application,

▪ controls either directly or through one or more interposed persons:

• 50% or more of the voting power in the company;


• 50% or more of the rights to dividends; or
• 50% or more of the rights to capital; and

Chart# 4► ASSOCIATE

COMPANY
COMPANY
Associates
Associates

Shareholder 'X' Shareholder 'X' Shareholder 'Y'


alone controls ≥50%
of voting powers/rights
to dividend/rights to
capital 'X', together with associate
'Y' controls ≥50% of voting
powers/rights to
dividend/rights to capital

(vii) two companies


▪ where a person,
▪ either alone or together with an associate or associates
▪ under another application,
▪ controls either directly or through one or more interposed persons:
• 50% or more of the voting power in both companies;

267
CAF-02 Tax Practices

• 50% or more of the rights to dividends in both companies; or


• 50% or more of the rights to capital in both companies.
Chart# 5► ASSOCIATE

Associates

COMPANY 'A' COMPANY 'B'

Shareholder 'X'
alone/ together with an
associate controls ≥50%
of voting powers/rights to
dividend/rights to capital
in both the companies

4. TAXATION OF WOMEN ENTERPRISES Cl 19 Part III of 2ndSchedule

Definition:
woman enterprise

▪ means

a startup established on or after first day of July 2021 as sole proprietorship concern
owned by a woman or an AOP all of whose members are women or a company
whose 100% shareholding is held or owned by women.

Taxation:

The tax payable by woman enterprises on profit and gains derived from business
chargeable to tax under the head “Income from Business” shall be reduced by 25%.

Provided that benefit of this clause shall not be available to a business


that is formed by the transfer or reconstitution or reconstruction or splitting up of an
existing business.

268
Tariq Hussain Tunio

PAST EXAMINATION QUESTIONS


Question # 1
Spring 2022 Q.4( c )

Following information pertains to three unlisted companies:

Company Paid up Total Annual Shareholders


capital reserves turnover
Rs. in million
A limited 30 80 150 60% shares are held by a foreign
company
B limited 80 (35) 220 40% shares are held by the Provincial
and Federal governments
C limited 40 5 500 100% shares are held by a local group

Required:

Under the provisions of the Income Tax Ordinance,2001 briefly discuss whether each of the
above companies can be classified as small, public or private. Also state the additional
information, if any, which may be required for determining the classification of these
companies.(07)

Question # 2
Spring 2021 Q.1(b)

Dr. Jamal is planning to establish a hospital as a non-profit organization.

Required:
Discuss the conditions that should be compiled with by Dr. Jamal, under the ITO,2001.(03)

Question # 3
Spring 2019 Q.5

a. Briefly discuss the difference between a public company and a private company,
within the meaning of ITO,2001.(04)
b. Certain types of payments by a private company to its shareholders can be treated
as ‘dividend’ under the ITO,2001. State the conditions necessary for the application
of this rule and the exceptions to such rule. (05)

Question # 4
Autumn 2018 Q.4(a)

On 25 August 20x8, the officier of Inland Revenue has issued a notice to Rahat Foods
(Private) Limited (RFPL) to deposit withholding income tax of Rs. 1,950,000 in respect of
loan amounting to Rs. 13,000,000 given to Nadeem Ahmad, a shareholder of RFPL, by
treating the amount of loan as dividend. The notice was served to the company on 30
August 20x8.

269
CAF-02 Tax Practices

According to RFPL’s records, the loan was given to Nadeem Ahmad on 25 May 20x7 when
accumulated profit of the company was Rs. 12,000,000.

In the light of the provisions of the ITO,2001 explain whether you agree with the notice
issued to RFPL by the Officer of Inland Revenue. (03)

Question # 5
Autumn 2016 Q.3(b)

Under the provisions of the Income Tax Ordinance, 2001 describe the following:

• meaning of the term 'Associates'. (02)

• circumstances in which a member of an association of persons and the association


may be regarded as associates. (02)

• situation in which members of an association of persons may not be regarded as


associates. (02)

Question # 6
Autumn 2014 Q.2(b)

Briefly discuss the provisions of Income Tax Ordinance, 2001 in respect of the following
situation: ABC (Private) Limited has decided to provide a loan of Rs. 5 million to one of its
shareholders, for the purchase of a house. (04)

Question # 7
Spring 2014 Q.7

Certain payments made by a private limited company to its shareholders can be treated as
'dividend'. Explain the above in the context of Income Tax Ordinance, 2001. Also identify
the exceptions to this rule. (07)

Question # 8
Spring 2014 Q.2(b)

Under the provisions of ITO,2001:


Explain the term 'Associates'. State the circumstances under which a shareholder in a
company and the company may be regarded as associates. (05)

Question # 9
Autumn 2011 Q.4(b)

Explain Industrial Undertaking as specified in the ITO,2001.

Question # 10
Autumn 2013 Q.5 (a)

270
Tariq Hussain Tunio

Differentiate between ‘Public Company’ and ‘Private Company’ within the meaning of
ITO,2001. (05)

Question # 11
Spring 2012 Q.6 (b)

Unde the ITO,2001 where a person is reasonably expected to act in accordance with the
intentions of another person, both persons are considered as associates.

Required:
(i) explain the term ‘’person’’ in the above context.(03)
(ii) State the circumstances in which a company and its shareholder shall be considered
as associates. (04)

Question # 12
Spring 2012 Q.5(a)

Tamba Pakistan (Pvt.) Limited is engaged in the manufacture of pharmaceutical products. Its
board of directors has approved a 3-year loan to one of its major shareholders.

Required:
Explain the tax implications of the above transactions on the company as well as the
shareholders. (04)

Question # 13
Autumn 2009 Q.2(a)

A company engaged in manufacturing activities has decided to provide loan to one of its
shareholders. Explain the tax implications on the company as well as the shareholder if the
company:
(i) Is registered under the Companies Act 2017 as a private limited company.
(ii) Is an unlisted public company. (06)

Question # 14
Autumn 2008 Q.5(b)

State the circumstances when two companies shall be considered as associates, under the
ITO,2001.(04)

Question # 15
Spring 2008 Q.4(b)

Certain types of payments by a private company to their shareholders can be treated as


‘’dividend’’ under the ITO,2001. State the conditions necessary for the application of this rule
and execptions to it. (05)

271

You might also like